Chemical Processes

Réussis tes devoirs et examens dès maintenant avec Quizwiz!

Qualitatively Comparing the distance between the peaks in the graph, which tells us the difference in boiling points for the compounds Quantitatively The area of each peak is directly proportional to the concentration of the compound in the sample.

How are gas chromatograms measured qualitatively and quantitatively?

Internal energy of a system increases when heat is added or work is being done on the system. Internal energy of a system decreases when heat is removed or work is being done by the system.

How can internal energy of a system be increased? How about decreased?

Multiply mOsm/L by 10^-3

How can mOsm/L be converted to mol/L?

By increasing the number of moles of the buffer in comparison to the moles of the strong acid/base.

How can the effectiveness of a buffer be strengthened?

An Arrhenius acid can be identified because their formula begins with an H. An Arrhenius base can be identified because their formula ends with an OH.

How can you identify whether or not an acid or base is an Arrhenius one?

The components separate by themselves based off of their densities. The layer with the greater density will settle at the bottom of the separatory funnel, while the layer with the lesser density will stay on top of the more dense layer.

How do the different components of a mixture separate in extractions?

Both values are equal to one mole of any given substance.

How do the numbers of molecules in 18 g of H2O compare to the number of formula units in 58.5 g of NaCl?

Step 1: Balance the reaction CH4 + 2O2 ---> CO2 + 2H2O Step 2: Find grams of H2O 25 g CH4 * 1 mol CH4 = 1.563 mol CH4 16 g CH4 1.563 mol CH4 * 2 mol H2O = 3.126 mol H2O 1 mol CH4 3.126 mol H2O * 18 g H2O = 56.268 g H2O 1 mol H2O

How many grams of H2O will be produced when you burn 25 g of methane? CH4 + O2 ---> CO2 + H2O

The equation is already balanced. 71.7 g AgCl * 1 mol AgCl * 1 mol CaCl2 * 111 g CaCl2 = 27.8 g CaCl2 143.4 g AgCl 2 mol AgCl 1 mol CaCl2

How many grams of calcium chloride are needed to prepare 71.7 g of silver chloride according to the following equation: CaCl2 + 2AgNO3 ---> Ca(NO3)2 + 2AgCl

1 cal = 4.184 J

How many joules are in a calorie (cal)?

Answer: C. Use the equivalence point equation: N(a)V(a) = N(b)V(b). Ba(OH)2 can dissociate to give two hydroxide ions, so its normality is 2 M * 2 = 4 N. H3PO4 can dissociate to give three hydronium ions, so its normality is 6 M * 3 = 18 N. Plugging into the equation, we get: (18 N)(4 L) = (4 N)(V(b)). Therefore, V(b) is 18 L.

How many liters of 2 M Ba(OH)2 are needed to titrate a 4 L solution of 6 M H3PO4? A. 1.33 L B. 12 L C. 18 L D. 56 L

Mg = 24.3 g/mol Cl = 35.5 * 2 = 71 g/mol 71 + 24.3 = 95.3 g/mol 9.53 g * 1 mol = 0.1 mol 95.3 g

How many moles are in 9.53 g of MgCl2?

Answer: D. The question is simple if one recalls that periods refers to the rows in the periodic table, while groups or families refer to the columns. Within the same period, an additional valence electron is added with each step towards the right side of the table.

How many valence electrons are present in elements in the third period? A. 2 B. 3 C. The number decreases as the atomic number increases D. The number increases as the atomic number increases

m = moles solute / kg solvent

How molality is calculated: m = _________ solute/________ solvent

M = moles of solute/liters of solution

How molarity of a solute is calculated: M = _________ of solute/_________ of solution

X = moles of a specific species/total moles of all species

How mole fraction is calculated: X = #moles of _____________________/#moles of __________________

N = #equivalents * molarity

How normality is calculated: N = _______________ * ________________

From left to right: 1. sp2 trigonal planar 120° 2. sp3 tetrahedral 109.5° 3. sp3 tetrahedral 109.5° 4. sp3 trigonal planar 107°

Identify the hybridization states, predict the molecular geometries, and predict the bond angle for all atoms (except for any that are bonded to only one atom).

From left to right: 1. sp2 trigonal planar 120° 2. sp2 trigonal planar 120° 3. sp3 tetrahedral 109.5° 4. sp linear 180° 5. sp linear 180°

Identify the hybridization states, predict the molecular geometries, and predict the bond angle for all atoms (except for any that have only one bond).

1. Alpha 2. Beta 3. Gamma 4. Delta 5. Alpha 6. Beta 7. Gamma 8. Delta

Identify the numbered carbons in relation to the carbonyl carbon.

Assign oxidation numbers: 0 0 +1 -1 2Na + Cl2 -------> 2NaCl Na loses electrons, which makes it undergo oxidation. Therefore, it is the reducing agent. Cl gains an electron, which makes it undergo reduction. Therefore, it is the oxidizing agent.

Identify the oxidizing and reducing agent in the following redox reaction: 2Na + Cl2 -------> 2NaCl

From top to bottom Bonding Antibonding Bonding Bonding

Identify the type of molecular orbital formed in the image.

Red = Isobaric Yellow = Isothermal Green = Adiabatic Blue = Isochoric

Identify the types of thermodynamic processes shown on the PV diagram.

Strong Non Weak Weak Non Non Strong Weak Strong

Identify whether the following compounds are strong electrolytes, weak electrolytes, or nonelectrolytes: NaCl CO2 (g) Hg2I2 NH3 O2 (g) C6H12O6 HCl CH3COOH KI

Affinity chromatography

Identify which type of chromatography is being performed here.

Ion-exchange chromatography

Identify which type of chromatography is being performed here.

Paper chromatography

Identify which type of chromatography is being performed here.

Size exclusion chromatography

Identify which type of chromatography is being performed here.

Step 1: Balance the reaction 2KClO3 ---> 2KCl + 3O2 Step 2: Find grams of oxygen 138.6 g KClO3 * 1 mol KClO3 = 1.131 mol KClO3 122.545 g KClO3 1.131 mol KClO3 * 3 mol O2 = 1.697 mol O2 2 mol KClO3 1.697 mol O2 * 32 g O2 = 54.304 g O2 1 mol O2

If 138.6 g of KClO3 is heated and decomposes completely, how many grams of oxygen gas would be produced? KClO3 ---> KCl + O2

alkane

If ______________ is not the highest priority functional group in an organic molecule, then the molecule will have a prefix of alkyl- (i.e.:methyl).

amide

If ______________ is not the highest priority functional group in an organic molecule, then the molecule will have a prefix of carbamoyl- or amido-.

carboxylic acid

If ______________ is not the highest priority functional group in an organic molecule, then the molecule will have a prefix of carboxy-.

alcohol

If ______________ is not the highest priority functional group in an organic molecule, then the molecule will have a prefix of hydroxy-.

ketone

If ______________ is not the highest priority functional group in an organic molecule, then the molecule will have a prefix of oxo- or keto-.

aldehyde

If ______________ is not the highest priority functional group in an organic molecule, then the molecule will have a prefix of oxo-.

Aldehyde

If _________________ is the highest priority functional group in an organic molecule, then the molecule will have a suffix of -al.

Alkane

If _________________ is the highest priority functional group in an organic molecule, then the molecule will have a suffix of -ane.

Alkene

If _________________ is the highest priority functional group in an organic molecule, then the molecule will have a suffix of -ene.

Ester

If _________________ is the highest priority functional group in an organic molecule, then the molecule will have a suffix of -oate.

Alcohol

If _________________ is the highest priority functional group in an organic molecule, then the molecule will have a suffix of -ol.

Answer: C. The highest priority functional group in this molecule is the carboxylic acid, so this will be a component of the backbone and provides the suffix of the molecule. This molecule is 2-methyl-3-oxopropanoic acid.

If all prefixes were dropped, what would be the name of the parent root of this molecule? A. Propanoate B. Propanol C. Propanoic acid D. Propanoic anhydride

3 eV * (1.60 x 10^-19 J) = 4.8 x 10^-19 J E = h*c/λ or λ = h*c/E λ = 6.626 x 10^-34 J*s * 3 x 10^8 m/s/4.8 x 10^-19 J λ = 4.14 x 10^-7 m = 414 x 10^-9 m = 414 nm

If an electron emits 3 eV of energy, what is the corresponding wavelength of the emitted photon? (Note: 1 eV = 1.60 x 10^-19 J, h = 6.626 x 10^-34 J*s)

Exothermic.

If intermolecular interactions prior to dissolution are weaker than intermolecular interactions formed by dissolution, is the reaction will be _________________.

Not affect

If the anion in a solution is a conjugate of a strong acid, would the addition of an acid increase, decrease, or not affect the solubility of the solute

Electronegative

In a polar covalent bond, the atom that has a greater attraction of the shared electrons is the atom that is more _______________________.

Acid Base

In a titration between a titrant and a titrand, usually one is a ___________ while the other is a ______________.

Q = mcΔT m = 2 kg = 2000 g c = 4.184 J/gK Tf = 100°C Ti = 20°C Q = 2000 g * 4.184 J/gK * (100°C - 20°C) Q = 669440 J Q(vap) = m * L(vap) m = 2000 g L(vap) = 2260 J/g Q(vap) = 2000 g * 2260 J/g Q(vap) = 4520000 J 669440 J + 4520000 J = 5189440 J

Say that we have a container of water that contains 2 kg of water and is sitting at a temperature of 20°C. How much heat do we need to add if we want to vaporize the water into steam? (Specific heat of water = 4.184 J/g*K) (Boiling point of water = 100°C) (Heat of vaporization for water = 2260 J/g)

-Phosphoric acid -Phosphate

Name the acid and it's anion: -H3PO4 -PO4(3-)

-Hydrobromic acid -Bromide

Name the acid and it's anion: -HBr -Br-

-Hydrochloric acid -Chloride

Name the acid and it's anion: -HCl -Cl-

-Hypochlorous acid -Hypochlorite

Name the acid and it's anion: -HClO -ClO-

-Chlorous acid -Chlorite

Name the acid and it's anion: -HClO2 -ClO2-

-Chloric acid -Chlorate

Name the acid and it's anion: -HClO3 -ClO3-

-Perchloric acid -Perchlorate

Name the acid and it's anion: -HClO4 -ClO4-

-Hydrofluoric acid -Fluoride

Name the acid and it's anion: -HF -F-

-Nitrous acid -Nitrite

Name the acid and it's anion: -HNO2 -NO2-

-Nitric acid -Nitrate

Name the acid and it's anion: -HNO3 -NO3-

Bromite

Name the common polyatomic ion: BrO2-

Bromate

Name the common polyatomic ion: BrO3-

Carbonate

Name the common polyatomic ion: CO3(2-)

Bicarbonate (hydrogen carbonate)

Name the common polyatomic ion: HCO3-

Biphosphate (hydrogen phosphate)

Name the common polyatomic ion: HPO4(2-)

Bisulfate (hydrogen sulfate)

Name the common polyatomic ion: HSO4-

Permanganate

Name the common polyatomic ion: MnO4-

Ammonium

Name the common polyatomic ion: NH4+

Phosphate

Name the common polyatomic ion: PO4(3-)

Thiosulfate

Name the common polyatomic ion: S2O3(2-)

Propanamide

Name the molecule.

ethanoic propanoic anhydride

Name the molecule.

Polar covalent bonds

The following are properties of _______________________ bonds: -Occurs only between atoms of different elements -Electronegativity difference between bonded atoms is moderate (between 0.5 and 1.7 in the Pauling scale) -Electrons are not shared equally between atoms

energy absorb/emit E = Energy for transition E1 = Rydberg unit of energy (-2.18 x 10^-18 J/-13.6 eV) n(i) = initial (starting) energy level n(f) final (ending) energy level

The following equation is used to determine the ____________ that an electron needs to __________________ to jump from one energy state to another: E = E1 * [(1/(n(i)^2)) - (1/(n(f)^2))] E = _______________________ E1 = ____________________________ n(i) = _____________________________ n(f) = ______________________________

Work function of the metal Planck's constant (6.626 x 10^-34 J*s/4.14 x 10^-15 eV*s) Threshold frequency of metal (given)

The following equation is used to find the ______________________ of a __________________: E° = h*f(T) E° = ___________________ h = ______________________ f(T) = ________________________

angular momentum electron v = velocity of an electron n = energy level of the valence electron h = Planck's constant (6.626 x 10^-34 J*s/4.14 x 10^-15 eV*s) m = mass of electron (9.11 x 10^-31 kg) r = atomic radius

The following equation is used to find the ______________________ of a ____________________: vmr = (nh)/(2π) v = ________________________ n = _________________________ h = _________________________ m = _______________________ r = ________________________

Bond length (single > double) Bond energy (double > single) Free rotation (single not double)

What differences would be observed in a molecule containing a double bond to the same molecule containing only a single bond?

All salts containing NH4+ (ammonium) cations or alkali metal cations are water soluble.

What do the rules of solubility say in regards to salts containing ammonium (NH4+) and alkali metal?

Aldehyde

What functional group is being shown here?

Alkene

What functional group is being shown here?

Alkyl Halide

What functional group is being shown here?

Carboxylic acid

What functional group is being shown here?

amide

What functional group is being shown here?

amine

What functional group is being shown here?

Heat will flow from the warmer object to the cooler object.

What happens when two objects of differing temperatures make thermal contact with one another?

r(n) = n^2 * r1 r1 = 5.3 x 10^-11 m n = 1 r(n) = 1^2 * 5.3 x 10^-11 m r(n) = 5.3 x 10^-11 m r(n) = n^2 * r1 r1 = 5.3 x 10^-11 m n = 2 r(n) = 2^2 * 5.3 x 10^-11 m r(n) = 2.12 x 10^-10 m r(n) = n^2 * r1 r1 = 5.3 x 10^-11 m n = 3 r(n) = 3^2 * 5.3 x 10^-11 m r(n) = 4.77 x 10^-10 m

What is the atomic radius of an atom with its valence electron at an energy level of n=1? What is the atomic radius of an atom with its valence electron at an energy level of n=2? What is the atomic radius of an atom with its valence electron at an energy level of n=3?

-NaOH ---> Na+ -CaOH ---> Ca+ -NH3 ---> NH4+ -Mg(OH)2 ---> MgOH+ -NH4OH ---> NH4+ -KOH ---> K+

What is the conjugate acid of the following bases? -NaOH -CaOH -NH3 -Mg(OH)2 -NH4OH -KOH

Historically, HPLC was performed at high pressures, whereas column chromatography uses gravity to pull the solution through the column. Now, HPLC is performed with sophisticated and variable solvent and temperature gradients, allowing for much more specific separation of compounds than column chromatography; high pressures are no longer required.

What is the major historical distinction between HPLC and column chromatography. What is the major distinction now?

Linear

What is the molecular geometry of BeCl2?

Square Pyramidal

What is the molecular geometry of BrF5?

Tetrahedral

What is the molecular geometry of CH4?

Shape: Square Planar Angles: 90°

What is the molecular geometry of this molecule? What are the bond angles of this molecule?

Shape: Square Pyramidal Angles: 90°

What is the molecular geometry of this molecule? What are the bond angles of this molecule?

Shape: T-shaped Angles: 90°

What is the molecular geometry of this molecule? What are the bond angles of this molecule?

Shape: Trigonal Bipyramidal Angles: 90° (axial-equatorial) 120° (equatorial-equatorial)

What is the molecular geometry of this molecule? What are the bond angles of this molecule?

Shape: Trigonal Planar Angles: 120°

What is the molecular geometry of this molecule? What are the bond angles of this molecule?

Shape: Trigonal Pyramidal Angles: 107°

What is the molecular geometry of this molecule? What are the bond angles of this molecule?

S: 32.1 g/mol O: 16 g/mol Cl: 35.5 * 2 = 71 g/mol 71 + 16 + 32.1 = 119.1 g/mol

What is the molecular weight of SOCl2?

Ethane thiol

What is the name of this molecule

1-Butene

What is the name of this molecule?

2-Pentanone (methyl-propyl ketone)

What is the name of this molecule?

Butyramide/Butanamide

What is the name of this molecule?

Chloroethane

What is the name of this molecule?

Cyclohexane

What is the name of this molecule?

Mass of solute = 20 g NaCl (s) Mass of solution = 100 g NaCl (aq) 20 g/100 g * 100% = 20% percent composition by mass = 20% NaCl solution

What is the percent composition by mass of a salt water solution if 100 g of the solution contains 20 g of NaCl?

From pKa - 1 to pKa + 1

What is the range of a titration curve where the titrand can serve as a buffer?

1 cal/g*K 4.184 J/g*K

What is the specific heat for water? (both units of measurements)

If two objects are in thermal contact with each other, and there is no net heat flow between the two objects, then they are in thermal equilibrium.

When are two objects in thermal equilibrium?

Name Numbers

When assigning numbers to substituents, you must pair the __________ of the substituents with their corresponding ________________ in the parent chain.

10:1 1:10

A buffer is considered to be effective when the HA/A- ratio ranges anywhere between __________ or _____________

There is no net dipole moment, since the individual dipole moments cancel each other out.

Determine the net dipole moment of the following molecule.

Water: M: H2O E: H2O Sulfur: M: S8 E: S Benzene: M: C6H6 E: CH

Give both the molecular and empirical formula for water. Give both the molecular and empirical formula for sulfur. Give both the molecular and empirical formula for benzene.

Less

When determining molecular geometry, the geometry with the ______ overall repulsion is preferable.

σ-bonds π-bonds

A head-on overlap of two orbitals are known as ______________. A side-by-side overlap of two unhybridized p-orbitals are known as ______________.

Paper chromatography Thin layer chromatography Simple column chromatography

Which types of chromatographies fall under the category of liquid chromatography?

Molecular weight Polarity Solubility Temperature of the column

While boiling point is the main factor in elution times in gas chromatography, what are some other factors that may affect elution time?

First ionization energy Second ionization energy

_________ ionization energy is the energy needed to remove one electron from an atom. _________ ionization energy is the energy needed to remove a second electron from an atom after already removing one.

14.

pKa + pKb = _________

Optical activity

________________ is the ability for a chiral molecule to rotate the plane of oscillation of plane-polarized light in a certain direction (clockwise or counterclockwise).

Elution

________________ is the process of washing out a compound through a stationary phase using a suitable mobile phase.

Enantiomers

_________________ are configurational isomers that are non-superimposable mirror images of each other.

substituents

___________________ are functional groups that are not part of the parent chain.

Aldehydes Ketones

___________________ are hydrocarbons that has a carbonyl (C=O) group at the end of a parent chain. __________________ are hydrocarbons that has a carbonyl (C=O) group somewhere in the middle of a parent chain.

Structural/constitutional isomers

___________________ are isomers that have the same molecular formula, but different structures.

Intermolecular forces

___________________ forces are able to impact the boiling point, melting point, enthalpy of fusion, and enthalpy of vaporization of a molecule.

Aufbau principle Pauli Exclusion Principle Hund's Rule

____________________ is the concept that electrons fill energy levels in order of increasing energy, completely filling one sublevel before beginning to fill the next. _____________________ is the concept that no 2 electrons in an atom can have the same 4 quantum numbers. ____________________ is the concept that electrons will fill into separate orbitals with parallel spins before pairing in an orbital to minimize electron repulsion.

Decomposition reaction

______________________ is a reaction in which a single reactant breaks down into two or more products

SIngle displacement reaction

______________________ is a reaction where an atom or ion in a compound is replaced by an atom or ion of another element.

Enantiomeric excess (ee) greater concentration lesser concentration

_________________________ is the degree to which a sample contains one enantiomer in excess over the other. It is calculated by subtracting the percentage of the enantiomer of _____________________ by the percentage of the enantiomers of _____________________.

Physical properties Chemical properties

__________________________ of a molecule are characteristics of a molecule that do not change the chemical composition of the molecule. ____________________________ of a molecule are characteristics of a molecule that changes the chemical composition during a reaction.

An alkyne

Which functional group consists of only a carbon-carbon triple bond?

Answer: B. This molecule is a chair conformation in which the two equatorial methyl groups are trans to each other. Because the axial methyl hydrogens do not compete for the same space as the hydrogens attached to the ring, this conformation ensures the least amount of steric strain. (A.) would be less stable because the diaxial methyl groups of hydrogens are closer to the hydrogens on the ring, causing greater steric strain. (C.) is incorrect it is in the more unstable boat conformation.

Which isomers of the following compound is the most stable?

Phenolphthalein

Based on the curve shown, what would be the best indicator to use for this titration?

bromthymol blue

Based on the curve shown, what would be the best indicator to use for this titration?

5,5-dimethylhexanal

Name the following molecule.

Greater Less

A solution would be considered acidic if the concentration of hydronium ions is _____________ than the concentration of hydroxide ions. A solution would be considered basic if the concentration of hydronium ions is _____________ than the concentration of hydroxide ions.

Titrand/Analyte

A ___________________ is the solution in a titration reaction whose concentration is unknown.

Ideal solution

A _________________________ is a solution with an enthalpy of dissolution that is equal to zero.

paramagnetic attracted to diamagnetic repelled by

A __________________________ element is an element that has at least one or more orbitals with unpaired electrons, causing it to be __________________________ a magnetic field. A _________________________ element is an element that has only paired electrons in its orbtials, which produces

Linear

A molecule that has five electron clouds, where three are nonbonding electron pairs and two are bonding electron pairs, would have a ________________ molecular geometry.

Answer: A. (+)-Glyceraldehyde and (-)-glyceraldehyde, or (R)- and (S)-2,3-dihydroxypropanol, are enantiomers. Enantiomers are nonsuperimposable mirror images. Each has only one chiral center (C-2), which has opposite absolute configuration in these two molecules.

(+)-Glyceraldehyde and (-)-glyceraldehyde refer to the (R) and (S) forms of 2,3-dihydroxypropanal, respectively. These molecules are considered: A. Enantiomers B. Diastereomers C. Meso compounds D. Structural isomers

Cis-isomer

A _____________________ is a geometric isomer that has two identical substituents on the same side of a double bond or a ring structure.

Weak acid-strong base titration

What type of titration reaction is shown in the titration curve above?

Weak acid-weak base titration

What type of titration reaction is shown in the titration curve above?

Weak base-strong acid titration

What type of titration reaction is shown in the titration curve above?

Weak base-weak acid titration

What type of titration reaction is shown in the titration curve above?

If a strong base is added, the acid in the buffer will neutralize the OH- ions. If a strong acid is added, the base in the buffer will neutralize the H3O+ ions.

What will happen if a strong base is added to a buffer solution? What about a strong acid?

#stereoisomers = 2^n n = #chirality centers

How is the number of stereoisomers in a molecule calculated?

15.4 mg = 0.0154 g 0.0154 g * 1 mol * 6.022 x 10^23 atoms = 1.277 x 10^20 atoms 72.63 g 1 mol

How many atoms are in a 15.4 mg sample of germanium?

Acceptable answers: -Group 1 metals -Ammonium (NH4+) -Nitrate (NO3-) -Acetate (CH3COO-)

Name two ions that form salts that are always soluble.

Meth- Eth- Prop- But- Greek Roots

Names for the first four alkanes have the prefix ____________ (one carbon), _____________ (two carbons), ________________ (three carbons), and ______________ (four carbons). Alkanes with more than four carbons uses ______________ _____________ to describe the number of carbons present.

London dispersion forces.

Noble gases are able to have melting and boiling points thanks to ________________________. _________________________ are present in the solid and liquid phases, but are negligible in gas phases.

1. 2 - one particle of Na+ and one particle of Cl- 2. 4 - one particle of Ag(3+) and three particles of Cl- 3. 3 - one particle of Pb(2+) and two particles of Cl-

1. What is the van't Hoff factor of NaCl? 2. What is the van't Hoff factor of AgCl3? 3. What is the van't Hoff factor of PbCl2?

ΔT(b) = i*K(b)*m i=4 since AlCl3 dissociates into 1 particle Al(3+) and 3 particles Cl-. m = mol solute/kg solvent 1.5 L water = 1500 mL water * 1 g/mL = 1500 g water = 1.5 kg water 400 g AlCl3/133 g/mol AlCl3 = 3 mol AlCl3 3 mol AlCl3/1.5 kg water = 2 mol/kg m = 2 mol/kg ΔT(b) = 4 * 0.512 K*kg/mol * 2 mol/kg ΔT(b) = 4 K

400 g AlCl3 is dissolved in 1.5 L of water at room temperature (Kb = 0.512 K*kg/mol). How much does the boiling point increase after adding thee AlCl3?

σ π

A _______ bond will form from a head-on overlap of an s, p, or hybridized orbital. A ________ bond will form only from the side-to-side overlap of 2 unhybridized p-orbitals.

Oxidizing agent Reducing agent

A ___________ agent undergoes a reduction reaction A ___________ agent undergoes an oxidation reaction

Dilute Concentrated

A ____________ solution is a solution in which the proportions of solute to solvent is considerably small. A __________________ solution is a solution in which the proportions of solute to solvent is considerably large.

Single Double Triple

A _____________ covalent bond is formed by the sharing of 1 electron pair. A ____________ covalent bond is formed by the sharing of 2 electron pairs. A ____________ covalent bond is formed by the sharing of 3 electron pairs.

Polyprotic species

A ______________ species is a molecule that is capable of donating more than one proton.

Answer: B. In a carbon with one double bond, sp2 hybridization occurs - that is, one s-orbital hybridizes with two p-orbitals to form three sp2-hybridized orbitals. The third p-orbital of the carbon atom remains unhybridized and takes part in the formation of the π bond of the double bond. Although there is an unhybridized p-orbital, there are no unhybridized s-orbitals, eliminating (D.).

A carbon atom participates in one double bond. As such, this carbon contains orbitals with: A. hybridization between the s-orbital and one p-orbital B. hybridization between the s-orbital and two p-orbitals C. hybridization between the s-orbital and three p-orbitals D. unhybridized s character

MiVi=MfVf 1.1 M * 0.3 L = 5.5 M * Vf 1.1 M * 0.3 L = Vf 5.5 M Vf = 0.06 L

A chemist wishes to prepare 300 mL of a 1.1 M NaOH solution from a 5.5 M NaOH stock solution. What volume of stock solution should be diluted with pure water to obtain the desired solution?

W = P*ΔV P = 3.6 x 10^5 Pa ΔV = 1.5 m^3 - 1.0 m^3 = 0.5 m^3 W = 3.6 x 10^5 Pa * 0.5 m^3 = 180000 J = 180 kJ Work is being done by the gas: -180 kJ W = -180 kJ ΔU = Q + W ΔU = 300 kJ - 180 kJ = 120 kJ ΔU = 120 kJ

A gas in a cylinder is kept at a constant pressure of 3.6 x 10^5 Pa while 300 kJ of heat is added to it, causing the gas to expand from 1.0 m^3 to 1.5 m^3. Find the work done by the gas and the change in internal energy of the gas.

1. Balance the equations (both stoichiometry and charge) -1 -1 -1 -1 +1 -1 0 IO3- + I- + H+ -----> I3- + H2O -3 -3 -1 -8 +6 -3 0 IO3- + 8I- + 6H+ -----> 3I3- + 3H2O -3 -3 -1 -2 -1 -2 I3- + S2O3(2-) -----> I- + S4O6(2-) -5 -5 -1 -4 -3 -2 I3- + 2S2O3(2-) -----> 3I- + S4O6(2-) 2. Balance the reactions so that the product of the first matches the reactant of the second. IO3- + 8I- + 6H+ -----> 3I3- + 3H2O 3(I3- + 2S2O3(2-) -----> 3I- + S4O6(2-)) 3I3- + 6S2O3(2-) -----> 9I- + 3S4O6(2-) 3. Determine the moles of IO3- M = mol/L M = 0.01 L = 50 mL = 0.05 L mol = 0.05 * 0.01 = 0.0005 mol 4. Convert moles IO3- to moles S2O3(2-) 0.0005 mol (IO3-) * 6 mol (S2O3(2-))/1 mol (IO3-) = 0.003 mol (S2O3(2-)) 5. Find [S2O3(2-)] M = mol/L mol = 0.003 L = 32 mL = 0.032 L M = 0.003/0.032 = 0.094 M [S2O3(2-)] = 0.094 M

A group of students prepares to standardize a Na2S2O3 solution. 32 mL of the Na2S2O3 solution is titrated into 50 mL of a 0.01 M KIO3 solution to reach the equivalence point. They first titrate the KIO3 solution until it loses its color, then add a starch indicator until the reaction is complete. The reaction proceeds in these following steps: IO3- + I- + H+ -------> I3- + H2O I3- + S2O3(2-) -------> I- + S4O6(2-) Determine the concentration of the sodium thiosulfate solution at the beginning of the experiment.

Answer: B. In this question, three substances must be separated using a combination of techniques. The first step should be the most obvious: remove the sand by filtration. The remaining compounds - benzoic acid and naphthalene - are still dissolved in ether. If the solution is extracted with aqueous base, the benzoate anion is formed and becomes dissolved in the aqueous layer, while naphthalene, a nonpolar compound, remains in the ether. Finally, evaporation of the ether will yield purified naphthalene.

A mixture of sand, benzoic acid, and naphthalene in ether is best separated by: A. Filtration, followed by acidic extraction, followed by recrystallization B. Filtration, followed by basic extraction, followed by evaporation C. Extraction, followed by distillation, followed by gas chromatography D. Filtration, followed by size-exclusion column chromatography, followed by extraction

σ-bond π-bond

A molecule has free rotation along a __________. Free rotation of a molecule is restricted if the bond has a ____________.

Intramolecular Intermolecular Stronger

A polar covalent bond is an example of _______________ forces since it takes place inside a molecule. A dipole-dipole interaction is an example of ___________________ forces since it takes place between molecules. A polar covalent bond is __________________ in terms of strength compared to a dipole-dipole interaction.

Spontaneous

A reaction that has a decrease in free energy (ΔG negative), or favors the formation of products is ____________________.

Nonspontaneous

A reaction that has an increase in free energy (ΔG positive), or favors the formation of reactants is ________________________.

Answer: B. This description characterizes a combustion reaction because a hydrocarbon acts as a fuel when reacting with oxygen. Carbon dioxide (an oxide) and water are the products of such a reaction.

A reaction that utilizes oxygen and hydrocarbons as reactants and that produces carbon dioxide and water as products is best characterized as: A. Single displacement B. Combustion C. Metathesis D. Decomposition

Answer: C. A higher Ka implies a stronger acid. Weak acids usually have a Ka that is several orders of magnitude below 1. The pKa of a compound is the pH at which there are equal concentrations of acid and conjugate base; the pKa of this compound would be -log(1) = 0. With such a low pKa, this compound must be an acid. Therefore, the pH of any concentration of this compound must be below 7.

A solution is prepared with an unknown concentration of a theoretical compound with a Ka of exactly 1.0. What is the pH of this solution? A. Higher than 7 B. Exactly 7 C. Less than 7 D. There is not enough information to answer the question

RF(Acetic acid) = 2 cm/10 cm = 0.2 RF(hexane) = 7 cm/10cm = 0.7

A solution that contains acetic acid and hexane is run through a thin-layer chromatography. On the plate, we see that the acetic acid component moved about 2 cm up the plate, while hexane moved about 7 cm up the plate. If the solvent reached to about 10 cm up the plate, what would be the retention factors of acetic acid and hexane?

Less than Greater than

A solution would be acidic if it had a pH that is ________ than 7. A solution would be basic if it had a pH that is _______________ than 7

Mole fraction: 90 g NaCl/58.5 g/mol NaCl = 1.5 mol NaCl 10 L H2O = 10000 mL H2O * 1 g/mL = 10000 g H2O 10000 g H2O/18 g/mol H2O = 555.6 mol H2O 555.6 + 1.5 = 557.1 mol solution 1.5 mol NaCl/ 557.1 mol solution = 0.0027 mole fraction = 2.7 x 10^-3 Percent composition by mass: 10000 g H2O + 90 g NaCl = 10090 g solution 90 g NaCl/10090 g solution * 100% = 0.9% percent composition by mass = 0.9%

A stock solution for making typical IV saline bags contains 90.0 g of NaCl per 10 liters of water (density = 1 g/mL) What is the mole fraction and the percent composition by mass of NaCl in the saline solutions?

Concentration

A titration procedure is used to determine the ____________________ of a given solution.

We can use free receptors, targets, or antibodies, which competes with the stationary phase, and ultimately frees the protein. OR It can be washed out by an elutent that has a pH or salinity level that disrupts the bonds between the ligand and the protein.

After all other compounds have passed through the column in affinity chromatography, what can be used to elute our desired product?

A salt gradient is used to elute the charged molecules stuck to the column.

After all other compounds have passed through the column in ion-exchange chromatography, what can be used to elute our desired product?

Answer: D. Utilize the method described earlier to balance this redox reaction. The balanced half-reactions are: S8 + 24H2O ---> 8SO3(2-) + 48H+ + 32e- NO3- + 4H+ + 3e- ---> NO + 2H2O To get equal numbers of electrons in each half-reaction, the oxidation half-reaction will have to be multiplied by 3, and the reduction half-reaction will have to be multiplied by 32: 3S8 + 72H2O ---> 24SO3(2-) + 144H+ + 96e- 32NO3- + 128H+ + 96e- ---> 32NO + 64H2O This makes the overall reaction: 3S* + 32NO3- + 8H2O ---> 24SO3(2-) + 32NO + 16H+ The sum of the coefficients is therefore: 3 + 32 + 8 + 24 + 32 + 16 = 115.

After balancing the following oxidation-reduction reaction, what is the sum of the stoichiometric coefficients of all the reactants and the products? S8 (s) + NO3- (aq) ---> SO3(2-) (aq) + NO (g) A. 4 B. 50 C. 91 D. 115

Diols Glycols -diol preserved -diol

Alcohols with two hydroxyl groups are referred to as ___________ or ________________, and are indicated with the suffix _____________. The hydrocarbon name is ____________________ and __________ is added.

Answer: A. The greater the intensity, the greater the number of incident photons and, therefore, the greater the number of electrons that will be ejected from the metal surface (provided that the frequency of the light remains above the threshold). This means a larger current. Remember that the frequency of the light (assuming it is above the threshold frequency) will determine the kinetic energy of the ejected electrons; the intensity of the light determines the number of electrons ejected per time (the current).

All of the following statements about the photoelectric effect are true EXCEPT: A. the intensity of the light beam does not affect the photocurrent B. the kinetic energies of the emitted electrons do not depend on the light intensity C. a weak beam of light of frequency greater than the threshold frequency yields more current than an intense beam of light of frequency lower than the threshold frequency D. for light of a given frequency, the kinetic energy of emitted electrons increases as the value of the work function decreases

Indicator

An ________________ is a weak acid or base in a titrand solution that changes color when the equivalence point is reached.

Answer: A. The equation ΔTb = iKbm can be used to solve this problem. The change in boiling point is 374 - 373 = 1 K. Then plug that into: ΔTb/iKb = m 1 K/(1 * 0.512 K*kg/mol) = 2 m The van't Hoff factor for this solute is 1 because the molecule does not dissociate into smaller components. Then convert to grams of solute using the definition of molality. mol solute/kg solvent = molality mol solute = 2 m * 0.1 kg = 0.2 mol The mass used in this equation is 0.1 kg because 100 mL of water has a mass of 0.1 kg. Then determine the molar mass: molar mass = 70 g/0.2 mol = 350 g/mol, which is closest to choice A.

An aqueous solution was prepared by mixing 70 g of an unknown nondissociating solute into 100 g of water. The solution has a boiling point of 101.0°C. What is the molar mass of the solute? (Note: Kb = 0.512 K*kg/mol). A. 358.4 g/mol B. 32.3 g/mol C. 123.2 g/mol D. 233.6 g/mol

Answer: C. First, balance the chemical equation: 4Au + 8NaCN + O2 + 2H2O ---> 4Na[Au(CN)2] + 4 NaOH Now determine the number of moles of NaCN used in the reaction: 0.1 L * 2 M = 0.2 mol NaCN If 0.2 mol NaCN are used in the reaction, then: 0.2 mol NaCN * (4 mol Au/8 mol NaCN) = 0.1 mol Au is oxidized.

An assay is performed to determine the gold content in a supply of crushed ore. One method for pulling gold out of ore is to react it in a concentrated cyanide (CN-) solution. The equation is provided below: Au + NaCN + O2 + H2O ---> Na[Au(CN)2] + NaOH An indicator is used during this reaction, and approximately 100 mL of a 2 M NaCN solution is used to reach the endpoint. How many moles of Au are present in the crushed ore? A. 0.01 mol B. 0.02 mol C. 0.10 mol D. 0.20 mol

4 3 2

An atom that has a hybridization of sp3 has a steric number of ________. An atom that has a hybridization of sp2 has a steric number of ________. An atom that has a hybridization of sp has a steric number of __________.

Greater Less

An atom with a ______________ nuclear charge increases ionization energy, while an atom with ________ nuclear charge decreases ionization energy.

Answer: D. An electron in the n=4 shell and the l=2 subshell can have five different values for m(l): -2, -1, 0, +1, and +2. In each of these orbitals, electrons can have positive or negative spin. Thus, there are 5 x 2 = 10 possible combinations of quantum numbers for this electron.

An electron is known to be in the n=4 shell and the l=2 subshell. How many possible combinations of quantum numbers could this electron have? A. 1 B. 2 C. 5 D. 10

Positive

An endothermic reaction absorbs heat from the surroundings as a reaction proceeds, making the enthalpy (ΔH) of the reaction _______________.

Reactants Products

An endothermic reaction favors the _______________ in a reaction. An exothermic reaction favors the ________________ in a reaction.

Negative

An exothermic reaction gives off heat into the surroundings as a reaction proceeds, making the enthalpy (ΔH) of the reaction ___________________

Weaker

An indicator need to be _______________ than the titrand in a titration, otherwise the indicator will be titrated first.

Acid Anhydride One Carboxylic acids Acid

Anhydrides are named by replacing _________ with _______________ in the name of the corresponding carboxylic acid if the anhydride is formed from only ________ type(s) of carboxylic acid. If anhydride is not symmetrical, both ________________________ are named (without the _______ suffix), before the anhydride is added to the name.

-Liquid coating or solid support material -Inert gas -Boiling point of molecules

Answer the following in regards to gas chromatography: -What is the stationary phase? -What is the mobile phase? -What is the basis of separation of components?

Answer: B. Antimony (Sb) is on the right side of the periodic table, but not far right enough to be a nonmetal, (D.). It certainly does not lie far right enough to fall in Group VIIA (Group 17), which would classify it as a halogen, (C.). While sources have rarely classified antimony as a metal, (A.), it is usually classified as a metalloid, (B.).

Antimony is used in some antiparasitic medications - specifically those targeting Leishmania donovani. What type of element is antimony? A. Metal B. Metalloid C. Halogen D. Nonmetal

They are not optically active. They do not have a chiral center, meaning that they have a plane of symmetry and are unable to be optically active.

Are achiral molecules optically active? Why or why not?

No, not all R-enantiomers are dextrorotatory, because there is no correlation between R,S stereochemistry and optical activity.

Are all R-enantiomers dextrorotatory? Explain why or why not.

They are optically active. Chiral molecules contains a chiral center, and does not contain a plane of symmetry, meaning that it is optically active.

Are chiral molecules optically active? Why or why not?

Answer: A. The formula for methanol is H3COH, for methanal is HCHO, and for methanoic acid is HCOOH, If we assign oxidation numbers to carbon in each molecule, it starts at -2, then becomes 0, then becomes +2: +1 -2-2+1 +1 0+1-2 +1+2-2-2+1 H3COH -----> HCHO -----> HCOOH +3 In general, it is often easier to think of oxidation as a gain of bonds to oxygen (or a similarly electronegative element) or loss of bonds to hydrogen for organic compounds. Therefore, because the carbon is oxidized as one converts from an alcohol to an aldehyde to a carboxylic acid, the oxidation number must increase.

As methanol is converted to methanal, and then methanoic acid, the oxidation number of the carbon: A. increases B. decreases C. increases, then decreases D. decreases, then increases

Increases Increases

As n increases, the average distance of the electron from the nucleus _________________, and the energy _________________________ as well.

increases

At constant temperature entropy always ___________ upon dissolution.

Fe2O3 + 3H2SO4 ---> Fe2(SO4)3 + 3H2O

Balance the following reaction: Fe2O3 + H2SO4 ---> Fe2(SO4)3 + H2O

3Pb(NO3)2 + 2AlCl3 ---> 3PbCl2 + 2Al(NO3)3

Balance the following reaction: Pb(NO3)2 + AlCl3 ---> PbCl2 + Al(NO3)3

Zn + 2HCl ---> ZnCl2 + H2O

Balance the following reaction: Zn + HCl ---> ZnCl2 + H2

Bromthymol Blue

Based on the curve shown, what would be the best indicator to use for this titration?

Methyl Red

Based on the curve shown, what would be the best indicator to use for this titration?

phenolphthalein

Based on the curve shown, what would be the best indicator to use for this titration?

1. Any elements that are considered metals 2. Any elements that are considered nonmetals 3. Any elements that are considered metalloids

Based on their location in the periodic table, identify a few elements that are likely to possess the following properties: 1. Luster 2. Poor conductivity and electricity 3. Good conductivity but brittle

Answer: C. NH3 has three hydrogen atoms bonded to the central nitrogen, which also has a lone pair. These four groups - three atoms, one lone pair - lead NH3 to assume tetrahedral electronic geometry yet trigonal pyramidal molecular geometry. The nitrogen in ammonia is sp3-hybridized. By hybridizing all three p-orbitals and the one s-orbital, four groups are arranged about the central atom, maximizing the distances between the groups to minimize the energy of the molecule with a tetrahedral configuration. In contrast, BF3 has three atoms and no lone pairs, resulting in sp2-hybridization. Its shape is called trigonal planar.

Both BF3 and NH3 have three atoms bonded to the central atom. Which of the following is the best explanation for why the geometry of these two molecules is different? A. BF3 has three bonded atoms and no lone pairs, which makes its geometry trigonal pyramidal B. NH3 is nonpolar, while BF3 is polar C. NH3 has three bonded atoms and one lone pair, which makes its geometry trigonal pyramidal D. BF3 is nonpolar, while NH3 is polar

weaker higher

Bromine has more electrons than chlorine, therefore, chlorine would have ___________________________ London dispersion forces compared to bromine, meaning that bromine has a _______________ boiling point than chlorine.

[CH3COOH] = 1.00 M CH3COOH + H2O <------> CH3COO- + H3O+ Initial 1.00 M 0 0 Change -x +x +x Equilibrium 1.00 - x x x Find Ka: Ka = products/reactants = [x][x]/[1.00-x] assume x << 1, makes x negligible in 1.00 - x 1.8 x 10^-5 = x^2/1.00 1.8 x 10^-5 = x^2 sqrt(1.8 x 10^-5) = x x = 0.0042 x = [H3O+] = 0.0042 pH = -log[H3O+] = -log[0.0042] = 2.38 pH = 2.38

Calculate the pH of 1.00 M solution of vinegar (acetic acid in water). Ka = 1.8 x 10^-5

terminal one oxidized

Carboxylic acids are ________________ functional groups, and their carbon is usually carbon _____. Carboxylic acids are the most ______________ functional group with 3 bonds to oxygen.

dramatic small

Changes in molecular structure can cause _________________ shifts in absorption patterns of a substance, even _______________ changes have this effect.

conservation of mass and charge.

Chemical reactions must be balanced to reflect the laws of ________________________________________________.

Ag - M Al - M K - M P - NM Pb - M Li - M Pu - M U - M Cu - M As - MO Zn - M B - MO Si - MO Np - M O - NM He - NM

Classify the following elements as metals (M), nonmetals (NM), or metalloids (MO): -Ag -Al -K -P -Pb -Li -Pu -U -Cu -As -Zn -B -Si -Np -O -He

alkyl alphabetical order ketone

Common names for ketones are named by listing the ______ groups attached to the carbonyl carbon in ______________ ____________, followed by ____________.

F Br H H2N H F

Complete the Newman projection for the following compound:

P = XP° 540 g H2O/18 g/mol H2O = 30 mol H2O 190 g MgCl2/95 g/mol MgCl2 = 2 mol MgCl2 X(H2O) = 30 mol H2O/32 mol solution = 0.94 P = 0.94 * 3.2 kPa = 3 kPa

Determine the vapor pressure of a solution containing 190 g MgCl2 in 540 g water at room temperature. (Note: The vapor pressure of pure water at 25°C is 3.2 kPa).

Distillation takes advantage of differences in boiling points in order to separate solutions of miscible liquids.

Distillation separates compounds based on what property?

Enantiomers have the same physical properties. Enantiomers do not have the same chemical properties.

Do enantiomers have the same physical properties? What about chemical properties?

Water has greater potential energy than ice due to the alterations in the intramolecular and intermolecular forces of the particles, leading to more microstates. Water and ice have the same average kinetic energy since both are sitting at 0°C.

Does water have greater potential energy than ice at freezing temperature? What about kinetic energy?

Answer: C. Warm or hot solvent is generally used in gravity filtrations to keep the desired product soluble. This allows the product to remain in the filtrate, which can then be collected. In this case, the student likely used solvent that was way too cold, and the product crystallized out. The product should be present in the residue.

During gravity filtration, a student forgets to heat the solution before running it through the filter. After capturing the filtrate, the student analyzes the sample via infrared (IR) spectroscopy, and finds none of the desired product in the filtrate. What likely occurred to the student's project? A. The product degraded because of a prolonged filtration time. B. The product evaporated with collection of the filtrate C. The product precipitated and is present in the residue D. The product was dissolved in the solvent

When electrons transition from a higher-energy state to a lower-energy state, they will experience photon emission.

During which electronic transitions is photon emissions most common?

Left Right Down

Effective nuclear charge (Z(eff)) increases from _______ to _____________ on the periodic table. Principle quantum number increases by 1 the further __________ you go on the periodic table.

Increases Stronger

Electronegative elements that are positioned closer to an acidic proton _____________ the acid strength by pulling electron density out of the bond holding the acidic proton. Furthermore, molecules with electronegative elements are ____________ in acid strength than molecules without electronegative elements.

Representative elements Nonrepresentative elements

Elements in Groups 1-2 and 13-18 are known as ______________________ elements. Elements in Groups 3-12 and the lanthanide and actinide elements are known as ___________________ elements.

Strong bases

Examples of __________ ___________ includes the following: -NaOH (sodium hydroxide) -KOH (potassium hydroxide) -Ca(OH)2 (calcium hydroxide) -Mg(OH2) (magnesium hydroxide)

Strong acids

Examples of __________ ____________ includes the following: -HClO4 (perchloric acid) -HCl (hydrochloric acid) -HBr (hydrobromic acid) -HI (hydroiodic acid) -H2SO4 (sufuric acid) -HNO3 (nitric acid)

Weak acids

Examples of ___________ ____________ includes the following: -HF (hydrofluoric acid) -CH3COOH (acetic acid) -CH3OH (methanol)

Weak bases

Examples of ____________ ____________ includes the following: -NH3 (ammonia) -C6H5NH2 (aniline)

Polyvalent bases

Examples of _____________ ________________ includes the following: -Al(OH)3 (aluminum hydroxide) -Ca(OH)2 (calcium hydroxide) -Mg(OH)2 (magnesium hydroxide)

Polyvalent acids

Examples of _______________ ______________ includes the following: -H2SO4 (sulfuric acid) -H3PO4 (phosphoric acid) -H2CO3 (carbonic acid)

Assume 100 g compound Fe: 28.5 g * 1 mol = 0.51 mol ~ 0.5 mol 55.8 g S: 24 g * 1 mol = 0.75 mol 32 g O: 49.7g * 1 mol = 3.1 mol ~ 3 mol 16 g O: 3 x 2 = 6 x 2 = 12 S: 0.75 x 2 = 1.5 x 2 = 3 Fe: 0.5 x 2 = 1 x 2 = 2 Fe2S3O4

Experimental data from the combustion of an unknown compound indicates that it is 28.5% iron, 24% sulfur, 49.7% oxygen by mass. What is the empirical formula?

A form of chromatography in which a small sample is put into a column that can be manipulated with sophisticated solvent gradients to allow very refined separation and characterization.

Explain high performance liquid chromatography (HPLC)

Step 1: Find the molar mass of ammonium phosphate trihydride ((NH4)3PO4+3H2O) Molar mass = 18(1) + 3(14) + 7(16) + 31 = 203 g/mol Step 2: Find out the number of moles of (NH4)3PO4+3H2O that we have in our 25 mL solution 4 g (NH4)3PO4+3H2O = 1 mol (NH4)3PO4+3H2O = 0.02 mol 203 g (NH4)3PO4+3H2O Step 3: Find out how many moles are in 6 mL out of 25 mL 0.02 mol/25 mL = x/6 mL x = (0.02 mol * 6 mL)/25 mL = 0.0048 mol Step 4: Multiply Avogadro's number to find number of ions 0.0048 mol * 6.022 x 10^23 = 2.89 x 10^21 molecules Step 5: Find the amount of ammonium ions dissociated into the solution 3 NH4 ions in (NH4)3PO4+3H2O. 2.89 x 10^21 molecules * 3 = 8.67 x 10^21 ions NH4+

Find the number of ammonium ions in a 6 mL aqueous solution of ammonium phosphate trihydrate ((NH4)3PO4+3H2O) prepared by dissolving 4 g of ammonium phosphate trihydrate in enough water to make 25 mL of the solution.

Photon emission two more

Fluorescence is a special kind of _______________________ in which an excited electron returns to the ground state through ________ or _________ intermediate excited states.

partially positive partially negative

For a dipole-dipole interaction to exist, it requires it to be between a ___________________ atom and a ________________________ atom, such as the case of polar covalent bonds like HCl.

Zero Attractive force

For an electron where its energy level is n=infinity, the electron has an energy value of _________, meaning that there is no _______________________ acting on the electron.

Highest Lowest Highest Double

For numbering of carbon rings, numbering starts at the carbon with the ________________ priority substituent. Numbering continues in the manner of giving the _____________ possible numbers to the __________________ priority substituted carbon. If there is a tie between assigning priority in a molecule with double and triple bonds, the ______________ bond takes precedence.

1. Ionic compounds 2. Covalent compounds with hydrogen bonding 3. Polar covalent compounds 4. Nonpolar covalent compounds

For the following compounds, order them in order from strongest total intermolecular forces to weakest total intermolecular forces: -Nonpolar covalent compounds -Ionic compounds -Polar covalent compounds -Covalent compounds with hydrogen bonding

Not bonded Bonded shell Free Bond

Formal charge of an atom is equal to the number of valence electrons found in an atom when it is _______ ___________ subtracted by the number of valence electrons in the ____________ ___________. FC = (#VE(______)) - (#VE(_________))

Least stable ---> most stable 1. A. 2. C. 3. B. 4. D.

From the Newman Projections shown, order them from least stable to most stable/highest in energy to lowest in energy.

IUPAC: 1-hexanol Common: hexyl alcohol

Give both the common and IUPAC name for the following molecule.

IUPAC: 2-butanone Common: ethylmethylketone

Give both the common and IUPAC name for the following molecule.

IUPAC: 2-propanone Common: dimethylketone and acetone

Give both the common and IUPAC name for the following molecule.

Step 1: Balance the reaction Reaction is already balanced Step 2: Find grams of iodine 4.63 g FeI3 * 1 mol FeI3 = 0.0106 mol FeI3 436.545 g FeI3 0.0106 mol FeI3 * 3 mol I2 = 0.0159 mol I2 2 mol FeI3 0.0159 mol I2 * 253.8 g I2 = 4.035 g I2 1 mol I2

How many grams of iodine must react to give 4.63 g of ferric oxide? 2Fe + 3I2 ---> 2FeI3

It makes the bands of the components to elute parallel to one another, to avoid the two bands from mixing with one another. The bands would mix with each other, preventing us from isolating the components.

In column chromatography, what is the importance of making sure that the stationary phase is leveled? What would happen to the components of the sample if the stationary phase was not leveled?

Gravity

In column chromatography, what is the main mechanism of action that pushes the mobile phase through the stationary phase?

-To prevent our stationary phase (like silica gel) from spilling from the flask when the stopcock opens. -To make sure that the layer of the stationary phase is leveled in order to have an even extraction of the components of the sample.

In column chromatography, what is the significance of the layer of cotton at the bottom of the column? What is the significance of the layer of sand above and below the stationary phase?

F- K

In each of the following pairs, which has the larger radius? -F or F- -K or K+

Polar: Stationary phase Nonpolar: Mobile phase

In normal-phase paper chromatography, TLC, and simple column chromatography, which phase is typically polar and which phase is typically nonpolar?

Voltage Endpoint

Indicators in a redox titration changes color when a particular __________ value has been reached, which indicates when the ___________ has been reached.

Weaker

Intermolecular forces are ______________ in terms of strength when compared to intramolecular forces.

kJ/mol

Ionization energy is measured in ____________.

Geminal diol

Is the following compound a geminal diol or a vicinal diol?

Small or no Large Small Large Higher Lower

It is preferable for a Lewis structure to have ____________ or __________ formal charges than to have ___________ formal charges. It is preferable for a Lewis structure to have __________ separations between atoms of opposite charges than to have ______________ separations between atoms of opposite charges. It is preferable for a Lewis structure to have negative formal charges on atoms of ______________ electronegativity than atoms of ______________ electronegativity.

Answer: B. Because the lactoferrin proteins are likely to be charged, as is the resin described in the question, this is an example of ion-exchange chromatography. The charged protein molecules will stick to the column, while the remainder of the milk washes through and can later be washed off of the column and collected.

Lactoferrin, a milk protein, is a valuable antimicrobial agent that is extracted from pasteurized, defatted milk utilizing a column containing a charged resin. This is an example of which of the following chromatographic techniques? A. Thin-layer chromatography B. Ion-exchange chromatography C. Affinity chromatography D. Size-exclusion chromatography

Atomic number

Mendeleev's table was arranged by atomic weight, but the modern periodic table is arranged by ____________ ___________.

Mobile electrons Fixed positive metal ions

Metallic bonds are formed through the attraction of _______________________ (referred to as the sea of electrons) and the _______________________________.

Answer: B. Like atomic orbitals, molecular orbitals each can contain a maximum of two electrons with opposite spins. The 2n^2 rule in (D.) refers to the total number of electrons that can exist in a given energy shell, not in a molecular orbital.

Molecular orbitals can contain a maximum of: A. One electron B. Two electrons C. Four electrons D. 2n^2 electrons, where n is the principal quantum number of the combining atomic orbitals.

-Acetic acid -Acetate

Name the acid and it's anion: -CH3COOH -CH3COO-

-Boric acid -Borate

Name the acid and it's anion: -H3BO3 -BO3(3-)

Sulfate

Name the common polyatomic ion: SO4(2-)

1. methane 2. ethane 3. propane 4. butane 5. pentane 6. hexane 7. heptane 8. octane 9. nonane 10. decane 11. undecane 12. dodecane

Name the following alkanes.

Cyclopropanol, acetone, and prop-2-ene-1-ol are structural isomers of each other.

Of the following compounds: -cyclopropanol -2-propanol -acetone -prop-2-ene-1-ol ...which are structural isomers of each other?

The one on the left is stronger since the electronegative atom (Cl) is closer to the red acidic proton in the left molecule than the right molecule.

Of the two acidic molecules, which one is stronger? Explain your reasoning.

actual yield theoretical yield actual yield theoretical yield

Percent yield is calculated by dividing the ___________________ by the ________________________ and multiplied by 100%. %yield = ______________/_____________ * 100%

Trigonal Planar

Predict the molecular geometry of AlF3.

Linear

Predict the molecular geometry of MgF2.

Tetrahedral

Predict the molecular geometry of SiH4.

-Carboxylic acid -Anhydride -Ester -Amide -Aldehyde -Ketone -Alcohol -Alkene -Alkyne -Alkane

Put the following functional groups in order of decreasing priority: -Alcohol -Aldehyde -Alkane -Alkene -Alkyne -Amide -Anhydride -Carboxylic acid -Ester -Ketone

1. O 2. Sb 3. Tl 4. Ne

Rank the following elements by decreasing electronegativity: -Antinomy (Sb) -Neon (Ne) -Oxygen (O) -Thallium (Tl)

Bonds R,S-configuration

Relative configuration will be retained in a molecule if the _________________ of the stereocenter are kept the same. Absolute configuration is retained if the __________________________ of the stereocenter is kept the same.

Conjugated bonds Conjugation Unhybridized p-orbitals π electrons p-orbital

Resonance delocalization of electrons occurs in molecules that have ______________________. ____________________ requires alternating single and multiple bonds because this pattern aligns a number of __________________________________ down the backbone of the molecule. ____________________ can delocalize through this _________________ system, adding stability to molecules.

Equilibrium Hybrid

Resonance structures are not in any sort of __________________, therefore the electron density is distributed throughout the entire molecule. Therefore, the true form of the molecule will be a __________ of the resonance structures.

Answer: B. Silica gels are polar. Polarity is used to selectively attract specific solutes within a nonpolar solvent phase. Although silica gels have other properties, this is the most important to TLC.

Silica gel is often used in thin-layer chromatography. What property does silica gel probably possess that makes it useful for this purpose? A. Acidity B. Polarity C. Specifically sized pores D. Aqueous solubility

Q = mcΔT m = 1 kg c = 233 J/kg*K ΔT = 962°C - 20°C = 924 K Q = 1 kg * 233 J/kg*K * 924 K Q = 219486 J Q(fus) = mL m = 1 kg L = 1.05 x 10^5 J/kg Q(fus) = 1 kg * 1.05 x 10^5 J/kg Q(fus) = 105000 J Q(total) = Q + Q(fus) Q(total) = 219486 J + 105000 J = 324486 J Q(total) = 324486 J

Silver has a melting point of 962°C and a heat of fusion of approximately 1.05 x 10^5 J/kg. The specific heat of silver is 233 J/kg*K. Approximately how much heat is required to completely melt a 1 kg silver chain with an initial temperature of 20°C.

Answer: C. This is the only option that would be effectively separated by a simple distillation. (B) would require vacuum distillation because the boiling points are over 150°C. In (A) and (D), the boiling points are within 25°C of each other and would therefore require fractional distillation in order to be separated.

Simple distillation could be used to separate which of the following compounds? A. Toulene (bp of 111°C) and water (bp of 100°C) B. Naphthalene (bp of 218°C) and butyric acid (bp of 119°C) C. Propionaldehyde (bp of 50°C) and acetic acid (bp of 119°C) D. Benzene (bp of 80°C) and isopropyl alcohol (bp of 83°C)

Assume that we have 100 grams of the compound: C = 44.7% = 44.7 g H = 7.5% = 7.5 g O = 47.8% = 47.8 g Divide mass by atomic mass to get moles: 44.7 g C * 1 mol C ~ 48 g C = 3.75 mol C 12 g C 12 g C 7.5 g H * 1 mol H = 7.5 mol H 1 g H 47.8 g O * 1 mol O ~ 48 g O = 3 mol O 16 g O 16 g O Multiply by 4 to get rid of decimals: 3.75 mol C * 4 = 15 mol C 7.5 mol H * 4 = 30 mol H 3 mol O * 4 = 12 mol O C15H30O12 Simplify: C15H30O12/3 = C5H10O4

Suppose we have a compound with mass percentages of: 44.7% C 7.5% H 47.8% O Find the empirical formula.

Fractional charge

The _______________ _______________ of a molecule is the total charge on the terminal atoms divided by the number of terminal atoms.

Bicarbonate buffer system

The ________________ is a system in our blood that helps us maintain a physiological level of pH using carbonic acid and bicarbonate ions

to work full-time/part-time

The ________________ is the energy (or work) required to remove an electron completely from a metal surface.

van't Hoff factor

The ______________________ is the number of particles into which a compound dissociates in a solution.

Atomic radius

The average distance between a nucleus and its outermost electron is known as the ___________ ___________.

Ionic radius

The average distance from the center of the nucleus to the edge of an electron cloud for a cation and anion is known as the ______________ ____________.

higher lowest

The carbon with a hydroxyl group (-OH) substituent has ____________ priority compared to double or triple bonds in a molecule, thus will receive the _______________ numbered carbon compared to the multiple bonds.

Weak Strong

The equivalence point of a titration reaction between a _________ acid and ______________ base would have a pH>7.

Answer: C. In this situation, heat will transfer from the warm gas to the metal and then to the cold gas. Convection requires flow of a fluid to cause heat transfer, invalidating (B) as an answer. In this case, the gas is not flowing, but rather is in contact with the metal. (A) is an invalid answer because heat transfer through radiation, is also implausible not only because gases are unlikely to emit heat in the form of waves, but also because the radiation would be unlikely to penetrate the thick metal containers, Enthalpy, (D), is not a form of heat transfer. Conduction, (C), is the most likely option; it happens when two substances make direct contact with one another. Here, gas A makes contact with the metal container, which makes contact with gas B.

The figure shown depicts a thick metal container with two compartments. Compartment A is full of a hot gas, while compartment B is full of a cold gas. What is the primary mode of heat transfer in this system? A. Radiation B. Convection C. Conduction D. Enthalpy

The mass of an electron

The following is the value of _______________________: 9.11 x 10^-31 kg

Polar covalent

The intermolecular forces present in a ____________________ compound includes London dispersion forces and dipole-dipole interactions between partially charged ions.

Larger Smaller Smaller Larger

The ionic radii of anions are _____________ than their neutral atom The ionic radii of cations are _____________ than their neutral atom Metals closer to the metalloid line have ____________ ionic radii than metals further away. Nonmetals closer to the metalloid line have ___________ ionic radii than nonmetals further away.

molar solubility = molarity molarity = 4 x 10^-10 M Fe(OH)3 (s) <-----> Fe(3+) (aq) + 3OH- (aq) I 0 0 C -x +x +3x E x 3x x = 4 x 10^-10 M Ksp = [Fe(3+)]^1 * [OH-]^3 Ksp = x * (3x)^3 Ksp = (4 x 10^-10 M) * (3 * (4 x 10^-10 M))^3 Ksp = (4 x 10^-10 M) * (1.7 x 10^-27 M) Ksp = 6.8 x 10^-37

The molar solubility of Fe(OH)3 in an aqueous solution was determined to be 4 x 10^-10 mol/L. What is the value of the Ksp for Fe(OH)3 at the same temperature and pressure?

Water

The most common example of an amphiprotic amphoteric species is __________

Double-displacement reaction

The only type of reaction that is not considered a redox reaction is _______________________.

ΔG = Gibbs Free Energy Change ΔH = Enthalpy Change ΔS = Entropy Change T = Temperature (Kelvin)

The spontaneity of dissolution is dependent on the change in Gibbs Free Energy, which is determined by: ΔG = ΔH - TΔS ΔG = ____________________ ΔH = _________________ ΔS = _______________ T = _____________

Conjugate base Conjugate acid

The stronger the acid, the weaker the _______________ will be. The stronger the base, the weaker the ______________ will be.

Groups Periods

The vertical columns in the periodic table are called __________ The horizontal rows in the periodic table are called __________

2 Carboxyl group Amino group

There are _________ equivalence points in the titration of a nonpolar/neutral amino acid. One corresponding to the titration of the _________________. One corresponding to the titration of the __________________.

Cyclobutane Puckered conformation

To reduce strain on itself, ___________________ takes on a ______________________ conformation, as shown here.

True

True or false, the line spectrum is unique for every element on the periodic table.

It is better to use gravity filtration if the desired product is in the filtrate. It is better to use vacuum filtration if the desired product is the solid residue.

Under what conditions would it be more preferred to use gravity filtration over vacuum filtration. What about vacuum filtration over gravity filtration.

higher weaker

Water has a _____________ boiling point compared to acetone because water has hydrogen bonding interactions while acetone has dipole-dipole interactions. Dipole-dipole interactions are ______________ in terms of strength compared to hydrogen bonding.

Visible lights Ultraviolet lights Infrared lights

Wavelengths from 380 nm to 740 nm are in the range of ___________________. Wavelengths below 380 nm fall into the range of ____________________, which we cannot see. Wavelengths above 740 nm fall into the range of ________________, which we cannot see.

-Nuclear Charge -Electron Shielding -Distance between the nucleus and valence electron

What are the factors that affect the difference between first and second ionization energy?

Metals

What are the gray colored elements in the periodic table classified as?

The two liquids have a boiling point that is under 150°C. The difference in boiling points between the two liquids is less than 25°C.

What are the requirements needed for fractional distillation to be used?

.. .. .. O === S --- O : .. .. .. .. .. : O --- S === O .. ..

What are the resonance structures of SO2?

It means that the enantiomer is dextrorotatory.

What does it mean if an enantiomer is labeled with a (+) on it?

It is the degree to which a sample contains one enatiomer in excess over the other. It is calculated by subtracting the percentage of the enantiomer of greater concentration with the percentage of enantiomers of lesser concentration. %greater enantiomer - %lesser enantiomer = ee

What is enantiomeric excess (ee)? How is it calculated?

Conformational isomers can interconvert into different stereoisomers by rotation around a single bond. Configurational isomers can only interconvert by breaking and reforming covalent bonds.

What is the difference between conformational isomers and configurational isomers?

Normal-phase implies that the stationary phase is more polar than the mobile phase. Reverse-phase implies that the mobile phase is more polar than the stationary phase.

What is the difference between normal-phase chromatography and reverse-phase chromatography?

H \ .. C === O / .. H

What is the dot structure of HCHO?

H I I H ---- N(+) ---- H I I H

What is the dot structure of NH4+?

ethyl

What is the name of this substituent?

isopropyl

What is the name of this substituent?

It is better to do three washes with 10 mL than to do one with 30 mL; more of the compound of interest would be extracted with multiple sequential extractions than one large one.

When doing an extraction, would it be better to do three extractions with 10 mL of solvent, or one extraction with 30 mL?

Down Left

When going ___________ in the periodic table, you are adding electrons to a higher energy level, which is further away from the nucleus, causing an increase in the atomic radii. When going __________ in the periodic table, the nuclear charge decreases, leading to less attraction of valence electrons to the nucleus, resulting in the valence shell to expand, resulting in an increase in atomic radii.

On carbon 3 and carbon 5

Where are the hydroxyl group(s) located on a hexane-3,5-diol molecule?

-2 0 +1 .. : N --- C === O : .. -1 0 0 .. .. N === C === O .. .. 0 0 -1 .. : N === C --- O : ..

Write the resonance structures for [NCO]-.

ΔU = Q + W Work is done on the gas: W = 40 J ΔU = -150 J -150 J = Q + 40 J Q = -190 J

You have a certain gas in a cylindrical container with a piston at the top of it. While 40 J of work is done on the gas, the internal energy goes down by 150 J. What was the value of the heat added to the gas?

Ketones Aldehydes Number assigned Propane

____________ contains a carbonyl group somewhere in the middle of the chain. Unlike _______________, the carbonyl carbon will need to have a ______________ ______________ to it, since it could be anywhere in the middle of the parent chain. The only exception to this rule is ________________, which will have a carbonyl carbon on carbon 2 by default.

Nuclear charge

_____________ ____________ itself does not affect the difference in ionization energies, but it still plays a role in determining it through the calculation of effective nuclear charge.

Electron affinity

_______________ _______________ is the energy dissipated by a gaseous species when it gains an electron. X + e- -----> X- + energy

Absorption Emission

_____________________ is the process of an electron absorbing energy and jumping to a higher energy level. ____________________ is the process of an electron emitting light, which causes the electron to fall to a lower energy level.

Empirical formula

_____________________ is the ratio between elements in a compound in a simpler form than the molecular formula.

Combination reaction

________________________ is a reaction in which two or more reactants form one product.

Osmotic pressure Molarity

______________________________ is the pressure needed to be applied to a solution to prevent the transfer of water through a semipermeable membrane from an area of less solute concentration to an area of greater solute concentration. It is dependent on the ________________ of a solution.

Answer: B. π bonds are formed by the parallel overlap of unhybridized p-orbitals. The electron density is concentrated above and below the bonding axis. A σ bond, on the other hand, can be formed by the head-to-head overlap of two s-orbitals or hybridized orbitals. In a σ bond, the density of the electrons is concentrated between the two nuclei of the bonding atoms.

π bonds are formed by which of the following orbitals? A. Two s-orbitals B. Two p-orbitals C. One s- and one p-orbital D. Two sp2-hybridized orbitals

A. Totally eclipsed (0°) B. Gauche staggered (60°) C. Eclipsed (120°) D. Anti staggered (180°)

Determine the conformations of the following Newman Projections of butane. Also include the dihedral/torsional angle of each one of them.

There is no net dipole moment, since each individual dipole moments cancel each other out.

Determine the net dipole moment of the following molecule.

^ I I I I I +

Determine the net dipole moment of the following molecule.

ΔT(b) = i * K(b) * m i = 3 since MgCl2 dissociates into 3 particles m = moles solute/kg solvent 190 g MgCl2/95 g/mol MgCl2 = 2 mol 1500 g water = 1.5 kg water 2 mol/1.5 kg = 1.333 mol/kg ΔT(b) = 3 * 0.512 K*kg/mol * 1.333 mol/kg = 2 K 373 K is the boiling point of water 373 K + 2 K = 375 K

Determine the new boiling point of a solution containing 190 g MgCl2 in 1500 g water at room temperature (K(b) = 0.512 K*kg/mol).

IUPAC: Propanoic acid Common: Propionic acid

Give both the common and IUPAC name for the following molecule.

Heat of fusion Heat of vaporization

What is the heat of transformation at a substance's melting point? What is the heat of transformation at a substance's boiling point?

Ka of CH3OH = 2.9 x 10^-16 pKa = -log[Ka] = -log[2.9 x 10^-16] = 15.54 pKa = 15.54

What is the pKa of CH2OH? Ka = 2.9 x 10^-16

Kb of NH3 = 1.8 x 10^-5 pKb = -log[Kb] = -log[1.8 x 10^-5] = 4.74

What is the pKb of NH3? Kb = 1.8 x 10^-5

Isobaric expansion

What kind of thermodynamic process is being shown in the PV diagram?

-The fluorine bond and the chlorine bond are in the plane of the paper. -The bromine bond is pointing out of the plane away from us -The hydrogen bond is pointing out of the plane towards us.

Identify the 3D spatial arrangement of the tetrahedral molecule.

Both these molecules have unfilled valence electron shells with relatively few paired electrons; therefore, they are paramagnetic.

Magnetic resonance angiography (MRA) is a technique that can resolve defects like stenotic (narrowed) arteries. A contrast agent gadolinium or manganese injected into the blood stream interacts with the strong magnetic fields of the MRI device to produce such images. Based on their orbital configurations, are these contrast agents paramagnetic or diamagnetic?

Answer: B. To solve this equation correctly, we must first convert eV to joules: 10.2 eV * 1.6 x 10^-19 J = 1.6 x 10^-18 J Next we determine the wavelength: E = hc/λ λ = hc/E = 6.626 x 10^-34 J*s * 3 x 10^8 m/s/1.6 x 10^-18 J λ = 1.24 x 10^-7 m

When a hydrogen atom electron falls to the ground state from the n=2 state, 10.2 eV of energy is emitted. What is the wavelength of this radiation? (Note: 1 eV = 1.60 x 10^-19 J, and h= 6.626 x 10^-34 J*s) A. 5.76 x 10^-9 m B. 1.22 x 10^-7 m C. 3.45 x 10^-7 m D. 2.5 x 10^15 m

At equilibrium

When a solution is saturated, the reaction between the solute and solvent is ________________________.

100% ionization Right-side Products

When a strong acid/base undergoes equilibria, it undergoes _________________, which means that the total concentration of the acid turns into products. Because of this, equilibrium will lean to the __________-side of the reaction, favoring the _______________.

ΔU = Q + W Work is done on the gas: W = +60 J Heat is lost: Q = -150 J ΔU = 60 J - 150 J ΔU = -90 J

You have a certain gas in a cylindrical container with a piston at the top of it. 60 J of work is being done on the gas, and the gas loses 150 J of heat to its surrounding. What is the change in internal energy?

ΔU = Q + W Work done by the gas: W = -70 J Heat is added: Q = 180 J Ui = 200 J Uf - 200 J = 180 J - 70 J Uf - 200 J = 110 J Uf = 310 J

You have a certain gas in a cylindrical container with a piston at the top of it. A gas starts with 200 J of internal energy. While you add 180 J of heat to the gas, the gas does 70 J of work. What is the final internal energy of the gas?

Phenol is polar and weakly acidic, while hexane is nonpolar. Phenol can be separated from hexane by the introduction of a base. Say that NaOH is used, it will deprotonate the phenol, leaving it negatively charged, along with an Na+ cation. Sodium phenolate will go on to form the aqueous layer with water, while hexane remains in the organic layer. Because phenol is denser than hexane, the aqueous layer will settle at the bottom of the funnel, thus be extracted first. Open the stopcock, and you will successfully extract phenol.

You have an aqueous solution of hexane and phenol. You want to extract phenol from the solution. How do you go about this?

Al(OH)3: Al(OH)3 is a base, and has 3 OH- groups in it. Thus has 3 base equivalents. The example also mentions that there is 2 M of Al(OH)3. 3 * 2 = 6 N Al(OH)3 = 6 H2SO4: H2SO4 is an acid, and has 2 H+ groups to make 2 H3O+ ions. Thus has 2 base equivalents. The example also mentions that there is 16 M of H2SO4. 16 * 2 = 32 N H2SO4 = 32

Calculate the normality of the following solutions: 2 M Al(OH)3 16 M H2SO4

azimuthal quantum number (l)

The ___________________ quantum number can take on the value of an integer in the 0 to n-1 range. The _________________ quantum number can take on any value from -l to +l.

A thermodynamic process in which no heat is neither added nor removed from the system.

What is an adiabatic thermodynamic process?

NaBr: MW = 103 g/mol 100 g * 1 mol = 0.97 mol 103 g SrCl2: MW = 158.6 g/mol 100 g * 1 mol = 0.63 mol 158.6 g C6H12O6: MW = 180 g/mol 100 g * 1 mol = 0.56 mol 180 g

Calculate the number of moles in 100 g of each of the following: NaBr SrCl2 C6H12O6

Lewis structure: H I I H ----- N -----H I I H Formal charge is the number of valence electrons when the atom is free minus the number of valence electrons when the atom is bonded. FC(N) = 5 -4 = +1 FC(N) = +1

Calculate the formal charge of the central N atom of [NH4]+.

The entropy of a perfect crystal (atoms are perfectly aligned) of any pure substance approaches zero as the temperature approaches absolute zero.

Describe the Third Law of Thermodynamics.

If object A is in thermal equilibrium with object B, and object B is in thermal equilibrium with object C, then object A is in thermal equilibrium with object C.

Describe the Zeroth Law of Thermodynamics

-London dispersion forces between the molecules -Dipole-dipole interactions between the partially positive hydrogen and the partially negative sulfur.

Identify all the intermolecular forces present in the shown molecule.

Phosphorous's atomic number is 15, thus has a nuclear charge of 15. There are 2 inner electrons on the n=1 valence and 8 inner electrons on the n=2 valence, thus it's electron shielding is 8 + 2 = 10. Z(eff) = 15 - 10 = +5 Z(eff) = +5

What is the effective charge of phosphorous?

[Ne] 3s(2) 3p(8)

What is the electron configuration of a chloride (Cl-) anion?

pH = -log[H3O+] [H3O+] = 1.5 x 10^-4 pH = -log[1.5 x 10^-4] = 3.82 pH < 7 --> acidic pH of OJ is 3.82 and it is an acidic solution.

What is the pH of orange juice if it has a concentration of H3O+ ions at 1.5 x 10^-4? Is the solution classified as an acid, base, or neutral?

An amine

Which functional group consists of a nitrogen atom that is attached to either 3 R-groups, 2 R-groups and a hydrogen, or 1 R-group and 2 hydrogens?

Benzene/Aromatic ring

Which functional group consists of a six-carbon ring with alternating single-double bonds?

A sulfide

Which functional group consists of a sulfur atom attached to two R-groups?

An ether

Which functional group consists of an oxygen that is attached to two R-groups?

An alkene

Which functional group consists of only a carbon-carbon double bond?

An alcohol

Which functional group consists of only an -OH group, or a hydroxyl group, attached to an R-group?

Phthalic anhydride

Name the molecule.

Propyl ethanoate

Name the molecule.

Propyl methanoate

Name the molecule.

Trigonal Bipyramidal

What is the molecular geometry of PCl5?

Seesaw

What is the molecular geometry of SF4?

Octahedral

What is the molecular geometry of SF6?

Shape: Bent/Angular Angles: 104.5°

What is the molecular geometry of this molecule? What are the bond angles of this molecule?

Shape: Bent/Angular Angles: 120°

What is the molecular geometry of this molecule? What are the bond angles of this molecule?

-London dispersion forces between the molecules

Identify all the intermolecular forces present in the shown molecule.

14

In acid-base calculations: pH + pOH = _________

molecular orbital

A _____________________ is formed when two atomic orbitals overlap one another.

The bicarbonate buffer system.

The reaction shown is an example of what?

N-dimethylethanamide

Name the molecule.

Answer: C. Draw out these structures. The two names describe the same molecule, which also happens to be a meso compound because it contains a plane of symmetry. These compounds are not enantiomers because they are superimposable mirror images of one another, not nonsuperimposable mirror images. These compounds are better termed meso-2,3-dihydroxybutanedioic acid.

(2R,3S)-2,3-dihydroxybutanedioic acid and (2S,3R)-2,3-dihydroxybutanedioic acid are: I. Meso compounds II. The same molecule III. Enantiomers A. I only B. III only C. I and II only D. I and III only

Answer: B. These two molecules are stereoisomers of one another, but are not nonsuperimposable mirror images. Therefore, they are not diastereomers. Note that these molecules differ by at least one, but not all, chiral carbons.

(R)-2-chloro-(S)-3-bromobutane and (S)-2-chloro-(S)-3-bromobutane are: A. Enantiomers B. Diastereomers C. Meso compounds D. The same molecule

Angle strain Torsional strain Nonbonded strain

-________________: Increased energy that results when bond angles deviate from their ideal values by being stretched and compressed. -_________________: Increased energy that results when molecules assume eclipsed or gauche staggered conformations. -___________________: Increased energy that results when nonadjacent atoms or groups compete for the same space; also called steric strain.

22.4 L

1 mol of an ideal gas at standard temperature and pressure (STP) has a volume of _____________.

6.022 x 10^23 particles

1 mole is equal to _____________________ particles.

Answer: C. Because they have the same molecular formula but different atomic connectivity, 3-methylpentane and hexane are constitutional isomers.

3-methylpentane and hexane are related in that they are: A. Enantiomers B. Diastereomers C. Constitutional isomers D. Conformational isomers

ΔT(f) = i * K(f) * m i = 4 since AgCl3 dissociates into 4 particles. m = moles solute/kg solvent 400 g AlCl3/133 g/mol = 3 mol 1.5 L water = 1500 mL * 1 g/mL = 1500 g = 1.5 kg water 3 mol AlCl3/1.5 kg water = 2 mol/kg ΔT(f) = 4 * 1.86 K*kg/mol * 2 mol/kg ΔT(f) = 14.9 K Freezing point of water is 273 K 273 K - 14.9 K = 258.1 K 258.1 K - 273 = -14.9°C

400 g AlCl3 is dissolved in 1.5 L of water at room temperature (K(f) = 1.86 K*kg/mol). What is the new freezing point of this solution?

Answer: C. ΔL = αLΔT α = 1.1 x 10^-5 K^-1 L = 20 m ΔT = 110°C - 10°C = 100 K ΔL = 1.1 x 10^-5 K^-1 * 20 m * 100 K ΔL = 0.022 m = 2.2 cm 2.5 cm - 2.2 cm = 0.3 cm from the ground

A 20 m steel rod at 10°C is dangling from the edge of a building and is 2.5 cm from the ground. If the rod is heated to 110°C, will the rod touch the ground? (Note: α = 1.1 x 10^-5 K^-1) A. Yes, because it expands by 3.2 cm. B. Yes, because it expands by 2.6 cm. C. No, because it expands by 2.2 cm. D. No, because it expands by 1.8 cm.

Dipole moment

A ___________ is a vector quantity with a magnitude that indicates the direction of the pull of a pair of electrons; oriented from the partially positive atom to the partially negative element.

Single bond Triple bond Double bond

A ____________ bond has the longest bond length of the three bonds, but has the weakest bond strength, and the lowest bond energy. A ____________ bond has the shortest bond length of the three bonds, but has the strongest bond strength, and the highest bond energy. A _____________ bond is middling in bond length of the three bonds, thus also having middling bond strength, and middling bond energy.

Photon

A ____________ is a particle of light and is considered to be massless.

Titrant

A ____________ is the solution in a titration reaction whose concentration is accurately known.

titration curve

A ______________ can be used to determine the following: -The equivalence point of an acid/base reaction -pH of a solution at the equivalence point, which is determined by the strength of the acid and base.

Saturated Unsaturated Super-Saturated

A ______________ solution is a solution in which the concentration of solute is at its maximum value in the solution. A __________________ solution is a solution in which the concentration of solute is not at its maximum value in the solution. A __________________ solution is a solution in which the concentration of solute has exceeded its maximum value in the solution.

Polyvalent acid Polyvalent base

A _______________ acid is an acid that is capable of donating more than one acid equivalent (H3O+) A _______________ base is a base that is capable of donating more than one base equivalent (OH-) or accepting more than one proton.

Titration curve

A _______________ is a graph in which the pH of the analyte solution is plotted with the volume of the titrant added in the titration process.

Parent chain

A _______________ is the longest carbon chain containing the highest-order functional group in an organic molecule.

Combustion reaction

A _______________ reaction is a chemical reaction in which a fuel (usually a hydrocarbon) is mixed with an oxidant (usually an oxygen), to form carbon dioxide and water. (CH4 + 2O2 ---> CO2 + 2H2O)

Amphoteric species

A _______________ species is a substance that can behave as an acid or base depending on the reaction it is in.

Amphiprotic species

A _______________ species is an amphoteric species that can specifically act as either a Bronsted-Lowry acid or Bronsted-Lowry base.

Reducing agent Oxidizing agent

A ________________ agent is the chemical species in a chemical reaction that causes another atom to undergo reduction, while the agent itself is oxidized. A ________________ agent is the chemical species in a chemical reaction that causes another atom to undergo oxidation, while the agent itself is reduced.

Oxidizing Agent Reducing Agent

A ________________ agent usually contains an oxygen or another strongly electronegative element (such as halogens). A __________________ agent usually contains a metal ion or hydrides (H-).

Cation Anion

A ________________ is an atom that loses an electron and becomes positively charged. A ________________ is an atom that gains an electron and becomes positively charged.

Decomposition reaction

A ________________ reaction is a chemical reaction in which one product breaks down into two or more species. (AB ---> A + B)

Electrolyte More Electrolyte Less Electrolyte

A _________________ is a substance that creates ions in a solution. The __________ soluble a molecule is, the stronger the ______________________. The ___________ soluble a molecule is, the weaker the _______________.

Eluent Eluate

A _________________ is the mobile phase that is entering the stationary phase, while the ________________ is the mobile phase that is exiting the stationary phase.

Combination reaction

A _________________ reaction is a chemical reaction in which two or more species come together to form a product. (A + B ---> AB)

Covalent-nonpolar bond Covalent-polar bond Ionic bond

A __________________ bond is a chemical bond where the electrons in a bond are shared equally. A ___________________ bond is a chemical bond where the electrons in a bond are shared unequally. A ___________________ bond is a chemical bond where the electrons in a bond are not shared, one atom takes both electrons from the other atom it is bonded to.

Redox reaction

A __________________ is a chemical reaction that involves the transfer of electrons from one chemical species to another.

Double-displacement/metathesis reaction

A __________________ reaction is a reaction that involves the switching of counterions between two compounds. (AB + CD -----> CB + AD)

Disproportionation reaction

A __________________ reaction is a redox reaction in which an element undergoes both oxidation and reduction in producing a products. (2H2O2 ---> 2H2O + O2)

Trans-isomer

A _____________________ is a geometric isomer that has two identical substituents on opposite sides of a double bond or a ring structure.

racemic mixture

A _____________________ is a solution that contains 50% of one enantiomer and 50% of the other enantiomer.

coordinate covalent bond

A _______________________ bond is a covalent bond in which one atom contributes both bonding electrons

Line spectrum

A _______________________ is a spectrum of specific colors of light emitted when an electron goes from an excited state to a ground state.

Meso compound

A ________________________ is a molecule that has chiral centers, but also has an internal plane of symmetry.

acid-base neutralization reaction

A ______________________________ reaction is a reaction between an acid and a base to form a solution of a salt and water.

Q = mcΔT m = 3 kg = 3000 g c = 2.090 J/gK Tf = 0°C Ti = -40°C Q = 3000 g * 2.090 J/gK * (0°C - (-40)°C) Q = 250800 J Q(fus) = m * L(fus) m = 3000 g L(fus) = 333 J/g Q(fus) = 3000 g * 333 J/g Q(fus) = 999000 J Q = mcΔT m = 3000 g c = 4.184 J/gK Tf = 100°C Ti = 0°C Q = 3000 g * 4.184 J/gK * (100°C - 0°C) Q = 1255200 J Q(vap) = m * L(vap) m = 3000 g L(vap) = 2260 J/g Q(vap) = 3000 g * 2260 J/g Q(vap) = 6780000 J Q = mcΔT m = 3000 g c = 2.010 J/gK Tf = 160°C Ti = 100°C Q = 3000 g * 2.010 J/gK * (160°C - 100°C) Q = 361800 J 250800 999000 1255200 6780000 + 361800 9646800 J

A block of ice has a mass of 3 kg and is sitting at a temperature of -40°C. How much heat is needed to heat this block of ice to 160°C? (Specific heat of ice = 2.090 J/gK) (Specific heat of water = 4.184 J/gK) (Specific heat of steam = 2.010 J/gK) (Melting point of ice = 0°C) (Boiling point of water = 100°C) (L(fus) = 333 J/g) (L(vap) = 2260 J/g)

Tetrahedral Trigonal planar Linear

A carbon atom that is sp3 hybridized would have a ____________________ molecular geometry. A carbon atom that is sp2 hybridized would have a ____________________ molecular geometry. A carbon atom that is sp hybridized would have a ____________________ molecular geometry.

4 0 3 1 2 2

A carbon that is sp3 hybridized has _______ hybridized orbitals and _________ unhybridized p-orbitals. A carbon that is sp2 hybridized has _______ hybridized orbitals and _________ unhybridized p-orbitals. A carbon that is sp hybridized has ________ hybridized orbitals and _______ unhybridized p-orbitals.

Answer: A. To find the amount of heat needed to bring the substance to its melting point, you can use the specific heat. To heat one mole of the substance one unit Kelvin, it would take 1 J of heat. After the substance reaches its melting point, additional heat is needed to actually induce the phase change. Therefore, the total amount of heat required is greater than 1 J.

A certain substance has a specific heat of 1 J/mol*K and a melting point of 350 K. If one mole of the substance is currently at a temperature of 349 K, how much energy must be added in order to melt it? A. More than 1 J B. Exactly 1 J C. Less than 1 J but more than 0 J D. Less than 0 J

a) Since we have PbCl2 precipitate, we know that this is a saturated solution of Pb(2+) and Cl-. Out of the 10 grams of PbCl2 added, only 0.22 grams gets dissolved into solution. solubility g/L = 0.22g/0.05L = 4.4 g/L To find molar solubility, we must find the number of moles of the solute dissolved. 0.22 g/278.1 g/mol = 7.9 x 10^-4 mol solubility mol/L = 7.9 x 10^-4 mol/0.05 L = 0.016 M Solubility (g/L) = 4.4 g/L Molar Solubility (mol/L) = 0.016 M b) Since the solution is saturated, the rate of reaction of dissolution and precipitation is in equilibrium. PbCl2 (s) <-----> Pb(2+) (aq) + Cl- (aq) Balance reaction: 1PbCl2 (s) <-----> 1Pb(2+) (aq) + 2Cl- (aq) From a), we found out the molarity of PbCl2 is 0.016 M. Use ICE to find molarity of products: *multiply molarities by coefficients 1PbCl2 (s) <-----> 1Pb(2+) (aq) + 2Cl- (aq) N/A 0 0 - 1*(0.016) + 1*(0.016) + 2*(0.016) N/A 0.016 0.032 Ksp = [Pb(2+)][Cl-] (PbCl2 is left out because we always leave out pure solids and liquids in equilibrium expressions) Raise concentrations to the power of coefficients: Ksp = [Pb(2+)]^1 * [Cl-]^2 Ksp = [0.016 M]^1 * [0.032 M]^2 Ksp = 0.016 M * 0.001 M Ksp = 1.6 x 10^-5 M (Solving for Ksp is like finding Ka or Kb in acid-base equilibria) c) Nothing will change since only 0.22 g of that 100 g will be the total amount that will dissolve, regardless of how much solute is added. Molar solubility will still remain the same as well as our solubility product constant Ksp.

A chemist adds 10.00 g of PbCl2 to 50.00 mL of water at 25°C and finds that only 0.22 g of the solid goes into solution. (Molar mass PbCl2 = 278.1 g/mol) a) Calculate the solubility (in g/L and mol/L) of PbCl2 in water at 25°C b) Calculate the solubility product constant Ksp (at 25°C) for PbCl2. c) How would the results change if we had tried to dissolve 100 g of PbCl2 in the same volume of water?

ΔL = αLΔT α = 10^-6 K^-1 L = 2 m ΔT = 80 °C - 1080 °C = -1000 K ΔL = 10^-6 K^-1 * 2 m * -1000 K ΔL = -0.002 m ΔL = Lf - Li -0.002 m = Lf - 2 m Lf = 2 m - 0.002 m Lf = 1.998 m

A metal rod of length 2 m has a coefficient of linear expansion of 10^-6 K^-1. It is cooled from 1080 °C to 80 °C. What is the final length of the rod?

Fischer projection

A method of drawing organic molecules in which horizontal lines are coming out of the page (wedges) and vertical lines are going into the page (dashes) is known as a ___________________________________.

Trigonal Bipyramidal

A molecule that has five electron clouds, where all of them are bonding electron pairs, would have a ________________ molecular geometry.

Seesaw

A molecule that has five electron clouds, where one is a nonbonding electron pair and four are bonding electron pairs, would have a ___________________ molecular geometry.

T-shaped

A molecule that has five electron clouds, where two are nonbonding electron pairs and three are bonding electron pairs, would have a ________________ molecular geometry.

Tetrahedral

A molecule that has four electron clouds, where all of them are bonding electron pairs, would have a _______________ molecular geometry.

Trigonal Pyramidal

A molecule that has four electron clouds, where one is a nonbonding electron pair and the other three are bonding electron pairs, would have a _______________ molecular geometry.

Bent/Angular

A molecule that has four electron clouds, where two are nonbonding electron pairs and two are bonding electron pairs, would have a ________________ molecular geometry.

Octahedral

A molecule that has six electron clouds, where all of them are bonding electron pairs, would have a _____________________ molecular geometry.

Linear

A molecule that has six electron clouds, where four are nonbonding electron pairs and two are bonding electron pairs, would have a ________________ molecular geometry.

Square Pyramidal

A molecule that has six electron clouds, where one is a nonbonding electron pair and five are bonding electron pairs, would have a ___________________ molecular geometry.

T-shaped

A molecule that has six electron clouds, where three are nonbonding electron pairs and three are bonding electron pairs, would have a _______________ molecular geometry.

Square Planar

A molecule that has six electron clouds, where two are nonbonding electron pairs and four are bonding electron pairs, would have a __________________ molecular geometry.

Trigonal Planar

A molecule that has three electron clouds, where all of them are bonding electrons, would have a _________________ molecular geometry.

Bent/Angular

A molecule that has three electron clouds, where one is a nonbonding electron pair and the other two are bonding electron pairs, would have a ________________ molecular geometry.

Linear

A molecule that has two electron clouds, where both are bonding electron pairs, would have a ____________ molecular geometry.

Less

A molecule with ____________ intermolecular forces has a higher vapor pressure.

1. Step 1: Find the molar mass of all 4 compounds C4H7NO4 = 4(12) + 7(1) + 14 + 4(16) = 133 g/mol C5H6O5 = 5(12) + 6(1) + 5(16) = 146 g/mol C4H4O5 = 4(12) + 4(1) + 5(16) = 132 g/mol C5H9NO4 = 5(12) + 9(1) + 14 + 4(16) = 147 g/mol Step 2: Divide the molar mass of oxygen in each compound by total molar mass. C4H7NO4: 64 g/mol/133 g/mol = 48.12% C5H6O5: 80 g/mol/146 g/mol = 54.79% C4H4O5: 80 g/mol/132 g/mol = 60.61% C5H9NO4: 64 g/mol/147 g/mol = 43.54% Oxaloacetate (C4H4O5) has the greatest mass percentage of oxygen with 60.61%. 2. Write the reaction: C4H7NO4 + C5H6O5 ---> C5H9NO4 + C4H4O5 C5H6O5 = 7.3 g C4H7NO4 = 3.3 g Divide mass by molecular weight to find moles: C4H7NO4: 3.3 g * 1 mol = 0.025 mol 133 g/mol C5H6O5: 7.3 g * 1 mol = 0.05 mol 146 g/mol C4H7NO4 < C5H6O5 C4H7NO4 (aspartate) is our limiting reagent 3. From the previous question, we learned that aspartate is our limiting reagent. 3.3 g aspartate * 1 mol aspartate = 0.025 mol aspartate 133 g/mol aspartate 0.025 mol aspartate * 1 mol glutamate = 0.025 mol glutamate 1 mol aspartate 0.025 mol glutamate * 147 g glutamate = 3.675 g glutamate 1 mol glutamate %yield = actual yield * 100% theoretical yield %yield = 2.4 g glutamate * 100% = 65.31% 3.675 g glutamate

A researcher performs an experiment preparing the enzyme at a concentration of one mg/ml in 0.1 M potassium phosphate, pH 7.4. Immediately prior to use, she dilutes further in this buffer to a concentration of 0.05 - 0.25 u/mL. 3.3g of aspartate (C4H7NO4) and 7.3 g of α-ketoglutarate (C5H6O5) were added to the mix. After incubation, she was able to isolate 2.4 g of pure glutamate (C5H9NO4) from the reaction. The other product in the reaction is the α-keto acid oxaloacetate (C4H4O5). 1. Find the compound with the greatest mass percentage of oxygen. 2. Calculate the limiting reagent of the transaminase reaction. 3. Find the %yield for glutamate.

Answer: B. A resonance structure describes an arrangement of electrons in a molecule. Different resonance structures can be derived by moving electrons in unhybridized p-orbitals throughout a molecule containing conjugated bonds. In molecules that contain multiple resonance structures, some are usually more stable than others; however, each resonance structure is not necessarily the most common form a molecule takes, eliminating statement III. Statement I has reversed the terminology for resonance structures; the electron density in a molecule is the weighted average of all possible resonance structures, not the other way around.

A resonance structure describes: I. The hybrid of all possible structures that contribute to electron distribution II. The potential arrangement of electrons in a molecule III. The single form that the molecule most often takes A. I only B. II only C. I and II only D. I, II, and III

Find moles of NaI3: M = mol/L M = 0.7 M L = 32.60 mL = 0.0326 L mol = 0.7 * 0.0326 = 0.02282 mol Find mol of Pb(2+): 0.02282 mol NaI3 * (1 mol Pb(2+)/1 mol NaI3) = 0.02282 mol Pb(2+) Find mass of Pb(2+): g/(molar mass) = mol molar mass = 207.2 g mol = 0.02282 mol g = (molar mass) * mol g = 207.2 * 0.02282 = 4.73 g g Pb(2+) = 4.73 g (The 10.00 g is an extraneous value, and does not factor into the solution to the problem)

A sample is assayed for lead by a redox titration with I3- (aq). A 10.00 gram sample is crushed, dissolved in sulfuric acid, and passed over a reducing agent so that all the lead is in the form of Pb(2+). The Pb(2+) (aq) is completely oxidized to Pb(4+) by 32.60 mL of a 0.7 M solution of NaI3. The balanced equation for the reaction is: I3- (aq) + Pb(2+) (aq) ---> Pb(4+) (aq) + 3I- (aq) Calculate the mass of lead in this sample. (Molar mass Pb(2+) = 207.2 g)

Answer: D. Since both salts have a formula MX3 (one of one particle, and three of another), it is possible to directly compare the molar solubilities of each. When the solutions are mixed, [OH-] is above saturation levels for both the cobalt and the thallium in the solution. Since thallium (III) hydroxide has a smaller Ksp than that of cobalt(III) hydroxide, it will react first. The ion product of the mixed solution is higher than the Ksp for thallium(III) hydroxide, and the system will shift left to precipitate solid thallium(III) hydroxide. After the thallium(III) hydroxide precipitates, a small excess of OH- will remain, which gives an ion product slightly above the Ksp of cobalt(III) hydroxide. This will cause a small amount (1% - 3%) of cobalt(III) hydroxide to also precipitate.

A saturated solution of cobalt(III) hydroxide (Ksp = 1.6 x 10^-44) is added to a saturated solution of thallium(III) hydroxide (Ksp = 6.3 x 10^-46). What is likely to occur? A. Both cobalt(III) hydroxide and thallium(III) hydroxide will remain stable in solution. B. Cobalt(III) hydroxide precipitates and thallium(III) hydroxide remains stable in solution. C. Thallium(III) hydroxide precipitates and cobalt(III) hydroxide remains stable in solution. D. Both thallium(III) hydroxide and cobalt(III) hydroxide precipitate.

Answer: D. Remember that the equation for specific rotation is [α] = α/c*l. In this example, α is +12° (remember that dextrorotatory, or clockwise, rotation is considered positive), c = 0.5 g/mL, and l = 1 cm = 0.1 dm. Remember that path length is always measured in decimeters when calculating specific rotation. Therefore, the specific rotation can be calculated as: [α] = α/c*l = +12/(0.5 g/mL)(0.1 dm) = +240°

A scientist takes a 0.5 g/mol solution of an unknown pure dextrorotatory organic molecule and places it in a test tube with a diameter of 1 cm. He observes that a plane of polarized light is rotated 12° under these circumstances. What is the specific rotation of this molecule? A. -240° B. -24° C. +24° D. +240°

Hexane is a nonpolar compound, while propanamine is considered to be polar and basic. In order to form the aqueous and organic layer, we want to make sure that we can produce an ion from one compound, and leave the other compound uncharged. We can do this by adding in an acid. Say that we added HCl, which allows us to protonate the propanamine, which produces a Cl- anion. This will allow for our propanamine to separate into the aqueous layer, while hexane remains as the organic layer. Hexane is less dense than propanamine, meaning the aqueous phase will rest at the bottom while the organic phase rests at the top. The scientist can then turn the stopcock and extract the aqueous phase from the separatory funnel.

A scientist wants to extract propanamine from a solution consisting of it and hexane. How can the scientist do this?

1 2 3

A single bond has a bond order of _____________. A double bond has a bond order of _____________. A triple bond has a bond order of ______________.

1. Simple distillation 2. Acetone, Cyclohexane, Acetic acid

A student is trying to distill a mixture consisting of acetic acid, acetone, and cyclohexane. The boiling points of the respective compounds are 118°C, 56°C, and 81°C. 1. What kind of distillation should the student use? 2. Name the order in which the three compounds will be distilled.

1. The student should use simple distillation. 2. Hexane will distill first, since it has the lower boiling point

A student is trying to distill a mixture consisting of hexane and toulene. Hexane has a boiling point of 68°C while toulene has a boiling point of 110°C. 1. What kind of distillation should the student use? 2. Which compound will distill first?

1. Fractional distillation 2. Ethanol

A student is trying to distill a mixture consisting of water and ethanol. Water has a boiling point of 100°C while ethanol has a boiling point of 78°C. 1. What kind of distillation should the student use? 2. Which compound will distill first?

Answer: A. Calorimeters are our best approximations of isolated systems, where neither energy nor matter is exchanged with the environment. By failing to use an insulating layer and failing to fully contain the system, heat can be exchanged with the environment and matter may be dispersed, creating an open system.

A student making a coffee cup calorimeter fails to use a second coffee cup and inadequately seals the lid. What was her initial goal, and what was the result of this mistake? A. She was trying to create an isolated system but created an open system instead. B. She was trying to create an isolated system but created a closed system instead. C. She was trying to create a closed system but created an open system instead. D. She was trying to create a closed system but created an isolated system instead.

Strong electrolyte Weak electrolyte Nonelectrolyte

A substance is considered a _____________________ if it dissociates completely into its constituent ions in an aqueous solution. A substance is considered a ____________________ if it dissociates incompletely in an aqueous solution. A substance is considered a ____________________ if it does not dissociate at all into an aqueous solution, retaining their molecular structure in the solution.

Fluoro- Chloro- Bromo- Iodo-

Alkanes with halogen substituents (alkyl halides) are indicated by a prefix: ____________ = Fluorine substituent ____________ = Chlorine substituent ____________ = Bromine substituent ____________ = Iodine substituent

Answer: C. All atoms in the third period or greater have d-orbitals which can hold an additional 10 electrons. The typical "octet" electrons reside in s- and p-orbitals, but elements in period 3 or higher can place electrons into these d-orbitals.

Although the octet rule dictates much of molecular structure, some atoms can violate the octet rule by being surrounded by more than eight electrons. Which of the following is the best explanation for why some atoms exceed the octet? A. Atoms that exceed the octet already have eight electrons in their outermost electron shell B. Atoms that exceed the octet only do so when bonding with transition metals C. Atoms that exceed the octet can do so because they have d-orbitals in which extra electrons can reside D. Some atoms can exceed the octet because they are highly electronegative.

Answer: D. In the reaction, there is a single displacement, with the silver in silver oxide being replaced by the aluminum to form aluminum oxide. This single-displacement reaction also necessitates a transfer of electrons in an oxidation-reduction reaction; silver, for example, changes from the +2 oxidation state to neutral. Aluminum changes from neutral to the +3 oxidation state.

Aluminum metal can be used to remove tarnish from silver when the two solid metals are placed in water, according to the following reaction: 3 AgO + 2 Al ---> 3 Ag + Al2O3 This reaction is a: I. double-displacement reaction II. single-displacement reaction III. oxidation-reduction reaction IV. combination reaction A. II only B. IV only C. I and III only D. II and III only

Answer: D. The problem requires the MCAT favorite equation E = hc/λ, where h = 6.626 x 10^-34 J*s (Planck's constant), c = 3.00 x 10^8 m/s is the speed of light, and λ is the wavelength of the light. This question asks for the energy of one mole of photons, so we must multiply by Avogadro's number, Na = 6.02 x 10^23 1/mol. The setup is: E = hc/λ * Na 6.625 x 10^-34 J*s * 3 x 10^8 m/s * 6.02 x 10^23 1/mol 500 x 10^-9 m E = 2.39 x 10^5 J

An electron returns from an excited state to its ground state, emitting a photon at λ = 500 nm. What would be the magnitude of the energy change if one mole of these photons were emitted? (Note: h = 6.626 x 10^-34 J*s, Na = 6.02 x 10^23 1/mol) A. 3.98 x 10^-21 J B. 3.98 x 10^-19 J C. 2.39 x 10^3 J D. 2.39 x 10^5 J

H3O+ OH-

An equivalent for an acid would be ___________ while an equivalent for a base would be ____________.

25% 75% 4 33% 67% 3 50% 50% 2

An sp3 orbital contains _________% s character and _________% p character. In a sp3 hybridized atom, there are a total of ________ sp3 hybrid orbitals An sp2 orbital contains ___________% s character and __________% p character. In a sp2 hybridized atom, there are a total of _________ sp2 hybrid orbitals. An sp orbital contains ____________% s character and ____________% p character. In a sp hybridized atom, there are a total of __________ sp hybrid orbitals.

Alpha Beta Gamma Delta

Another naming convention used for ketones and aldehydes is naming the carbons in relation to the carbonyl carbon. Carbons adjacent to either side of the carbonyl carbon are known as _________________ carbons. Moving further away from the carbonyl carbon, the successive carbons are referred to as _____________, ________________, and ______________ carbons.

-Beads that are coated with specific receptors, enzymes, or antibodies (solid) -A solvent (liquid) -Binding affinity to molecules in stationary phase

Answer the following in regards to affinity chromatography: -What is the stationary phase? -What is the mobile phase? -What is the basis of separation of components?

-Beads coated with charged substances (solid) -A solvent (liquid) -Charge of molecules

Answer the following in regards to ion-exchange chromatography: -What is the stationary phase? -What is the mobile phase? -What is the basis of separation of components?

-Silica gel or alumina (solid) -A nonpolar solvent (liquid) -Polarity of molecules

Answer the following in regards to liquid column chromatography: -What is the stationary phase? -What is the mobile phase? -What is the basis of separation of components?

-Cellulose paper (solid) -Nonpolar solvent (liquid) -Polarity of molecules

Answer the following in regards to paper chromatography: -What is the stationary phase? -What is the mobile phase? -What is the basis of separation of components?

-Microporous beads (solid) -A solvent (liquid) -Size of molecules

Answer the following in regards to size-exclusion chromatography: -What is the stationary phase? -What is the mobile phase? -What is the basis of separation of components?

-Inert metal plate coated in silica gel or alumina (solid) -Nonpolar solvent (liquid) -Polarity of molecules

Answer the following in regards to thin layer chromatography: -What is the stationary phase? -What is the mobile phase? -What is the basis of separation of components?

They are not optically active. They may have a chiral center in the molecule, but they also have a plane of symmetry, which prevents it from being optically active.

Are meso molecules optically active? Why or why not?

1. S 2. R 3. Neither, it's achiral 4. R 5. S

Are the following R or S?

Different Different

Are the physical properties between constitutional isomers the same or different? How about chemical properties?

The oxidation state of Cl in a compound is typically -1, unless paired with a more electronegative element like O. +2 -1 +4 -1 +4 -1 +2 -1 SnCl2 + PbCl4 -------> SnCl4 + PbCl2 +2 -2 +4 -4 +4 -4 +2 -2 +2 -1 +4 -1 SnCl2 -------> SnCl4 Sn loses electrons, which means it is undergoing an oxidation reaction. Therefore, it is the reducing agent in the redox reaction. +4 -1 +2 -1 PbCl4 -------> PbCl2 Pb gains electrons, which means that it is undergoing a reduction reaction. Therefore, it is the oxidizing agent in the redox reaction.

Assign oxidation numbers to the atoms in the following reaction to determine the oxidizing and reducing agents. SnCl2 + PbCl4 -------> SnCl4 + PbCl2

Residue Filtrate

At the end of filtration, the solid left on the filter is known as the ______________, while the liquid that passed through the filter is known as the _______________.

Answer: C. The second equivalence point is the midpoint of the second steep increase in slope. This corresponds to approximately pH = 5.9.

At what pH is the second equivalence point in the titration curve shown? A. pH = 3.0 B. pH = 4.1 C. pH = 5.9 D. pH = 7.2

unique

Atomic absorption spectrum and atomic emission spectrum is ______________ for each element.

4Al + 3O2 ---> 2Al2O3

Balance the following reaction: Al + O2 ---> Al2O3

C2H4 + 3O2 ---> 2CO2 + 2H2O

Balance the following reaction: C2H4 (g) + O2 (g) ---> CO2 (g) + H2O (l)

2C2H6 + 7O2 ---> 4CO2 + 6H2O

Balance the following reaction: C2H6 (g) + O2 (g) ---> CO2 (g) + H2O (l)

2C4H10 + 13O2 ---> 8CO2 + 10H2O

Balance the following reaction: C4H10 (l) + O2 (g) ---> CO2 (g) + H2O (l)

C5H12 + 8O2 ---> 5CO2 + 6H2O

Balance the following reaction: C5H12 + O2 ---> CO2 + H2O

2Fe + 3Cl2 ---> 2FeCl3

Balance the following reaction: Fe + Cl2 ---> FeCl3

1. Split the reaction into half reactions +1 -2 -1 ClO- -------> Cl- reduction +3 -1 +6 -2 Cr(OH)4- -------> CrO4(2-) oxidation +3 -4 +6 -8 2. Balance all non- H and O atoms ClO- -------> Cl- Cr(OH)4- -------> CrO4(2-) 3. Balance all O atoms with water ClO- -------> Cl- + H2O Cr(OH)4- -------> CrO4(2-) 4. Balance all H atoms with H+ ions 2H+ + ClO- -------> Cl- + H2O Cr(OH)4- -------> CrO4(2-) + 4H+ 5. Balance charges by adding electrons +1 -1 +2 -1 -1 0 2e- + 2H+ + ClO- -------> Cl- + H2O +2 -1 -2 +4 Cr(OH)4- -------> CrO4(2-) + 4H+ + 3e- 6. Make the electrons in both half reactions equal 3(2e- + 2H+ + ClO- -------> Cl- + H2O) 6e- + 6H+ + 3ClO- -------> 3Cl- + 3H2O 2(Cr(OH)4- -------> CrO4(2-) + 4H+ + 3e-) 2Cr(OH)4- -------> 2CrO4(2-) + 8H+ + 6e- 7. Combine the half reactions and cancel out the electrons and any spectator molecules 3ClO- + 2Cr(OH)4- -------> 3Cl- + 3H2O + 2CrO4(2-) + 2H+ 8. Add OH- ions to both sides to neutralize H+ 3ClO- + 2Cr(OH)4- -------> 3Cl- + 3H2O + 2CrO4(2-) + 2H+ +2OH- +2OH- 2H2O 3ClO- + 2Cr(OH)4- + 2OH- -------> 3Cl- + 5H2O + 2CrO4(2-)

Balance the following redox reaction in a basic solution: ClO- + Cr(OH)4- -------> CrO4(2-) + Cl-

1. Split the reaction into half reactions +6 -2 +3 Cr2O7(2-) -------> Cr(3+) reduction +12 -14 +3 -1 0 Cl- -------> Cl2 oxidation -1 0 2. Balance all non- H and O atoms Cr2O7(2-) -------> 2Cr(3+) 2Cl- -------> Cl2 3. Balance all O atoms with water Cr2O7(2-) -------> 2Cr(3+) + 7H2O 2Cl- -------> Cl2 4. Balance all H atoms with H+ ions 14H+ + Cr2O7(2-) -------> 2Cr(3+) + 7H2O 2Cl- -------> Cl2 5. Balance charges by adding electrons +12 +6 +14 -2 +6 0 6e- + 14H+ + Cr2O7(2-) -------> 2Cr(3+) + 7H2O -2 0 2Cl- -------> Cl2 + 2e- 6. Make the electrons in both half reactions equal 6e- + 14H+ + Cr2O7(2-) -------> 2Cr(3+) + 7H2O 3(2Cl- -------> Cl2 + 2e-) 6Cl- -------> 3Cl2 + 6e- 7. Combine both half reactions, and cancel out electrons on both sides 14H+ + Cr2O7(2-) + 6Cl- -------> 2Cr(3+) + 7H2O + 3Cl2

Balance the following redox reaction in an acidic solution: Cr2O7(2-) + Cl- -------> Cr(3+) + Cl2

1. Split the reaction into half reactions 0 2+ Mg -------> Mg(2+) oxidation +1+5-2 +2-2 HNO3 -------> NO reduction +1+5-6 +2-2 2. Balance the non- H and O atoms Mg -------> Mg(2+) HNO3 -------> NO 3. Balance the O atoms with water Mg -------> Mg(2+) HNO3 -------> NO + 2H2O 4. Balance the H atoms with H+ ions Mg -------> Mg(2+) 3H+ + HNO3 -------> NO + 2H2O 5. Balance the charges by adding electrons 0 +2 Mg -------> Mg(2+) + 2e- +3 0 0 0 3e- + 3H+ + HNO3 -------> NO + 2H2O 6. Make the electrons in both half reactions equal 3(Mg -------> Mg(2+) + 2e-) 3Mg -------> 3Mg(2+) + 6e- 2(3e- + 3H+ + HNO3 -------> NO + 2H2O) 6e- + 6H+ + 2HNO3 -------> 2NO + 4H2O 7. Combine the half reactions 3Mg + 6H+ + 2HNO3 -------> 3Mg(2+) + 2NO + 4H2O

Balance the following redox reaction using the half-reaction method: Mg (s) + HNO3 (aq) -------> Mg(2+) (aq) + NO (g)

1. Separate into two half reactions MnO4- -------> Mn(2+) reduction I- -------> I2 oxidation 2. Balance all the atoms except for H and O in each reaction MnO4- -------> Mn(2+) 2I- -------> I2 3. Balance all the O atoms by introducing water MnO4- -------> Mn(2+) + 4H2O 2I- -------> I2 4. Balance all the H atoms by introducing H+ ions: MnO4- + 8H+ -------> Mn(2+) + 4H2O 2I- -------> I2 5. Balance the charges by adding electrons +7 +2 -1 8+ 2+ 0 5e- + MnO4- + 8H+ -------> Mn(2+) + 4H2O -2 0 2I- -------> I2 + 2e- 6. Make the number of electrons equal in both half reactions: 2(5e- + MnO4- + 8H+ -------> Mn(2+) + 4H2O) 10e- + 2MnO4- + 16H+ -------> 2Mn(2+) + 8H2O 5(2I- -------> I2 + 2e-) 10I- -------> 5I2 + 10e- 7. Add the half reactions together, and cancel out electrons 2MnO4- + 16H+ + 10I- -------> 2Mn(2+) + 8H2O + 5I2

Balance this redox reaction using the half-reaction method: MnO4- + I- -------> I2 + Mn(2+)

methyl red

Based on the curve shown, what would be the best indicator to use for this titration?

Answer: C. The central carbon in carbonate has no lone pairs. It has three resonance structures, each of which involves a double bond between carbon and one of the three oxygens. Having made four bonds, carbon has no further orbitals for bonding or to carry lone pairs. This makes carbonate's geometry trigonal planar. Alternatively, ClF3 also has three bonds, however, chloride still maintains two extra lone pairs. These lone pairs each inhabit one orbital, meaning that the central chloride must organize five items about itself: three bonds to fluorides and two lone pairs. The best configuration for maximizing the distance between all these groups is trigonal bipyramidal. (A.) and (B.) are true statements, but do not account for the difference in geometry.

Both CO3(2-) and ClF3 have three atoms bonded to a central atom. What is the best explanation for why CO3(2-) has a trigonal planar electronic geometry, while ClF3 has a trigonal bipyramidal electronic geometry? A. CO3(2-) has multiple resonance structures, while ClF3 does not B. CO3(2-) has a charge of -2, while ClF3 has no charge C. ClF3 has lone pairs on its central atom, while CO3(2-) has none D. CO3(2-) has lone pairs on its central atom, while ClF3 has none

Ka * Kb = Kw Ka * (3.7 x 10^-4) = (1 x 10^-14) Ka = (1 x 10^-14)/(3.7 x 10^-4) = (2.7 x 10^-11) Ka = 2.7 x 10^-11 pKa: pKa = -log[Ka] = -log[2.7 x 10^-11] = 10.57 pKa = 10.57 pKb: pKb = -log[Kb] = -log[3.7 x 10^-4] = 3.43 or... pKa + pKb = 14 pKb = 14 - 10.57 = 3.43 pKb = 3.43

Calculate the Ka value for the methylammonium ion (CH3NH3+). Kb for methylamine (CH3NH2) is 3.7 x 10^-4 What is the pKa value of CH3NH3+? What is the pKb value of CH3NH2?

Molar solubility = molarity Ni(OH)2 (s) <---> Ni(2+) (aq) + 2OH- I 0 0 C -x +x +2x E x 2x Ksp = [Ni(2+)] * [2OH-]^2 Ksp = [5.2 x 10^-6] * [2 * (5.2 x 10^-6)]^2 Ksp = 5.2 x 10^-6 * 1.1 x 10^-10 Ksp = 5.7 x 10^-16

Calculate the Ksp of Ni(OH)2 in water, given that its molar solubility is 5.2 x 10^-6 M.

CH3COOH + H2O <-------> CH3COO- + H3O+ I 2.0 0 0 C -x +x +x E 2.0 - x x x Ka = [CH3COO-][H3O+]/[CH3COOH] 1.8 x 10^-5 = [x][x]/[2.0 - x] (assume x << 1) 1.8 x 10^-5 = x^2/2.0 2.0 * 1.8 x 10^-5 = x^2 = 3.6 x 10^-5 sqrt(3.6 x 10^-5) = x x = 6 x 10^-3 Ka = 6 x 10^-3

Calculate the concentration of H3O+ in a 2.0 M aqueous solution of acetic acid, CH3COOH. (Ka = 1.8 x 10^-5)

E = h*c/λ 662 nm = 662 x 10^-9 m = 6.62 x 10^-7 m E = 6.626 x 10^-34 J*s * 3 x 10^8 m/s/6.62 x 10^-7 m E = 3.00 x 10^-19 J

Calculate the energy of a photon of wavelength 662 nm. (Note: h = 6.626 x 10^-34 J*s)

NaBr: Na = 23 g/mol Br = 80 g/mol NaBr = 103 g/mol SrCl2: Sr = 87.6 g/mol Cl = 35.5 * 2 = 71 g/mol 71 + 87.6 = 158.6 g/mol SrCl2 = 158.6 g/mol C6H12O6: C = 12 * 6 = 72 H = 1 * 12 = 12 O = 16 * 6 = 96 96 + 12 + 72 = 180 C6H12O6 = 180 g/mol

Calculate the molar masses of the following substances: NaBr SrCl2 C6H12O6

a) 1AgCl (s) <-----> 1Ag+ (aq) + 1Cl- (aq) I N/A 0 0 C -x +x +x E x x Ksp = [Ag+]^1 * [Cl-]^1 1.8 x 10^-10 = x^2 sqrt(1.8 x 10^-10) = sqrt(x^2) x = 1.3 x 10^-5 [Ag+] = x = [AgCl] Molar Solubility = 1.3 x 10^-5 M b) The addition of NH3 results in the formation of Ag(NH3)2 complexes Ag+ (aq) + 2NH3 (aq) <-----> Ag(NH3)2+ Because Kf is high, the reaction favors the products, meaning that the reaction is exothermic. Thus, Ag+ will be taken by the NH3 and essentially removed from the solution, which leads to a decrease in Ag+ ions. This all leads to an increase in solubility for AgCl. Form a net reaction between the dissolution and formation reactions: AgCl (s) + Ag+ (aq) + 2NH3 (aq) <-----> Ag+ (aq) + Cl- (aq) + Ag(NH3)2+ (aq) AgCl (s) + 2NH3 (aq) <-----> Ag(NH3)2+ (aq) + Cl- (aq) Multiply the two equilibrium constants of the two reactions: 1.8 x 10^-10 * 1.6 x 10^7 2.9 x 10^-3 K = 2.9 x 10^-3 Use ICE for net reaction: AgCl (s) + 2NH3 (aq) <-----> Ag(NH3)2+ (aq) + Cl- (aq) I N/A 3.0 M 0 0 C -x -2x +x +x E N/A 3.0 - 2x x x K = ([Ag(NH3)2+]^1 * [Cl-]^1)/([NH3]^2) 2.9 x 10^-3 = x^2/(3.0 - 2x)^2 Since K > 1 x 10^-4, we cannot assume x<<1 2.9 x 10^-3 = (x/(3.0 - 2x))^2 sqrt(2.9 x 10^-3) = sqrt((x/(3.0 - 2x))^2) 0.054 = x/(3.0 - 2x) 0.054 * (3.0 - 2x) = x 0.016 - 0.11x = x 0.016 = 1.11x 0.016/1.11 = 1.11x/1.11 x = 0.14 M Molar Solubility in 3.0 M NH3 = 0.14 M

Calculate the molar solubility of AgCl (Ksp = 1.8 x 10^-10 at 25°C) in... a) pure water b) 3.0 M NH3 (Kf = 1.6 x 10^7 at 25°C)

KCl shares a common ion with PbCl2 PbCl2 (s) <-----> Pb(2+) (aq) + Cl- (aq) Balance reaction: 1PbCl2 (s) <-----> 1Pb(2+) (aq) + 2Cl- (aq) Use ICE: 1PbCl2 (s) <-----> 1Pb(2+) (aq) + 2Cl- (aq) I N/A 0 0.1 C -x + 1x + 2x E N/A x 2x + 0.1 Ksp = [Pb(2+)]^1 * [Cl-]^2 1.6 x 10^-5 = x * (2x + 0.1)^2 Since Ksp < 10^-4, to make things easier, we assume x<<1 1.6 x 10^-5 = x * (0.1)^2 1.6 x 10^-5 = x * 0.01 x = 0.0016 M x = [Pb(2+)] = [PbCl2] Molar solubility = 0.0016 M in KCl

Calculate the molar solubility of PbCl2 (Ksp = 1.6 x 10^-5 at 25°C) in a 0.100 M solution of KCl.

Remember: Na+ cations will not react with water, but CH3COO- anions will, since this salt is formed from a weak acid and a strong base. So make a chemical equation for CH3COO- and water. CH3COO- + H2O <------> CH3COOH + OH- I 0.25 0 0 C -x +x +x E 0.25 - x x x Kb = [CH3COOH][OH-]/[CH3COO-] Kb = [x][x]/[0.25 - x] (assume x<<1) Don't know the Kb of CH3COO-, so we find it by using the Ka of acetic acid. Ka * Kb = Kw 1.8 x 10^-5 * Kb = 1.0 x 10^-14 Kb = 1.0 x 10^-14/1.8 x 10^-5 = 5.6 x 10^-10 5.6 x 10^-10 = x^2/0.25 0.25 * 5.6 x 10^-10 = x^2 1.4 x 10^-10 = x^2 sqrt(1.4 x 10^-10) = x x = 1.2 x 10^-5 = [OH-] pOH = -log[OH-] = -log[1.2 x 10^-5] = 4.92 pH + pOH = 14 pH = 14 - 4.92 = 9.08 pH = 9.08 Solution is basic

Calculate the pH of 0.25 M solution of CH3COO-Na+ and state whether the solution is acidic, basic, or neutral. Ka of CH3COOH = 1.8 x 10^-5

Nitric acid (HNO3) is a strong acid. Strong acids undergoes 100% ionization in a solution. HNO3 + H2O 100%-----> H3O+ + NO3- Since ionization is 100%, [HNO3] = [H3O+] [H3O+] = 0.030 M pH = -log[H3O+] = -log(0.030) pH = 1.52 pH = 1.52

Calculate the pH of a 0.030 M HNO3 solution.

Remember: Cl- anions will not affect pH, NH4+ cations will. So make a chemical equation for NH4+ and water. NH4+ + H2O <---------> NH3 + H3O+ I 0.050 0 0 C -x +x +x E 0.050 - x x x Ka = [x][x]/[0.050 - x] (assume x << 1) We don't know the Ka of NH4+, so we use the Kb of the conjugate base NH3 to find Ka. Ka * Kb = Kw Ka * 1.8 x 10^-5 = 1.0 x 10^-14 Ka = 1.0 x 10^-14/1.8 x 10^-5 = 5.6 x 10^-10 5.6 x 10^-10 = x^2/0.050 0.050 * 5.6 x 10^-10 = x^2 2.8 x 10^-11 = x^2 sqrt(2.8 x 10^-11) = x = 5.3 x 10^-6 x = [H3O+] pH = -log[H3O+] = -log[5.3 x 10^-6] = 5.28 pH = 5.28 Solution is acidic

Calculate the pH of a 0.050 M solution of NH4+Cl- and state whether the solution is acidic, basic, or neutral. Kb of NH3 = 1.8 x 10^-5

Since NaOH is a strong base, it will dissociate at 100%. NaOH -----> Na+ + OH- Since ionization is 100%, [NaOH] = [OH-] [OH-] = 0.20 M pOH = -log[OH-] = -log(0.20) = 0.70 pOH + pH = 14 pH = 14 - pOH = 14 - 0.70 = 13.30 pH = 13.30

Calculate the pH of a 0.20 M solution of NaOH.

[NH3] = 0.500 M NH3 + H2O <-------> NH4+ + OH- I 0.500 0 0 C -x +x +x E 0.500 - x x x Kb = [NH4+][OH-]/[NH3] 1.8 x 10^-5 = [x][x]/[0.500 - x] assume x is negligible 1.8 x 10^-5 = x^2/0.500 (1.8 x 10^-5) * 0.500 = x^2 9.0 x 10^-6 = x^2 sqrt(9.0 x 10^-6) = x = 3 x 10^-3 x = [OH-] = 3 x 10^-3 pOH = -log[OH-] = -log(3 x 10^-3) = 2.52 pH + pOH = 14 pH = 14 - 2.52 pH = 11.48

Calculate the pH of a 0.500 M solution of NH3. Kb = 1.8 x 10^-5

We're including the 0.15 M NH3 along with the 0.35 M NH4NO3. NH3 + H2O <----------> NH4+ + OH- I 0.15 0.35 0 C -x +x +x E 0.15 - x 0.35 + x x Kb = [NH4+][OH-]/[NH3] 1.8 x 10^-5 = [0.35 + x][x]/[0.15 - x] (assume x << 1) 1.8 x 10^-5 = 0.35 * x/0.15 (0.15 * 1.8 x 10^-5)/0.35 = x x = 7.7 x 10^-6 x = [OH-] pOH = -log[OH-] = -log[7.7 x 10^-6] = 5.11 pH + pOH = 14 pH = 14 - 5.11 = 8.89 pH = 8.89

Calculate the pH of a solution that is 0.15 M NH3 (Kb = 1.8 x 10^-5) and 0.35 M NH4NO3.

We're including the 1.00 M of CH3COONa along with the 1.00 M of CH3COOH. CH3COOH + H2O <--------> CH3COO- + H3O+ I 1.00 1.00 0 C -x +x +x E 1.00 - x 1.00 + x x Ka = [H3O+][CH3COO-]/[CH3COOH] 1.8 x 10^-5 = [x][1.00 + x]/[1.00 - x] (assume x << 1) 1.8 x 10^-5 = x * 1.00/1.00 x = 1.8 x 10^-5 x = [H3O+] pH = -log[H3O+] = -log[1.8 x 10^-5] = 4.74 pH = 4.74

Calculate the pH of a solution that is 1.00 M of CH3COOH (Ka = 1.8 x 10^-5) and 1.00 M of NaOH.

Find moles: mol = g/molar mass mol = 0.11 g/74 g/mol = 0.0015 mol Multiply by 2, since there are two moles of OH- in Ca(OH)2. 0.0015 x 2 = 0.0030 mol Find concentration (molarity): molarity = moles/liters convert: 250 mL/1000 = 0.250 L molarity = 0.0030 mol/0.250 L = 0.012 M [Ca(OH)2] = 0.012 M Ca(OH)2 -----> Ca(2+) + 2OH- Since 100% ionization, [Ca(OH)2] = [OH-] [OH-] = 0.012 M pOH = -log[OH-] = -log(0.012) = 1.92 pH + pOH = 14 pH = 14 - 1.92 = 12.08 pH = 12.08

Calculate the pH of an aqueous solution that contains 0.11 g of Ca(OH)2 in a total volume of 250 mL. (molar mass of Ca(OH)2 is 74 g/mol)

Find moles: 0.52 g/ 40 g/mol = 0.013 mol Find molarity: 0.013 mol/ 0.431 L = 0.03 M Find pH pOH = -log[0.03] = 1.52 14 - 1.52 = 12.48 pH = 12.48

Calculate the pH of an aqueous solution that contains 0.52 g of NaOH in a total volume of 431 mL. (molar mass of NaOH is 40 g/mol)

Find moles: (0.56 g/ 36 g/mol) = 0.0156 mol Find molarity: 0.0156 mol/ 0.576 L = 0.027 M Find pH: -log[0.027] = 1.57 pH = 1.57

Calculate the pH of an aqueous solution that contains 0.56 g of HCl in a total volume of 576 mL. (molar mass of HCl is 36 g/mol)

Find moles: 0.62 g/ 171 g/mol = 0.0036 Ba(OH)2 has 2 acid equivalents 0.0036 x 2 = 0.0072 mol Find molarity: 0.0072 mol/ 0.35 L = 0.021 M Find pH: pOH = -log[0.021] = 1.68 14 - 1.68 = 12.32 pH = 12.32

Calculate the pH of an aqueous solution that contains 0.62 g of Ba(OH)2 in a total volume of 350 mL. (molar mass of Ba(OH)2 is 171 g/mol)

Find moles: (0.3 g/ 98 g/mol) = 0.007 H2SO4 has 2 acid equivalents 0.007 x 2 = 0.014 mol Find molarity: 0.014 mol/ 0.051 L = 0.275 M Find pH: -log [0.275] = 0.56 pH = 0.56

Calculate the pH of an aqueous solution that contains 0.7 g of H2SO4 in a total volume of 51 mL. (molar mass of H2SO4 is 98 g/mol)

Find solubility in both mol/L and g/L Write and balance the reaction: Cu(OH)2 (s) <-----> Cu(2+) (aq) + OH- (aq) 1Cu(OH)2 (s) <-----> 1Cu(2+) (aq) + 2OH- (aq) I N/A 0 0 C - 1x + 1x + 2x E x 2x Ksp = [Cu(2+)]^1 * [OH-]^2 2.2 x 10^-20 = x^1 * (2x)^2 2.2 x 10^-20 = x^1 * 4x^2 2.2 x 10^-20 = 4x^3 2.2 x 10^-20 = 4x^3 4 4 5.5 x 10^-21 = x^3 cbrt(5.5 x 10^-21) = cbrt(x^3) x = 1.8 x 10^-7 M [Cu(2+)] = x [Cu(OH)2] = x so... [Cu(2+)] = [Cu(OH)2] Therefore: Molar solubility of Cu(OH)2 = 1.8 x 10^-7 M Now find solubility in g/L 1.8 x 10^-7 M = 1.8 x 10^-7 mol/L Multiply by molar mass: 1.8 x 10^-7 mol/L * 97.57 g/mol = 1.8 x 10^-5 g/L Solubility = 1.8 x 10^-7 mol/L & 1.8 x 10^-5 g/L

Calculate the solubility of copper (II) hydroxide. (Ksp = 2.2 x 10^-20 at 25°C).

Answer: A. As one moves from top to bottom in a group (column), extra electron shells accumulate, despite the fact that the valence configurations remain identical. These extra electron shells provide shielding between the positive nucleus and the outermost electrons, decreasing electrostatic attraction and increasing atomic radius. Because carbon and silicon are in the same group, and silicon is farther down in the group, silicon will have a larger atomic radius because of its extra electron shell.

Carbon and silicon are the basis of biological life and synthetic computing, respectively. While these elements share many chemical properties, which of the following best describes a difference between the two elements? A. Carbon has a smaller atomic radius than silicon B. Silicon has a smaller atomic radius than carbon C. Carbon has fewer valence electrons than silicon D. Silicon has fewer valence electrons than carbon

-ane -yl alkyl straight-chain

Carbon chain substituents are named like their alkane counterparts, with the suffix ____________ being replaced by ____________. An n-prefix in _________ substituents meant the substituent is "normal" or a _________________ alkane.

Two Hybridization 2s 2p promote demote hybridized sp3 hybrid

Carbon has 4 valence electrons, two paired in the 2s orbital and two unpaired in two of the 2p orbitals. The problem with this configuration implies that it implies that it can only form _______ bonds since there are only two unpaired electrons in 2p orbital, which does not match electron configuration. ____________________ is used to explain this by removing one electron from the _______ orbital and placing it in the empty ________ orbital. Then we ____________ the 2s orbital and _____________ the 2p orbitals forming 4 _____________________ orbitals known as ______ orbitals. This ___________ orbital allows us to have 4 unpaired electrons at the same energy level.

Kinetic energy (movement of particles) Potential energy (intramolecular and intermolecular forces)

Changes in temperature relates to changes in ______________ energy, while phase changes relates to changes in __________________ energy.

Water Neutral state Less dense Reactive Valence electron Z(eff) Effective Nuclear Charge Large Low Low Low One

Characteristics of alkali metals: -Highly reactive to ________ -Not found in _________ _________ in nature -________ __________ than other metals -Extremely _____________, easily able to lose ___________ ___________ to form a univalent cation. -They have a very low _____________ -___________ atomic radii, _______ electronegativities, ________ ionization energies, _______ electron affinity. -Have ________ valence electron

Water Neutral State Reactive Valence electrons Higher Smaller More Greater Two

Characteristics of alkaline earth metals: -Highly reactive to ________ -Not found in __________ _________ in nature -Extremely ___________, easily able to lose ____________ _____________ to form a divalent cation. -Z(eff) charge is slightly ___________ than alkali metals -Slightly ___________ atomic radii than alkali metals -Slightly ____________ electronegative than alkali metals -Slightly ____________ ionization energy and electron affinity than alkali metals. -Have ___________ valence electrons

Nonmetals Metals and metalloids Atomic radii electronegative ionization energy electron affinity Six

Characteristics of chalcogens: -____________ are crucial for biological functions -___________ and __________________ are genrally toxic -Small _____________ ___________, highly __________________, great _________________ ______________ and _________________ ________________ -Have ________ valence electrons

reactive neutral form Smaller More Greater Seven

Characteristics of halogens: -Very _____________ nonmetals, they don't appear in ___________ __________ in nature. -_____________ atomic radii than chalcogens -____________ electronegative than chalcogens -______________ ionization energy and electron affinity than chalcogens -Has ___________ valence electrons

Good Metals luster brittle

Characteristics of metalloids: -________ conductivity of electricity, not as great as ___________ -May have a metallic __________ -___________, not very malleable.

solid malleable ductile good

Characteristics of metals: -Typically ___________ at room temperature -_____________, able to alter its shape -____________, can be drawn into wires -__________ conductors of heat and electricity

Ionization energy Electron affinity Unreactive Bond Eight

Characteristics of noble gases: -High ________________ __________________ -Low/nonexistent ________________ ______________ -Very ______________, not very willing to ___________ -Has __________ valence electrons

states of matter brittle poor

Characteristics of nonmetals: -Can be found in a variety of different ___________ of _____________ at room temperature - ___________, they are not very malleable when solid -_________ conductors of electricity

low low low hard high oxidation states d-orbitals

Characteristics of transition metals: -_______ electron affinities, ________ ionization energies, ________ electronegativities. -Very ______, __________ melting and boiling points -Are able to exist in multiple different _____________ __________, due to having ______________ in their valence shell.

Answer: C. To be considered a chiral center, a carbon must have four different substituents. There are eight stereocenters in this molecule. The other carbons are not chiral for various reasons, Many are bonded to two hydrogens; others participate in double bonds, which count as two bonds to the same atom.

Cholesterol contains how many carbon centers? A. 5 B. 7 C. 8 D. 9

I I \ /\ / \ / \ / \ / \ / I I I I / / Ethyl Methyl Methyl 4-ethyl-2,3-dimethylheptane

Circle and name the substituents in the following molecule, then name the molecule.

CH2CH3 I CH3CH - CH2 - COOH /\ OH \ / \ / \ I / ________I__________/ / / \ \ H3C - CH3

Circle or highlight the parent chain in each of the following compounds.

1. Combination 2. Neutralization 3. Decomposition 4. Double displacement 5. Single displacement

Classify the following reactions: 2H2 + O2 ---> 2H2O Al(OH)3 + H3PO4 ---> 3H2O + AlPO4 2H2O ---> 2H2 + O2 NaNO3 + CuOH ---> NaOH + CuNO3 Zn + AgCl ---> ZnCl + Ag

Answer: D. π bonds do not permit free rotation, unlike σ bonds; this makes triple bonds more rigid than single bonds. Triple bonds are stronger and shorter bonds than single bonds, eliminating (A.) and (B.). Both single and triple bonds contain one σ bond, eliminating (C.).

Compared to single bonds, triple bonds are: A. Weaker B. Longer C. Made up of fewer σ bonds D. More rigid

Conjugated Isolated lone pair positive charge negative charge free radical four atoms

Conjugated system: -Two double bonds are considered ________________ if they are separated by only one single bond. -Two double bonds are considered ______________ if they are separated by more than just one single bond. -If an atom that is between two pi bonds has a _____________, a _________________, a __________________, or a ____________________, it can make the pi bonds conjugated. -If an atom that is between two pi bonds is already bound to _________________, then the pi bonds can not be conjugated, even if the atom has any of the previously mentioned properties.

Answer: C. (E)-2-butene can also be called trans-2-butene; (Z)-2-butene can also be called cis-2-butene. As such, they are cis-trans isomers. Remember that cis-trans isomers are a subtype of diastereomers in which the position of substituents differs about an immovable bond. Diastereomers are molecules that are non-mirror-image stereoisomers (molecules with the same atomic connectivity). These are not enantiomers because they are not mirror images of each other.

Consider (E)-2-butene and (Z)-2-butene. This is a pair of what type(s) of isomers? I. Cis-trans isomers II. Diastereomers III. Enantiomers A. I only B. II only C. I and II only D. I and III only

Answer: C. The oxidizing agent is the species that is reduced in any given equation. In this problem, six hydrogen atoms with +1 oxidation states in NH3 are reduced to three neutral H2 molecules.

Consider the following equation: 6Na (s) + 2 NH3 (aq) ---> 2Na3N (s) + 3H2 (g) Which species acts as the oxidizing agent? A. Na B. N in NH3 C. H in NH3 D. H2

Answer: A. Step I is a disproportionation reaction because chlorine starts with an oxidation state of 0 in the reactants and ends up with an oxidation state of +1 in HOCl and -1 as Cl-. In the other reactions, no element appears with different oxidation states in two different products. Therefore, only step I is a disproportionation reaction.

Consider the following steps in the reaction between oxalic acid and chlorine: I. Cl2 + H2O ---> HOCl + Cl- + H+ II. H2C2O4 ---> H+ + HC2O4- III. HOCl + HC2O4- ---> H2O + Cl- + 2CO2 Which of these steps, occurring in a aqueous solution, is an example of a disproportionation reaction? A. I only B. III only C. I and III only D. I, II, and III

Answer: C. Draw out the molecule, and you will see that the longest carbon chain with the substituents at the lowest possible carbon numbers is actually different from the one chosen in the original name. The correct IUPAC name for this molecule is 3-ethyl-4-methylhexane.

Consider the name 2,3-diethylpentane. Based on the structure implied by this name, the correct IUPAC name for this molecule is: A. 2,3-diethylpentane B. 1,2-diethylbutane C. 3-ethyl-4-methylhexane D. 3-methyl-4-ethylhexane

1 & 2: Diastereomers 1 & 3: Diastereomers 1 & 4: Enantiomers 2 & 3: Enantiomers 2 & 4: Diastereomers 3 & 4: Diastereomers

Consider the six pairs that the following four molecules can make. Which pairs are enantiomers? Diastereomers?

Answer: A. The terms in the answer choices refer to the magnetic spin of the two electrons. The quantum number m(s) represents this property as a measure of an electron's intrinsic spin. These electrons' spins are parallel, in that their spins are aligned in the same direction (m(s) = +1/2 for both species).

Consider the two sets of quantum numbers shown in the table, which describe two different electrons in the same atom. Which of the following terms best describes these two electrons? A. Parallel B. Opposite C. Antiparallel D. Paired

1. Balance the reaction: P4 + 6Cl2 ---> 4PCl3 Find moles of phosphorus: MW = (31 x 4) = 124 g/mol 1.45 g P4 * 1 mol P4 * 6 mol Cl2 * 70.9 g Cl2 = 4.963 g Cl2 124 g P4 1 mol P4 1 mol Cl2 2. 4.963 g Cl2 + 1.45 g P4 = 6.413 g PCl3

Considering the following equation: P4 (s) + Cl2 (g) ---> PCl3 (l) If we have 1.45 g of P4 in this reaction: 1. What are the grams of Cl2 required? 2. What are the grams of PCl3 produced?

n=3 1/λ = R * (1/(n(f)^2) - 1/(n(i)^2)) 1/λ = 1.097 x 10^7 1/m * (1/(2^2) - 1/(3^2)) 1/λ = 1523611.111 1/m λ = 6.56 x 10^-7 m = 656 x 10^-9 m = 656 nm It would emit a red light, since the wavelength will be 656 nm. n=4 1/λ = R * (1/(n(f)^2) - 1/(n(i)^2)) 1/λ = 1.097 x 10^7 1/m * (1/(2^2) - 1/(4^2)) 1/λ = 2056875 1/m λ = 4.86 x 10^-7 m = 486 x 10^-9 m = 486 nm It would emit a greenish light, since the wavelength will be 486 nm. 1/λ = R * (1/(n(f)^2) - 1/(n(i)^2)) 1/λ = 1.097 x 10^7 1/m * (1/(2^2) - 1/(5^2)) 1/λ = 2303700 1/m λ = 4.34 x 10^-7 m = 434 x 10^-9 m = 434 nm It would emit a blue light, since the wavelength will be 434 nm. 1/λ = R * (1/(n(f)^2) - 1/(n(i)^2)) 1/λ = 1.097 x 10^7 1/m * (1/(2^2) - 1/(6^2)) 1/λ = 2437777.778 1/m λ = 4.10 x 10^-7 m = 410 x 10^-9 m = 410 nm It would emit a violet light, since the wavelength will be 410 nm.

Considering the line spectrum of hydrogen shown, what color of light would be emitted if an excited electron fell from n=3 to n=2? What about from n=4 to n=2? What about from n=5 to n=2? What about from n=6 to n=2?

Attacking unhybridized p-orbital Lewis acid-base reactions

Coordinate covalent bonding usually involves the lone pair of one atom ______________ another atom that has a ________________________ to form a bond. Coordinate covalent bonds are typically formed in ________________________ reactions.

The change of total internal energy of a system is equal to the amount of heat transferred to the system plus the amount of work exerted on the system. Energy can neither be created nor destroyed, it can just be converted from one form to another.

Define the First Law of Thermodynamics.

It is a form of column chromatography where the stationary phase consists of only one of two conformations, which will allow the binding of only one of the enantiomers. The chirality of the stationary phase will match with the chirality of the desired enantiomer.

Describe chiral column chromatography.

It is a form of gas chromatography where the stationary phase consists of only one of two conformations of the enantiomers, which will allow the binding of only one of the enantiomers. The chirality of the stationary phase will match the chirality of the desired enantiomer.

Describe chiral gas chromatography.

If both substituents are on the same side of the ring (both wedges are the same) then it is a cis-isomer. If both substituents are on opposite sides of the ring (both wedges are different) then it is a trans-isomer. If both substituents are on an equatorial position or axial position, then it is a trans-isomer. If one substituent is on an equatorial position and the other is on an axial position, then it is a cis-isomer.

Describe cis-trans isomers for cyclic rings.

Macroscopic An increase in disorder. Statistical The measure of the spontaneous dispersal of energy at a specific temperature, increasing the number of available microstates for a given molecule.

Describe entropy on a macroscopic level and in statistical terms.

Linear thermal expansion is only applicable to solids. Rising temperatures lead to an increase in length Falling temperatures lead to a decrease in length The change in length is proportional to the original length and the change in temperature.

Describe how linear thermal expansion can change the physical properties of matter based on temperature.

Volumetric thermal expansion is applicable to both solids and liquids. Rising temperatures lead to an increase in volume. Falling temperatures lead to a decrease in volume. The change in volume is proportional to the original volume and the change in temperature.

Describe how volumetric thermal expansion can change the physical properties of matter based on temperature.

When two objects come into thermal contact with one another, thermal energy will spontaneously flow from the hotter object to the cooler object, until thermal equilibrium is achieved. In a closed system, energy will spontaneously and irreversibly go from being localized to being spread out (dispersed). The total change in entropy of a system plus its surroundings will always increase for a spontaneous process.

Describe the Second Law of Thermodynamics.

When a cyclohexane undergoes a chair flip, all axial groups and equatorial groups switch. Equatorials become axial and axials become equatorial. The process is slow if there are any bulky substituents. The bulkiest substituent will favor the equatorial position in order to reduce steric (nonbonded) strain with axial groups.

Describe the chair flip scenario of a cyclohexane in a chair conformation.

Bronsted-Lowry Acid & Base: -Bronsted-Lowry Acid is the proton donor -Bronsted-Lowry Base is the proton acceptor Conjugate Acid & Base: -Conjugate Acid is formed from a Bronsted-Lowry Base accepting a proton -Conjugate Base is formed from a Bronsted-Lowry Acid donating a proton Lewis Acid & Base: -Lewis Acid is an electron pair acceptor -Lewis Base is an electron pair donor Arrhenius Acid & Base: -Acid produces an excess of H+ ions when it dissociates in an aqueous solution -Base produces an excess of OH- ions when it dissociates in an aqueous solution

Describe the differences between the following pairs: -Bronsted-Lowry Acid & Bronsted-Lowry Base -Conjugate Acid & Conjugate Base -Lewis Acid & Lewis Base -Arrhenius Acid & Arrhenius Base

1. Non-polarized light passes through polarized filter 2. Plane-polarized light is produced by non-polarized light passing through the filter 3. Plane-polarized light passes through a polarimeter tube containing a solution of the compound. Plane-polarized light rotates a certain amount of degrees when it hits the compound. 4. Plane-polarized light hits the analyzer, which we rotate to see the angle the light is now.

Describe the process of determining optical activity of a stereoisomer.

It is the process of isolating a solid from a liquid. This is done by pouring a liquid-solid mixture onto a filter paper that allows only the solvent to pass through.

Describe the process of filtration.

1. Sample is injected into injection port 2. Sample enters column and is vaporized into a gas. 3. Vaporized compounds passes through the 30-ft. coiled tube in the column. Compounds travel at different rates due to stationary phase in the tube. 4. Compounds pass through the detector, which records the concentration of all the compounds found in the sample. 5. Detector takes signals and displays them in a way that is analyzed by computer.

Describe the process of gas chromatography.

It is a separation method. It involves dissolving our product in a minimum amount of hot solvent and letting it recrystallize as it cools. The solvent should be one in which the product is only soluble at high temperatures. Thus, when the solution cools, only the desired product will recrystallize out of the solution, excluding impurities.

Describe the process of recrystallization.

1. Assign the four substituents attached to the chiral carbon a priority number based on their atomic number (The highest priority goes to the group with the highest atomic number, while the lowest priority goes to the group with the lowest atomic number). 2. If any substituents have the same atomic number, continue down the chain that is connected to the substituents until you find a difference in atomic number. 3. Make sure that the lowest priority group is pointing away from you (is on the dashed wedge). If the lowest priority group is not pointing away from you, we must rotate the molecule to get the that substituent pointing away from you. 4. Draw a curved arrow from the highest priority substituent to the second highest priority substituent, then draw a curved arrow from the second highest priority substituent to the third highest priority substituent. If the arrow turns to the right, it is labeled R; if the arrow turns to the left, it is labeled S.

Describe the steps for assigning R,S configuration to chiral centers.

Answer: D. Bond lengths decreases as the bond order increases, and they also decrease with larger differences in electronegativity. In this case, because both C2H2 and HCN have triple bonds, we cannot compare the bond lengths based on bond order. We must then rely on other periodic trends. The bond length decreases when moving to the right along the periodic table's rows because more electronegative atoms have shorter radii. The nitrogen in HCN is likely to hold its electrons closer, or in a shorter radius, than the carbons in C2H2.

Despite the fact that both C2H2 and HCN contain triple bonds, the lengths of these triple bonds are not equal. Which of the following is the best explanation for this finding? A. In C2H2, the bond is shorter because it is between atoms of the same element B. The two molecules have different resonance structures C. Carbon is more electronegative than hydrogen D. Nitrogen is more electronegative than carbon

-P in PO4(3-): s: 2 p: 6 d: 2 f: 0 total: 10 -O in PO4(3-) s: 2 p: 6 d: 0 f: 0 total: 8 -Ir: s: 2 p: 0 d: 7 f: 0 total: 9 -Cf: s: 2 p: 0 d: 0 f: 10 total: 12

Determine how many valence electrons come from each subshell in the following atoms: -P in PO4(3-) -O in PO4(3-) -Ir -Cf

Cis

Determine if the following molecule is cis or trans.

Trans

Determine if the following molecule is cis or trans.

I. Not meso II. Meso III. Meso IV. Not meso

Determine if the following molecules are meso or not meso.

from left to right: Chiral Achiral Chiral Achiral

Determine the chirality of the molecules shown.

+----------------->

Determine the dipole moment of the following compound.

+----------->

Determine the dipole moment of the following compound.

<----------------+

Determine the dipole moment of the following compound.

ΔT(f) = i * K(f) * m i = 2 since NaCl dissociates into 2 particles m = moles solute/kg solvent 1800 g water = 1.8 kg water 58.5 g NaCl/58.5 g/mol NaCl = 1 mol NaCl 1 mol NaCl/1.8 kg of water = 0.56 mol/kg ΔT(f) = 2 * 1.86 K*kg/mol * 0.56 mol/kg = 2.08 K

Determine the freezing point depression of a solution containing 58.5 g of NaCl in 1800 g of water at room temperature. (K(f) = 1.86 K*kg/mol).

+ I I I I V

Determine the net dipole moment of the following molecule.

H3PO4: 0.25 * 3 = 0.75 N PO4(3-) Convert grams to molarity: 95 g * 1 mol * 1 mol = 10 M * 3(e-) = 30 N 95 g 0.1 L

Determine the normality of the following solution: 0.25 M H3PO4 95 g PO4(3-) in 100 mL solution

-Fe(2+): Paramagnetic -Ag+: Diamagnetic -Hg: Diamagnetic -B: Paramagnetic -Na+: Diamagnetic -Xe: Diamagnetic -Cu+: Diamagnetic F-: Diamagnetic Y: Paramagnetic Mg: Diamagnetic O(2-): Diamagnetic As: Paramagnetic

Determine whether or not the following are diamagnetic or paramagnetic: -Fe(2+) -Ag+ -Hg -B -Na+ -Xe -Cu+ -F- -Y -Mg -O(2-) -As

A. Meso, has a plane of symmetry B. Not meso, has no plane of symmetry C. Meso, can rotate on single bond between carbons, which gives the molecule a plane of symmetry. D. Meso, can rotate on single bond between carbons, which gives the molecule a plane of symmetry E. Meso, has a plane of symmetry F. Not meso, can rotate on single bond between chiral carbons, but rotation will not yield a plane of symmetry here.

Determine whether the following molecules are meso compounds or not meso compounds.

Debye units (coulomb-meters) dipole moment magnitude of total charge distance between charge

Dipole moment is measured in _____________ units. Dipole is calculated by p = qd p = _______________ q = _______________ d = _______________

Decreases the stability

Does an increase in angle strain, torsional strain, or nonbonded strain increase or decrease the stability of a cycloalkane?

No, an isobaric process will still be represented by the equation: ΔU = Q + W

Does the equation for the first law of thermodynamics change if the process is isobaric? If yes, what changes?

Yes, the equation does change if the process is isochoric. An isochoric process is defined as a process in which the volume of the system does not change. This would mean that: ΔV = 0 If ΔV is zero, then regardless of what the value of the pressure is, the value of the work will equal zero. W = 0 Since work will equal zero in an isochoric process, the value of the change in internal energy will equal the value of heat added/removed to/from the system. ΔU = Q

Does the equation for the first law of thermodynamics change if the process is isochoric? If yes, what changes?

Yes, the equation does change if the process is isothermal. An isothermal process is defined as a process in which the temperature in the system does not change. Which means: ΔT = 0 Since temperature is proportional to internal energy then that means that the change in internal energy will not change either. In other words: ΔU = 0 This means that heat and work must be equal in magnitude but in opposite directions (one value is positive while the other value is negative). So this would mean that: I Q I = I W I

Does the equation for the first law of thermodynamics change if the process is isothermal? If yes, what changes?

Yes, the equation does change if the process is adiabatic. An adiabatic process is defined as a process in which there is no net flow of heat between the system and the surroundings. Therefore: Q = 0 Since the value of heat is equal to zero, the change in internal energy of the system will be determined by the work done on/by the system. ΔU = W

Does the equation for the first law of thermodynamics change if the process was adiabatic? If yes, what changes?

No, temperature is constant when an object undergoes a phase change.

Does the temperature of an object change when it undergoes a phase change?

.............H2 .............C........OH .........../...\...../ H3C.........C ...................H2 ...........OH ............I ...........C ........./...\ H3C.....CH3 ...........H2 ...........C........CH3 ........./....\..../ H3C........O

Draw all the structural isomers of C3H8O.

...........H2.......H2 ...........C.........C ......../...\...../.....\ H3C......C..........CH3 ................H2 ...............CH3 ................I ...............CH.......CH3 ............./....\...../ ....H3C........C .....................H2

Draw all the structural isomers of C5H12

O(-) I I C // \ // \ O O(-) O(-) I I C / \\ / \\ O(-) O O II II C / \ / \ O(-) O(-)

Draw the Lewis dot structures for the carbonate ion (CO3(2-)) and its two other resonance structures.

greater greater

Due to the removal of the first electron, the attractive force on the valence electron will be much _____________ than previously, causing the valence to be drawn closer to the nucleus. By getting closer to the nucleus, the attractive force exerted on the valence electron will be ______________ than before first ionization. This results in a massive increase in ionization energy for removing another electron.

Answer: D. When assigning oxidation numbers, one starts with elements of known oxidation state first, and determines the oxidation state of the other elements by deduction. As a noble gas, argon, (A.), will always have an oxidation state of 0. As a Group VIIA element, fluorine, (B.), will have an oxidation state of 0 (by itself) or -1 (in a compound). As a Group IIA element, strontium, (C.), will likely have an oxidation state of 0 (by itself) or +2 (in a compound). Like most transition metals, iridium, (D.), can have various oxidation states, ranging from -3 to +8. Therefore, one would have to determine the oxidation states of other atoms in an iridium-containing compound to determine iridium's oxidation number.

During the assigning of oxidation numbers, which of the following elements would most likely be determined last? A. Ar B. F C. Sr D. Ir

High temperatures Low temperatures

Endothermic dissolutions are favored at ____________ temperatures Exothermic dissolutions are favored at ______________ temperatures.

Na = 23 * 2 = 46 C = 12 O = 16 * 3 = 48 48 + 46 + 12 = 106 Na: 46 g/mol/106 g/mol * 100% = 43.4% C: 12 g/mol/106 g/mol * 100% = 11.3% O: 48 g/mol/106 g/mol * 100% = 45.3%

Find the percent composition of sodium, carbon, and oxygen in sodium carbonate (Na2CO3).

The aldehyde

For a molecule with a double bond, an aldehyde, and an alcohol, which functional group would determine the suffix when naming?

Since no base is added, we just have the acid. Since HCl is a strong acid, it dissociates 100%: HCl + H2O -----> H3O+ + Cl- [HCl] = [H3O+] = 0.5 M pH = -log[H3O+] = -log[0.5] = 0.301 pH = 0.301

For a titration curve for the titration of 20.00 mL of 0.500 M HCl with 0.500 M NaOH: What is the pH after the addition of 0.0 mL NaOH?

[HCl] = mol/L L = 20 mL = 0.02 L mol = 0.02 * 0.5 = 0.1 mol HCl [NaOH] = mol/L L = 10 mL = 0.01 L 0.5 = mol/0.01 mol = 0.01 * 0.5 = 0.005 mol NaOH HCl + NaOH -----> H2O + NaCl 0.01 0.005 -0.005 -0.005 0.005 0 [HCl] = [H3O+] L = 20 mL + 10 mL = 30 mL = 0.03 L [H3O+] = 0.005/0.03 = 0.17 M pH = -log[0.17] = 0.77 pH = 0.77

For a titration curve for the titration of 20.00 mL of 0.500 M HCl with 0.500 M NaOH: What is the pH after the addition of 10.00 mL NaOH?

[NaOH] = mol/L L = 20 mL = 0.02 L mol = 0.02 * 0.5 = 0.01 mol NaOH [HCl] = mol/L L = 20 mL = 0.02 L mol = 0.02 * 0.5 = 0.010 mol HCl HCl + NaOH -----> H2O + NaCl 0.01 0.01 -0.01 -0.01 0 0 Since both strong acid and strong base are neutralized, the pH should equal the pH of water. pH = 7

For a titration curve for the titration of 20.00 mL of 0.500 M HCl with 0.500 M NaOH: What is the pH after the addition of 20.00 mL NaOH?

[HCl] = mol/L L = 20 mL = 0.02 mL mol = 0.02 * 0.5 = 0.01 [NaOH] = mol/L L = 20.2 mL = 0.0202 L mol = 0.5 * 0.0202 = 0.0101 0.0101 NaOH -0.01 HCl 1 x 10^-4 NaOH [NaOH] = [OH-] L = 20 mL + 20.2 mL = 40.2 mL = 0.0402 L [OH-] = 1 x 10^-4/0.0402 = 0.0025 M pOH = -log[0.0025] = 2.6 pH = 14 - 2.6 = 11.4 pH = 11.4

For a titration curve for the titration of 20.00 mL of 0.500 M HCl with 0.500 M NaOH: What is the pH after the addition of 20.20 mL NaOH?

[HCl] = mol/L L = 20 mL = 0.02 L mol = 0.02 * 0.1 = 0.002 mol HCl [NH3] = mol/L L = 40 mL = 0.04 L mol = 0.1 * 0.04 = 0.004 mol NH3 NH3 + HCl -------> NH4+ + Cl- 0.004 0.002 0 -0.002 -0.002 +0.002 0.002 0 0.002 L = 20 mL + 40 mL = 60 mL = 0.06 L [NH3] = 0.002/0.06 = 0.033 M [NH4+] = 0.002/0.06 = 0.033 M Henderson Hasselbach pOH = pKb + log([NH3]/[NH4+]) pKb = -log[Kb] pKb = -log[1.8 x 10^-5] = 4.74 pOH = 4.74 + log[0.033/0.033] pOH = 4.74 pH = 14 - 4.74 = 9.26 pH = 9.26

For a titration curve for the titration of 40.0 mL of 0.100 M NH3 with 0.100 M HCl: What is the pH after the addition of 20.0 mL of 0.100 M HCl? (Kb(NH3) = 1.8 x 10^-5))

[HCl] = mol/L L = 40 mL = 0.04 L mol = 0.1 * 0.04 = 0.004 mol HCl [NH3] = mol/L L = 40 mL = 0.04 L mol = 0.1 * 0.04 = 0.004 mol NH3 NH3 + HCl --------> NH4+ + Cl- 0.004 0.004 0 -0.004 -0.004 +0.004 0 0 0.004 L = 40 mL + 40 mL = 80 mL = 0.08 L [NH4+] = 0.004/0.08 = 0.05 NH4+ + H2O <-------> NH3 + H3O+ 0.05 0 0 -x +x +x 0.05 - x x x Ka = x^2/0.05-x x<<1 Ka * Kb = Kw Ka = 1 x 10^-14/1.8 x 10^-5 Ka = 5.6 x 10^-10 x^2 = 0.05 * 5.6 x 10^-10 x = sqrt(2.8 x 10^-11) x = 5.3 x 10^-6 x = [H3O+] pH = -log[5.3 x 10^-6] = 5.28 pH = 5.28

For a titration curve for the titration of 40.0 mL of 0.100 M NH3 with 0.100 M HCl: What is the pH after the addition of 40.0 mL of 0.100 M HCl? (Kb(NH3) = 1.8 x 10^-5)

[HCl] = mol/L L = 60 mL = 0.06 L mol = 0.06 * 0.1 = 0.006 mol HCl [NH3] = mol/L L = 40 mL = 0.04 L mol = 0.04 * 0.1 = 0.004 mol NH3 NH3 + HCl ---------> NH4+ + Cl- 0.004 0.006 -0.004 -0.004 0 0.002 HCl dissociates 100% [HCl] = [H3O+] L = 40 mL + 60 mL = 100 mL = 0.1 L [H3O+] = 0.002/0.1 = 0.02 M pH = -log[0.02] = 1.7 pH = 1.70

For a titration curve for the titration of 40.0 mL of 0.100 M NH3 with 0.100 M HCl: What is the pH after the addition of 60.0 mL of 0.100 M HCl? (Kb(NH3) = 1.8 x 10^-5))

NH3 + H2O <-----> NH4+ + OH- 0.1 0 0 -x +x +x 0.1 - x x x Kb = x^2/0.1-x x<<1 (1.8 x 10^-5) * 0.1 = x^2 x = sqrt(1.8 x 10^-6) = 0.0013 pOH = -log[0.0013] = 2.89 pH = 14 - 2.89 = 11.11 pH = 11.11

For a titration curve for the titration of 40.0 mL of 0.100 M NH3 with 0.100 M HCl: What is the pH before any acid is added? (Kb(NH3) = 1.8 x 10^-5)

CH3COOH + H2O <---> CH3COO- + H3O+ 0.2 M 0 0 -x +x +x 0.2 - x x x 1.8 x 10^-5 = x^2/0.2-x x<<1 (1.8 x 10^-5) * 0.2 = x^2 x^2 = 3.6 x 10^-6 x = sqrt(3.6 x 10^-6) = 1.9 x 10^-3 x = [H3O+] pH = -log[1.9 x 10^-3] = 2.72 pH = 2.72

For a titration curve for the titration of 50.0 mL of 0.200 M CH3COOH with 0.0500 M NaOH: What is the pH after the addition of 0.0 mL of NaOH? (Ka(CH3COOH) = 1.8 x 10^-5)

[NaOH] = mol/L L = 100 mL = 0.1 L mol = 0.05 * 0.1 = 0.005 mol NaOH [CH3COOH] = mol/L L = 50 mL = 0.05 L mol = 0.2 * 0.05 = 0.01 mol CH3COOH CH3COOH + NaOH <-----> H2O + CH3COO- Na+ 0.01 0.005 0 -0.005 -0.005 +0.005 0.005 0 0.005 L = 100 mL + 50 mL = 150 mL = 0.15 L [CH3COOH] = 0.005/0.15 = 0.033 M [CH3COO-] = 0.005/0.15 = 0.033 M Henderson Hasselbach pH = pKa + log[(CH3COO-)/(CH3COOH)] pKa = -log[Ka] = -log[1.8 x 10^-5] = 4.74 pH = 4.74 + log[0.033/0.033] pH = 4.74

For a titration curve for the titration of 50.0 mL of 0.200 M CH3COOH with 0.0500 M NaOH: What is the pH after the addition of 100 mL of NaOH? (Ka(CH3COOH) = 1.8 x 10^-5)

[NaOH] = mol/L L = 200 mL = 0.2 L mol = 0.05 * 0.2 = 0.01 mol NaOH [CH3COOH] = mol/L L = 50 mL = 0.05 L mol = 0.05 * 0.2 = 0.01 mol CH3COOH CH3COOH + NaOH -------> H2O + CH3COO- Na+ 0.01 0.01 0 -0.01 -0.01 +0.01 0 0 0.01 L = 50 mL + 200 mL = 250 mL = 0.25 L [CH3COO-] = 0.01/0.25 = 0.04 M CH3COO- + H2O <-------> CH3COOH + OH- 0.04 0 0 -x +x +x 0.04 - x x x Kb = x^2/0.04-x x<<1 Ka * Kb = Kw Kb = (1 x 10^-14)/(1.8 x 10^-5) = 5.6 x 10^-10 5.6 x 10^-10 = x^2/0.04 x^2 = (5.6 x 10^-10) * 0.04 x = sqrt(2.24 x 10^-11) = 4.7 x 10^-6 x = [OH-] pOH = -log[4.7x10^-6] = 5.33 pH = 14 - 5.33 = 8.67 pH = 8.67

For a titration curve for the titration of 50.0 mL of 0.200 M CH3COOH with 0.0500 M NaOH: What is the pH after the addition of 200.0 mL of NaOH? (Ka(CH3COOH) = 1.8 x 10^-5)

[NaOH] = mol/L L = 300 mL = 0.3 L mol = 0.3 * 0.05 = 0.015 mol NaOH [CH3COOH] = mol/L L = 50 mL = 0.05 L mol = 0.05 * 0.2 = 0.01 mol CH3COOH CH3COOH + NaOH <-------> H2O + CH3COO- Na+ 0.01 0.015 -0.01 -0.01 0 0.005 L = 300 mL + 50 mL = 350 mL = 0.35 L [NaOH] = [OH-] [OH-] = 0.005/0.35 = 0.014 M pOH = -log[0.014] = 1.85 pH = 14 - 1.85 = 12.15 pH = 12.15

For a titration curve for the titration of 50.0 mL of 0.200 M CH3COOH with 0.0500 M NaOH: What is the pH after the addition of 300.0 mL of NaOH?

s subshell p subshell d subshell f subshell

For azimuthal quantum numbers: - When l = 0, it is indicating the ______________ subshell. - When l = 1, it is indicating the _______________ subshell. - When l = 2, it is indicating the ______________ subshell. - When l = 3, it is indicating the _______________ subshell.

Bonding orbital Antibonding orbital Bonding orbital Antibonding orbital

For determining molecular orbitals, if the signs of the wave functions are the same (positive and positive or negative and negative) then a ______________ orbital is formed. If the signs of the wave functions are different (positive and negative) then a ________________ orbital is formed. ______________ orbitals are lower in energy, and thus more stable. while __________________ orbitals are higher in energy, thus less stable.

-Invert -Retain -Invert -Retain

For each of the Fischer projection manipulations listed below, is stereochemisty (R,S configuration) retained or inverted? -Switching a pair of substituents -Switching two pairs of substituents -Rotating a molecule 90° -Rotating a molecule 180°

1. Group 1 and 2 2. Group 6 and 16 3. Groups 1-15 4. All groups (most notably Groups 3 - 12) 5. Almost all groups (most notably Groups 14 - 17) 6. Group 18

For each of the properties listed below, write down the groups of the periodic table that exhibit those properties. 1. High reactivity to water 2. Six valence electrons 3. Contain at least one metal 4. Multiple oxidation states 5. Negative oxidation states 6. Possess a full octet in the neutral state

Oxygen Nitrogen Fluorine Oxygen Nitrogen Fluorine

For hydrogen bonding interactions to exist, the hydrogen must be bonded to either a ______________, ______________, or ______________ atom, and must interact with a partially negative ___________________, _________________, or ________________ atom of another molecule.

1. # orbitals = n^2 1^2 = 1 #orbitals = 1 l = 0 -> n-1 n-1 = 0 l = 0 m(l) = possible integers from -l to +l m(l) = 0 l = 0 ---> s-orbital #possible orbitals = 1 s-orbital 2. Only one s-orbital s-orbital can only fit 2 electrons in the subshell Total number of electrons in s-subshell = 2 3. #electrons in n=1 = (2n^2) (2(1^2)) = 2 #electrons in n=1 = 2

For n=1, find, 1. The number of possible orbitals 2. The number of electrons in each subshell 3. The total number of electrons in the shell

1. # orbitals = n^2 2^2 = 4 #orbitals = 4 l = 0 -> n-1 2-1 = 1 l = 0 or 1 m(l) = possible integers from -l to +l m(l) = 0 if l=0 m(l) = -1, 0, or +1 if l=1 l = 0 ---> s-orbital l = 1 ---> p-orbital One possible orbital in s-subshell (m(l) = 0) Three possible orbitals in p-subshell (m(l) = -1, 0, +1) 2. One s-orbital s-orbital can only fit 2 electrons in the subshell Three p-orbitals 2 electrons fit into each p-orbital Total number of electrons in s-subshell = 2 Total number of electrons in p-subshell = 6 3. #electrons in n=2 = (2n^2) (2(2^2)) = 8 #electrons in n=2 = 8

For n=2, find, 1. The number of possible orbitals 2. The number of electrons in each subshell 3. The total number of electrons in the shell

1. # orbitals = n^2 3^2 = 9 #orbitals = 9 l = 0 -> n-1 3-1 = 2 l = 0, 1, 2 m(l) = possible integers from -l to +l m(l) = 0 if l=0 m(l) = -1, 0, or +1 if l=1 m(l) = -2, -1, 0, 1, 2 if l=2 l = 0 ---> s-orbital l = 1 ---> p-orbital l = 2 ---> d-orbital One possible orbital in s-subshell (m(l) = 0) Three possible orbitals in p-subshell (m(l) = -1, 0, +1) Five possible orbitals in d-subshell (m(l) = -2, -1, 0, 1, 2) 2. One s-orbital s-orbital can only fit 2 electrons in the subshell Three p-orbitals 2 electrons fit into each p-orbital Five d-orbitals 2 electrons fit into each d-orbital Total number of electrons in s-subshell = 2 Total number of electrons in p-subshell = 6 Total number of electrons in d-subshell = 10 3. #electrons in n=3 = (2n^2) (2(3^2)) = 18 #electrons in n=3 = 18

For n=3, find, 1. The number of possible orbitals 2. The number of electrons in each subshell 3. The total number of electrons in the shell

1. # orbitals = n^2 4^2 = 16 #orbitals = 16 l = 0 -> n-1 4-1 = 3 l = 0, 1, 2, 3 m(l) = possible integers from -l to +l m(l) = 0 if l=0 m(l) = -1, 0, or +1 if l=1 m(l) = -2, -1, 0, 1, 2 if l=2 m(l) = -3, -2, -1, 0, 1, 2, 3 if l=3 l = 0 ---> s-orbital l = 1 ---> p-orbital l = 2 ---> d-orbital l = 3 ---> f-orbital One possible orbital in s-subshell (m(l) = 0) Three possible orbitals in p-subshell (m(l) = -1, 0, +1) Five possible orbitals in d-subshell (m(l) = -2, -1, 0, 1, 2) Seven possible orbitals in f-subshell (m(l) = -3, -2, -1, 0, 1, 2, 3) 2. One s-orbital s-orbital can only fit 2 electrons in the subshell Three p-orbitals 2 electrons fit into each p-orbital Five d-orbitals 2 electrons fit into each d-orbital Seven f-orbitals 2 electrons fit into each f-orbital Total number of electrons in s-subshell = 2 Total number of electrons in p-subshell = 6 Total number of electrons in d-subshell = 10 Total number of electrons in f-subshell = 14 3. #electrons in n=4 = (2n^2) (2(4^2)) = 32 #electrons in n=4 = 32

For n=4, find, 1. The number of possible orbitals 2. The number of electrons in each subshell 3. The total number of electrons in the shell

Step 1: Balance the reaction Reaction is already balanced Step 2: Find theoretical yield 35.15 g C6H6 * 1 mol C6H6 = 0.451 mol C6H6 78.114 g C6H6 0.451 mol C6H6 * 1 mol C6H5NO2 = 0.451 mol C6H5NO2 1 mol C6H6 0.451 mol C6H5NO2 * 123 g C6H5NO2 = 55.473 g C6H5NO2 1 mol C6H5NO2 Step 3: Find %yield %yield = actual yield * 100% theoretical yield %yield = 52.145 g C6H5NO2 * 100% = 94% 55.473 g C6H5NO2

For the following chemical reaction: C6H6 + HNO3 ---> C6H5NO2 + H2O we start out with 35.15 g of benzene and an excess of nitric acid. Calculate the %yield if our preparation yielded 52.145 g of nitrobenzene.

1. Nonpolar covalent compounds 2. Polar covalent compounds 3. Covalent compounds with hydrogen bonding 4. Ionic compounds

For the following compounds, order them in order from lowest melting/boiling point to highest melting/boiling point: -Nonpolar covalent compounds -Ionic compounds -Polar covalent compounds -Covalent compounds with hydrogen bonding

Step 1: Balance the reaction 2As + 6NaOH ---> 2Na3AsO3 + 3H2 Step 2: Find mL NaOH Remember that 1 mole of an ideal gas at standard temperature and pressure (STP) has a volume of 22.4 L. 4.5 L H2 * 1 mol H2 = 0.2 mol H2 22.4 L H2 0.2 mol H2 * 6 mol NaOH = 0.4 mol NaOH 3 mol H2 Recall that M = mol/L Therefore mol (NaOH) = 4 M * 1 L = 4 mol 0.4 mol NaOH * 1 L NaOH = 0.1 L 4 mol NaOH 0.1 L = 100 mL NaOH

For the following equation: As (s) + NaOH (aq) ---> Na3AsO3 (aq) + H2 (g) Calculate how many mL of a 4 molar (M) solution of sodium hydroxide are required to produce 4.5 L of hydrogen gas.

+1 -1 0 0 +1 -1 2KI + H2 -------> 2K + 2HI K goes from a +1 oxidation state to a 0 oxidation state, meaning that it underwent reduction. This means that K is the oxidizing agent. H goes from a 0 oxidation state to a +1 oxidation state, meaning that it underwent oxidation. This means that H is the reducing agent.

For the following reaction, identify the oxidation states of each atom, the oxidizing agent, and the reducing agent: 2KI + H2 -------> 2K + 2HI

0 +3 -3 0 +3 -3 Al + BPO4 -------> B + AlPO4 B goes from a +3 oxidation state to a 0 oxidation state, meaning that it underwent reduction, making B the oxidizing agent. Al goes from a 0 oxidation state to a +3 oxidation state, meaning that it underwent oxidation, making Al the reducing agent.

For the following reaction, identify the oxidation states of each atom, the oxidizing agent, and the reducing agent: Al + BPO4 -------> B + AlPO4

Answer: A. This is an example of reverse-phase chromatography. The solvent system is polar, which means that the most polar compound will travel the furthest up the card, resulting in the largest Rf. This gives compound III the largest Rf, which corresponds to spot A.

Four compounds, I, II, III, and IV, are separated by chromatographic techniques. Compound III is the most polar, II the least polar, and I and IV have intermediate polarity. The solvent system is 85:15 ethanol : methylene chloride. Which spot on the card likely belongs to compound III? A. A B. B C. C D. D

Answer: B. Fractional distillation is the most effective procedure for separating two liquids that boil within a few degrees of each other. Ethyl acetate and ethanol boil well within 25°C of each other and thus would be good candidates for fractional distillation. Fractional distillation could also be used for the liquids in (C), but would require lower pressures because of their high boiling points.

Fractional distillation under atmospheric pressure would most likely be used to separate which of the following compounds? A. Methylene chloride (bp of 40°C) and water (bp of 100°C) B. Ethyl acetate (bp of 77°C) and ethanol (bp of 80°C) C. Aniline (bp of 184°C) and benzyl alcohol (bp of 205°C) D. Aniline (bp of 184°C) and water (bp of 100°C)

IUPAC: 3-butene-2-one Common: methylvinylketone

Give both the common and IUPAC name for the following molecule.

IUPAC: Butanal Common: Butyraldehyde

Give both the common and IUPAC name for the following molecule.

IUPAC: Ethanal Common: Acetaldehyde

Give both the common and IUPAC name for the following molecule.

IUPAC: Ethanoic acid Common: Acetic acid

Give both the common and IUPAC name for the following molecule.

IUPAC: Ethanoic anhydride Common: Acetic anhydride

Give both the common and IUPAC name for the following molecule.

IUPAC: Ethanol Common: Ethyl alcohol

Give both the common and IUPAC name for the following molecule.

IUPAC: Isobutanol Common: Isobutyl alcohol

Give both the common and IUPAC name for the following molecule.

IUPAC: Isopropanol Common: Isopropyl alcohol

Give both the common and IUPAC name for the following molecule.

IUPAC: Methanal Common: Formaldehyde

Give both the common and IUPAC name for the following molecule.

IUPAC: Methanoic acid Common: Formic acid

Give both the common and IUPAC name for the following molecule.

IUPAC: Methyl ethanoate Common: Methyl acetate

Give both the common and IUPAC name for the following molecule.

IUPAC: Methyl methanoate Common: Methyl formate

Give both the common and IUPAC name for the following molecule.

IUPAC: Propanal Common: Propionaldehyde

Give both the common and IUPAC name for the following molecule.

IUPAC: Tert-butanol Common: Tert-butyl alcohol

Give both the common and IUPAC name for the following molecule.

1. Ethanol 2. 5-methyl-2-heptanol 3. hept-6-en-1-ol

Give the IUPAC name for the following molecules.

Answer: D. Because this solution is composed of a much larger molecule and a much smaller molecule, size-exclusion chromatography would effectively remove the smaller insulin molecule into the fraction retained in the column and allow the titin to be eluted. Affinity chromatography (C) could also be used, but comes with the risk of rendering titin unusable; the eluent run through an affinity chromatography column often binds to the target molecule.

Given a solution of insulin (mw = 5.8 kD) and titin (mw = 3816 kD), which chromatographic technique would be the most effective for separating out usable molecules of titin? A. Thin-layer chromatography B. Ion-exchange chromatography C. Affinity chromatography D. Size-exclusion chromatography

Answer: C. The total work done by the cycle is the sum of the work of paths A, B, and C, or the area within the cycle. Because the area bounded by A, B, and C is a triangle, with a base of 5 m^3 and a height of 3 Pa, we can calculate the area as 1/2(5 m^3)(3 Pa) = 7.5 J. Clockwise loops tend to does positive work by the gas, while counterclockwise loops do negative work.

Given the cycle shown, what is the total work done by the gas during the cycle? A. -10 J B. 0 J C. 7.5 J D. 17.5 J

Group 1 Group 2 Group 16 Group 17 Group 18

Group ____ on the periodic table consists of alkali metals. Group ____ on the periodic table consists of alkaline earth metals. Group ____ on the periodic table consists of chalcogens. Group ____ on the periodic table consists of halogens. Group ____ on the periodic table consists of noble gases.

superheating

Heating to above a phase transition temperature without the occurrence of the transformation.

Negligible

Helium, neon, and argon are the three elements that have _________________ electronegativities.

3 same extra opposite repelled electron shielding decrease

How Elements in Group 16 defy ionization trend: Group 15 elements have ____ electrons in the p-subshell, with each of them in their own orbital spinning in the ___________ direction. Group 16 elements has a ________ electron added to one of those orbitals, but spinning in the _______________ direction. This causes the electron being added to be ______________ by the electron that was already present in that orbital, causing more _______________ ____________, causing a ______________ in effective nuclear charge, resulting in the decrease in ionization energy.

p-orbital higher p-orbital farther atomic radius p-orbital repelled electron shielding lower

How Group 13 elements defy ionization trend: Group 13 elements has an electron that is added to the ____-orbital, which is ____________ in energy than the s-orbital. Electrons in the _____-orbital are _____________ from the nucleus than the s-orbital, which increases ___________ ______________, which decreases ionization energy. Furthermore, the placement of of the electrons in the ____-orbital will be _______________ by the electrons in the s-orbital along with the other electrons in the inner shells. This leads to an increase in _____________ _____________, which leads to a ____________ effective nuclear charge, which decreases ionization energy

Same Repelled Increase No

How Group 15 elements defy electron affinity trend: Group 15's p-orbitals all have an electron in each of them spinning in the ___________ direction. The addition of an extra electron into one of these orbitals will cause it to be _________________ by the electron that is already present. This leads to a ________________ in electron shielding, which leads to _______ attraction between the valence electron and nucleus. This results in nitrogen not having any electron affinity.

Higher 2p6 3s1 Shielded 0 No

How Group 18 elements defy electron affinity trend: When we add an electron to a Group 18 element, it will cause a valence of a ____________ energy level to be formed. In the case of neon, we go from a _______ subshell to a _____ subshell. This addition will cause the newly added electron to be ___________ by all the inner electrons. With all of this considered, Group 18 elements will have an effective nuclear charge of ___, meaning that the added electron will have ____ attraction to the nucleus, meaning it does not have an affinity for an electron.

p-subshell shielded Electron shielding Energy

How Group 2 elements defy electron affinity trend: When adding an electron to a Group 2 element, the electron will be added into the ________-subshell instead of the s-subshell. Because of the new subshell it is placed in, it will be _______________ by both the 1s electrons, but also the 2s electrons. The increase in ______________ ____________ will cause the added electron to not exhibit attraction to the nucleus, meaning it would take ___________ to add the electron. This results in it not having an electron affinity

Find the priority of the substituents on the double-bond or ring, and see whether the highest priority substituent is on the same side or opposite sides.

How are E,Z isomers determined?

If amino group is on the right of Fischer projection, it has a D-configuration. If amino group is on the left of Fischer projection, it has a L-configuration.

How are amino acids classified as L- or D- enantiomers?

A complex ion is an ion formed when electron pair donors and electron pair acceptors form coordinate covalent bonds.

How are complex ions formed?

Larger substituents will take the equatorial position while the smaller substituents will take the axial position.

How are substituents arranged in a cycloalkane?

If the entropy of an endothermic reaction is greater than the enthalpy, the change in Gibbs Free Energy will be negative, making the reaction spontaneous.

How can an endothermic reaction be spontaneous?

Enantiomer pairs have the same magnitudes of rotation, but the direction of rotation is different.

How do enantiomer pairs compare to each other in regards to rotation of plane-polarized light?

Use the formula: Q = m*c*ΔT Q = heat lost or gained m = mass of the object c = specific heat of the object ΔT = change in temperature of that object (Tf - Ti)

How do we determine the heat gained or lost by an object when the temperature of the object changes?

Use this formula: Q = m * L Q = heat of phase change m = mass of the object L = heat of transformation (fusion/vaporization

How do we determine the heat needed for an object to undergo a phase change

Divide the value of energy in joules by 1.6 x 10^-19 J Multiply the value of energy in electron volts by 1.6 x 10^-19 J

How do you convert joules into electron volts? How do you convert electron volts into joules?

Treat a mixture with a chiral salt, followed by separation of resulting diastereomers.

How do you separate two enantiomers from one another?

The addition of a solute decreases the evaporation rate of a solvent without affecting the condensation rate, thus decreasing the vapor pressure of a solvent.

How does adding a solute to a solvent affect the vapor pressure of a solvent?

A compound that boils at lower temperatures will elute faster. A compound that boils at higher temperatures will elute slower.

How does boiling point affect the elution time in gas chromatography?

An increase in concentration of the enantiomer present in a solution would increase the rotation of plane-polarized light. A decrease in concentration of the enantiomer present in a solution would decrease the rotation of plane-polarized light. If the concentration of the enantiomers present in a solution was doubled, then the rotation of plane-polarized light would be doubled.

How does changing the concentration of an enantiomer affect the rotation of plane-polarized light?

If the length of the polarimeter was doubled, the rotation of plane-polarized light would be doubled as well. If the length of the polarimeter was halved, the rotation of the plane-polarized light would be halved as well.

How does changing the length of the polarimeter tube affect the rotation of plane-polarized light?

A compound with a low molecular weight will elute faster. A compound with a high molecular weight will elute slower.

How does molecular weight affect the elution time in gas chromatography?

A less polar compound will elute faster. A more polar compound will elute slower.

How does polarity affect the elution time in gas chromatography? (Assuming the stationary phase is polar)

A compound with low solubility for the liquid stationary phase will elute faster. A compound with high solubility for the liquid stationary phase will elute slower.

How does solubility affect the elution time in gas chromatography?

The solubility of solids can be increased by increasing the temperature.

How does temperature affect the solubility of solids?

Solubility of gases can be increased by decreasing temperature or increasing the partial pressure of the gas above the solvent.

How does temperature and partial pressure affect the solubility of gases?

A higher column temperature can lead to compounds eluting faster. A lower column temperature can lead to compounds eluting slower.

How does the temperature of the column affect the elution time in gas chromatography?

The work function describes the minimum amount of energy necessary to emit an electron. Any additional energy from a photon will be converted to excess kinetic energy during the photoelectric effect.

How does the work function relate to the energy necessary to emit an electron from a metal?

Compression is characterized by an increase in pressure and a decrease in volume. Expansion is characterized by an increase in volume and a decrease in pressure.

How is compression and expansion characterized in a PV diagram?

ΔL = αLΔT ΔL = change in length of solid α = coefficient of linear expansion L = original length ΔT = change in temperature

How is linear thermal expansion calculated?

It is passed through a polarizing filter.

How is non-polarized light converted to a plane-polarized light?

Rf = distance travelled by substance / distance travelled by solvent

How is retardation/retention factor calculated in thin-layer chromatography?

[α] = α/(c * l) [α] = specific rotation α = observed rotation c = concentration (in g/mL) l = length of polarimeter tube (in decimeters)

How is specific rotation calculated?

ΔS = Q(rev)/T ΔS = change in entropy of a system Q(rev) = heat gained or lost in a reversible process T = temperature in Kelvin

How is the change in entropy of a system calculated?

ΔV = βVΔT ΔV = change in volume β = coefficient of volumetric expansion V = original volume ΔT = change in temperature.

How is volumetric thermal expansion calculated?

W = PΔV W = work P = pressure ΔV = change in volume

How is work defined in a PV diagram for a thermodynamic process?

Answer: B. First balance the atoms in the equation: Cr2O7(2-) + 14H+ ---> 2Cr(2+) + 7H2O Now, adjust the number of electrons to balance the charge. Currently, the left side has a charge of +12 (-2 from dichromate and +14 from protons). The right side has a charge of +4 (+2 from each chromium cation). To decrease the charge on the left side from +12 to +4, we should add 8 electrons: Cr2O7(2-) + 14H+ + 8e- ---> 2Cr(2+) + 7H2O

How many electrons are involved in the following half-reaction after it is balanced? Cr2O7(2-) + H+ + e- ---> Cr(2+) + H2O A. 2 B. 8 C. 12 D. 16

Answer: B. The maximum number of stereoisomers of a compound equals 2^n, where n is the number of chiral carbons in the compound. In this molecule, C-1 (the aldehydic carbon) is not chiral, nor is C-5 (because it is attached to two hydrogen atoms). Therefore, with three chiral centers, there are 2^3 = 8 stereoisomers.

How many stereoisomers exist for the following aldehyde? A. 2 B. 8 C. 9 D. 16

Answer: A. Each single bond has one σ bond, and each double bond has one σ and one π bond. In this question, there are five single bonds (five σ bonds) and one double bond (one σ bond and one π bond), which gives a total of six σ bonds and one π bond. Thus, the correct answer is (A.).

How many σ bonds and π bonds are present in the shown compound? A. Six σ bonds and one π bond B. Six σ bonds and two π bonds C. Five σ bonds and one π bond D. Five σ bonds and two π bonds

Q = m*c*ΔT m = 2 kg = 2000 g c = 4.184 J/g*K Tf = 50°C Ti = 20°C Q = 2000 g * 4.184 J/g*K * (50°C - 20°C) Q = 8368 J/K * 30 K Q = 251040 J

How much heat is needed to heat a 2 kg tank of water from a temperature of 20°C to 50°C? (Specific heat of water is 4.184 J/g*K)

Answer: D. Q = mcΔT m = 500 g = 0.5 kg c = 126 J/kg*K ΔT = 1064°C - 25°C = 1039 K Q = 0.5 kg * 126 J/kg*K * 1039 K Q = 65457 J Q(fus) = m * L(fus) m = 0.5 kg L(fus) = 6.37 x 10^4 J/kg Q(fus) = 0.5 kg * 6.37 x 10^4 J/kg Q(fus) = 31850 J 65457 J + 31850 J = 97307 J = 97 kJ

How much heat is required to completely melt 500 g gold earrings, given that their initial temperature is 25°C? (The melting point of gold is 1064°C, its heat of fusion is 6.37 x 10^4 J/kg, and its specific heat is 126 J/kg*K) A. 15 kJ B. 32 kJ C. 66 kJ D. 97 kJ

%composition by mass = (mass of solute/mass of solution) * 100%

How percent composition by mass is calculated: %composition by mass = (mass of _________/mass of ___________) * 100%

MiVi=MfVf Mi = Initial concentration Mf = Final concentration Vi = Initial volume Vf = Final volume

How the concentration of a solution after dilution is calculated: MiVi = MfVf Mi = ____________ Mf = ____________ Vi = _____________ Vf = _____________

∏ = i * M * R * T ∏ = osmotic pressure i = van't Hoff factor M = Molarity R = Ideal Gas Constant (0.082 L*atm/molK) T = Temperature

How to calculate osmotic pressure: ∏ = i * M * R * T ∏ = ____________________ i = ______________________ M = ___________________________ R = _______________________ T = ____________________

ΔT(b) = i*K(b)*m ΔT(b) = increase in boiling point m = molality K(b) = proportionality constant of solvent i = van't Hoff factor

How to calculate the change in boiling point for a solvent when a solute is added: ΔT(b) = i*K(b)*m ΔT(b) = ____________________ m = ________________ K(b) = __________________________ i = ________________________

ΔT(f) = i * K(f) * m ΔT(f) = decrease in freezing point i = van't Hoff factor K(f) = proportionality constant of solvent m = molality

How to calculate the change in freezing point for a solvent when a solute is added: ΔT(f) = i * K(f) * m ΔT(f) = ___________________________ i = _______________________ K(f) = ________________________________ m = __________________

P = Vapor pressure of solvent with solute added X = Mole fraction of solvent P° = Vapor pressure of solvent without solute

How to calculate the change in vapor pressure of a solvent when a solute is added: P = X * P° P = __________________ X = _________________ P° = __________________

The addition of a complex ion results in a certain ion in a solution to be taken up by the it, essentially resulting in a decrease of that ion in the solution. The solubility of the solute added to the solution will increase with the introduction of the complex ion.

How would the addition of a complex ion affect the solubility of a solute.

-London dispersion forces between the molecules -Hydrogen bonding between the partially positive hydrogen (bonded to an oxygen) and the partially negative oxygen. -Dipole-dipole interactions between the partially positive carbon and the partially negative oxygen.

Identify all the intermolecular forces present in the shown molecule.

Cr: [Ar] 4s(1)3d(5) He: 1s(2) Ag: [Kr] 5s(1)4d(10) Ca: [Ar] 4s(2) Ar: [Ne] 3s(2)3p(6) Si: [Ne] 3s(2)3p(2) Au: [Xe] 6s(1)4f(14)5d(10) C: [He] 2s(2)2p(2) Br: [Ar] 4s(2)3d(10)4p(5) Mo: [Kr] 5s(1)4d(5)

Identify the electron configuration of the following elements: -Cr -He -Ag -Ca -Ar -Si -Au -C -Br -Mo

Cl-: [Ne] 3s(2)3p(6) Fe(2+): [Ar] 3d(6) Cr(3+): [Ar] 3d(3) O-: [He] 2s(2)2p(5) Ba(2+): [Kr] 5s(2)4d(10)5p(6) H-: 1s(2) S(2-): [Ne] 3s(2)3p(6) Pt(2+): [Xe] 4f(14)5d(8) Be(2+): 1s(2) Cu+: [Ar] 3d(10)

Identify the electron configuration of the following: -Cl- -Fe(2+) -Cr(3+) - O- -Ba(2+) -H- -S(2-) -Pt(2+) -Be(2+) -Cu+

Electrolyte Nonelectrolyte Electrolyte Nonelectrolyte

Identify the following as being either an electrolyte or nonelectrolyte: HCl Sucrose MgBr2 CH4

Anion PO4(3-) Anion ClO- Cation NH4+ Anion P3- Anion HCO3- Anion NO2- Cation Cr(2+)

Identify the following ions as cations or anions, and then provide the formula or chemical symbol: Phosphate Hypochlorite Ammonium Phosphide Bicarbonate Nitrite Chromium (II)

1. Neutralization reaction 2. Combination reaction 3. Combustion reaction 4. Double displacement/metathesis reaction 5. Single displacement reaction 6. Decomposition reaction

Identify the following reactions and determine what type of reaction they are: 1. HCl (aq) + NaOH (aq) ---> NaCl (aq) + H2O (l) 2. 2Hg (g) + O2 (g) ---> 2H2O (g) 3. CH4 + 2O2 ---> CO2 + 2H2O 4. CaCl2 (aq) + 2AgNO3 (aq) ---> Ca(NO3)2 (aq) + 2AgCl (s) 5. Cu (s) + AgNO3 (aq) ---> Ag (s) + CuNO3 (aq) 6. 2HgO (s) ---> 2Hg (l) + O2 (g)

From left to right: 1. sp3 tetrahedral 109.5° 2. sp3 tetrahedral 109.5° 3. sp3 bent/angular 104° 4. sp3 tetrahedral 109.5° 5. sp3 tetrahedral 109.5°

Identify the hybridization states, predict the molecular geometries, and predict the bond angle for all atoms (except for any that have only one bond).

1. Split the reaction Zn -------> Zn(2+) Cu(2+) -------> Cu 2. Balance charges with electrons Zn -------> Zn(2+) + 2e- oxidation 2e- + Cu(2+) -------> Cu reduction

Identify the oxidation and reduction half-reactions in the following redox reaction: Zn + Cu(2+) -------> Zn(2+) + Cu

HCl: acid NaOH: base Fe(OH)3: base H2SO4: acid Ca(OH)2: base HNO3: acid

Identify whether or not the following are an Arrhenius acid or an Arrhenius base: -HCl -NaOH -Fe(OH)3 -H2SO4 -Ca(OH)2 -HNO3

Gas chromatography (GC)

Identify which type of chromatography is being performed here.

High performance liquid chromatography (HPLC)

Identify which type of chromatography is being performed here.

Simple column chromatography

Identify which type of chromatography is being performed here.

Thin layer chromatography (TLC)

Identify which type of chromatography is being performed here.

Find concentration of NaOH 0.005 mol NaOH/0.50 L buffer = 0.01 M NaOH Since NaOH is a strong base, it is the same concentration as OH NaOH will react with NH4+ (strong bases will react with acids) A- = NH3 HA = NH4+ NH4+ + OH- <---------> H2O + NH3 I 0.20 0.01 0.24 C -0.01 -0.01 +0.01 E 0.19 0 0.25 pH = pKa + log [A-]/[HA] find Ka with Kb Ka * Kb = Kw Ka * 1.8 x 10^-5 = 1.0 x 10^-14 Ka = 1.0 x 10^-14/1.8 x 10^-5 Ka = 5.6 x 10^-10 pKa = -log[Ka] = -log[5.6 x 10^-10] = 9.25 pH = 9.25 + log(0.25/0.19) pH = 9.25 + log(1.31) = 9.25 + 0.12 = 9.37 pH = 9.37 The pH of the buffer slightly increased with the addition of NaOH, but the change is not substantial.

If 0.005 mol of NaOH is added to 0.50 L of buffer solution (0.24 M NH3 + 0.20 M NH4+Cl-), what is the resulting pH? If the original buffer had a pH = 9.33 how much of a change is present when NaOH was added? (Kb of NH3 = 1.8 x 10^-5)

Since HCl is a strong acid, [HCl] = [H3O+] Find concentration of HCl 0.03 mol/0.50 L = 0.06 M HCl Base reacts to strong acids, so HCl reacts with NH3 A- = NH3 HA = NH4+ NH3 + H3O+ <---------> NH4+ + H2O I 0.24 0.06 0.20 C - 0.06 - 0.06 + 0.06 E 0.18 0 0.26 pH = pKa + log[A-]/[HA] Find Ka with Kb Ka * Kb = Kw Ka * (1.8 x 10^-5) = 1.0 x 10^-14 Ka = 1.0 x 10^-14/1.8 x 10^-5 Ka = 5.6 x 10^-10 pKa = -log[Ka] = -log[5.6 x 10^-10] = 9.25 pKa = 9.25 pH = 9.25 + log(0.18/0.26) = 9.25 + log(0.69) pH = 9.25 - 0.16 = 9.09 pH = 9.09 The pH of the buffer slightly decreased with the addition of HCl, but the change is not substantial.

If 0.03 mol of HCl is added to 0.50 L of the buffer solution (0.24 M NH3 and 0.20 M NH4+Cl-), what is the resulting pH? If the original buffer had a pH = 9.33 how much of a change is present when HCl was added? (Kb of NH3 = 1.8 x 10^-5)

Calculate moles of NaOH: 10 g NaOH/40 g/mol NaOH = 0.25 mol Find molality: 0.25 mol NaOH/0.5 kg H2O = 0.5 m

If 10 g NaOH are dissolved in 500 g of water, what is the molality of the solution?

Find moles: 184 g/92 g/mol = 2 mol C3H8O3 180 g/18 g/mol = 10 mol H2O Combine the moles to get moles of solution: 2 mol (C3H8O3) + 10 mol (H2O) = 12 mol Determine mole fractions of each: C3H8O3: 2 mol/12 mol = 1/6 = 0.17 H2O: 10 mol/12 mol = 5/6 = 0.83

If 184 g glycerol (C3H8O3) is mixed with 180 g water, what will be the mole fractions of the two components? (Note: Molar mass of H2O = 18 g; molar mass C3H8O3 = 92 g/mol)

Identify the limiting reagent: Fe: 27.9 g * 1 mol = 0.5 mol 55.9 g S: 24.1 g * 1 mol = 0.75 mol 32 g Fe is limiting S is excess 0.75 mol - 0.5 mol = 0.25 mol 0.25 mol * 32 g = 8 g S 1 mol

If 27.9 g of Fe react with 24.1 g of S to produce FeS, what would be the limiting reagent? How many grams of excess reagent would be present in the vessel at the end of the reaction?

Unsaturated dissolve super-saturated precipitation saturated not dissolve

If IP < Ksp, the solution is ____________, and the solute will ______________ If IP > Ksp, the solution is ________________, and the solute will _______________. If IP = Ksp, the solution is _______________, and the solute will ______________________________

10^-4

If K is less than or equal to __________, you assume x is negligible in an ICE chart?

anhydride

If ______________ is not the highest priority functional group in an organic molecule, then the molecule will have a prefix of alkanoyloxycarbonyl-.

alkene

If ______________ is not the highest priority functional group in an organic molecule, then the molecule will have a prefix of alkenyl- (i.e.: methenyl).

ester

If ______________ is not the highest priority functional group in an organic molecule, then the molecule will have a prefix of alkoxycarbonyl-.

alkyne

If ______________ is not the highest priority functional group in an organic molecule, then the molecule will have a prefix of alkynyl- (i.e.: methynyl).

Amide

If _________________ is the highest priority functional group in an organic molecule, then the molecule will have a suffix of -amide.

Carboxylic acid

If _________________ is the highest priority functional group in an organic molecule, then the molecule will have a suffix of -oic acid.

Ketone

If _________________ is the highest priority functional group in an organic molecule, then the molecule will have a suffix of -one.

Alkyne

If _________________ is the highest priority functional group in an organic molecule, then the molecule will have a suffix of -yne.

Anhydride

If _________________ is the highest priority functional group in an organic molecule, then the molecule will have a suffix of anhydride.

Nonpolar covalent Polar covalent Ionic

If a bond has a ΔEN (difference in electronegativity) between 0 - 0.5, the bond is ____________________. If a bond has a ΔEN between 0.5 - 1.7, the bond is _______________________. If a bond has a ΔEN greater than 1.7, the bond is ____________________.

Answer: B. Recall that oxygen has an oxidation state of -2. Therefore, in tungsten(IV) oxide, (A.), tungsten has an oxidation state of +4. In tungsten(VI) oxide, (B.), it has an oxidation state of +6. In tungsten(III) oxide, (C.), it is +3. In tungsten pentoxide, (D.), it is +5.

If a certain metal has multiple oxidation states, its acidity as an oxide generally increases as the oxidation state increases. Therefore, which of the following tungsten compounds is likely to be the strongest acid? A. WO2 B. WO3 C. W2O3 D. W2O5

It means that the component has a high affinity for the stationary phase, and thus moves slowly through the stationary phase.

If a component of the analyte has high absorption, what does that mean?

It means that the component has high affinity for the mobile phase, and thus moves quickly through the stationary phase.

If a component of the analyte has high solubility, what does that mean

Highest Substituents -e -oic acid

If a molecule contains a carboxylic acid, it will have the ______________ priority, while other functional groups are named ________________ using prefixes. Carboxylic acids are named by replacing the ______ in the name of the parent alkane/alkene/alkyne with the suffix _______________.

no net dipole moment nonpolar a net dipole moment polar

If a molecule has a molecular geometry where the dipole moments of each bond cancels each other out, the molecule has __________________ and is ___________________. If a molecule has a molecular geometry where the dipole moments of each bond does not cancels each other out, the molecule has _________________ and is _____________________.

KE(photoelectron) = E(photon) - E° E° = 3.43 x 10^-19 J E(photon) = hν (ν = c/λ) E(photon) = h*(c/λ) 525 nm = 5.25 x 10^-7 m = 6.626 x 10^-34 J*s * (3 x 10^8 m/s/5.25 x 10^-7 m) E(photon) = 3.79 x 10^-19 KE(photoelectron) = 3.79 x 10^-19 J - 3.43 x 10^-19 J KE(photoelectron) = 3.6 x 10^-20 J The KE of the photoelectron is positive, so a photoelectron will be produced. 3.6 x 10^-20 J = (1/2)mv^2 m(electron) = 9.11 x 10^-31 kg 3.6 x 10^-20 kg*(m^2/s^2) = (1/2)(9.11 x 10^-31 kg)*v^2 7.2 x 10^-20 kg*(m^2/s^2) = (9.11 x 10^-31 kg) * v^2 7.9 x 10^10 (m/s)^2 = v^2 sqrt(7.9 x 10^10 (m/s)^2) = sqrt(v^2) v = 2.8 x 10^5 m/s

If a photon of wavelength 525 nm hits metallic cesium (work function = 3.43 x 10^-19 J), will it be able to produce a photoelectron? If so, what is the velocity of the photoelectron produced?

KE(photoelectron) = E(photon) - E° E° = 3.43 x 10^-19 J E(photon) = hν ν = c/λ E(photon) = h*(c/λ) λ = 625 nm = 6.25 x 10^-7 m E(photon) = 6.626 x 10^-34 J*s * (3 x 10^8 m/s/6.25 x 10^-7 m) E(photon) = 3.18 x 10^-19 J KE(photoelectron) = 3.18 x 10^-19 J - 3.43 x 10^-19 J KE(photoelectron) = -2.5 x 10^-20 J KE(photoelectron) is negative, so photon will not have enough energy to produce a photoelectron.

If a photon of wavelength 625 nm hits metallic cesium (work function 3.43 x 10^-19 J), will it be able to produce a photoelectron? If so, what is the velocity of the photoelectron produced?

Substituents oxo- keto- oxo-

If aldehydes or ketones are not the highest priority group in a molecule, they are treated as __________________. In cases like this, we use the prefix ________, which is used for the carbonyl carbon of either aldehydes or ketones. Sometimes, ketones will be indicated with the prefix __________ instead of ______________.

Answer: C. The Kelvin unit and Celsius degree are the same size; that is, a change of 10 K is equal to a change of 10°C. One degree Celsius is equal to 1.8 degrees Fahrenheit; therefore, 10°C = 18°F.

If an object with an initial temperature of 300 K increases its temperature by 1°C every minute, by how many degrees in Fahrenheit will its temperature increased in 10 minutes? A. 6°F B. 10°F C. 18°F D. 30°C

If the photons have a frequency of 6.00 x 10^14 Hz, each photon has an energy of: Ep = hν = 4.14 x 10^-15 eV*s * 6.00 x 10^14 Hz = 2.48 eV Clearly then, any given photon has more than enough energy to allow an electron in the metal to overcome the 2.26 eV barrier. In fact, the maximum excess kinetic energy carried away by the electron turns out to be: KE = Ep - E° = 2.48 - 2.26 = 0.22 eV

If blue light of frequency 6.00 x 10^14 Hz is incident on rubidium (E° = 2.26 eV), will there be photoejection of electrons? If so, what is the maximum kinetic energy that an ejected electron will carry away? (Note: h = 6.626 x 10^-34 J*s = 4.14 x 10^-15 eV*s)

Find moles: 11 g CaCl2/ 111.1 g/mol CaCl2 = 0.1 mol CaCl2 Find molarity: 0.1 mol CaCl2/ 0.1 L solution = 1 M

If enough water is added to 11 g of CaCl2 to make 100 mL of solution, what is the molarity of the solution?

n=2 l=1: 2p: B, C, N, O, F, Ne n=3 l=0: 3s: Na, Mg n=5 l=3: 5f: Ac, Th, Pa, U, Np, Pu, Am, Cm, Bk, Cf, Es, Fm, Md, No, Lr n=4 l=2 4d: Y, Zr, Nb, Mo, Tc, Ru, Rh, Pd, Ag, Cd

If given the following quantum numbers, which element(s) do they likely refer to? 1. n=2 l=1 2. n=3 l=0 3. n=5 l=3 4. n=4 l=2

No, it is possible to force heat from a cold object to a hot object, but not spontaneously. It requires work to actually do this.

If heat flows from a colder object to a hotter object in thermal contact, does this disobey the second law of thermodynamics?

Endothermic.

If intermolecular interactions prior to dissolution are stronger than intermolecular interactions formed by dissolution, the reaction will be __________________.

Stop working Denature Disable

If our blood reaches a pH below physiological pH, the cells of our body will _______ _________________. Enzymes will _________________, which will ___________ their catalytic abilities

pKa = -log[Ka] = -log(1.8 x 10^-5) = 4.74 pKa = 4.74

If the Ka of an acid is 1.8 x 10^-5, then what is its pKa?

Frequency Threshold frequency Energy Frequency Threshold frequency Kinetic energy Energy Work function

If the _______________ of the photon is less than the _____________________ (f < f(T)), then no electron will be ejected from the metal, because the photons do not have sufficient ___________ to dislodge the electron from its atom. If the _________________ of the photon is greater than the _____________________ (f > f(T)), then a electron will be ejected. The maximum _____________________ of the ejected electron (photoelectron) is equal to the ______________ of the photon and the ____________________ of the metal.

Energy Work function Energy Energy Work function

If the _________________ of the photon is less than the ________________ of the metal (E(photon) < E°), then no electron will be ejected from the metal, because the photons do not have sufficient ___________ to dislodge the electron from its atom. If the ______________ of the photon is greater than the _____________ of the metal (E(photon) > E°), then a electron will be ejected.

Increase

If the anion in a solution is a conjugate of a weak acid, would the addition of an acid increase, decrease, or not affect the solubility of the solute?

Answer: C. In butane, the position at which the two methyl groups are 120° apart is an eclipsed conformation. This has a moderate amount of energy, although not as high as a totally eclipsed conformation in which the two methyl groups are 0° apart.

If the methyl groups of butane are 120° apart, as seen in a Newman projection, this molecule is in its: A. Highest-energy gauche form B. Lowest-energy staggered form C. Middle-energy eclipsed form D. Highest-energy eclipsed form

Answer: C. To determine the speed of the electrons ejected, we must first calculate their kinetic energy: KE = E(photon) - E° E(photon) = hν = 6.626 x 10^-34 J*s * 1.0 x 10^14 Hz E(photon) = 6.626 x 10^-20 J KE = 6.626 x 10^-20 J - 6.622 x 10^-20 J = 4 x 10^-23 J KE = 1/2mv^2 v^2 = 2(4 x 10^-23 kg*m^2/s^2)/9.11 x 10^-31 kg v = 9370.997 m/s = 9.38 x 10^3 m/s

If the work function of a metal is 6.622 x 10^-20 J and a ray of electromagnetic radiation with a frequency of 1.0 x 10^14 Hz is incident on the metal, what will be the speed of the electrons ejected from the metal? (Note: h = 6.626 x 10^-34 J*s and mass of an electron = 9.11 x 10^-31 kg) A. 2.62 x 10^-6 m/s B. 1.07 x 10^-4 m/s C. 9.38 x 10^3 m/s D. 3.81 x 10^5 m/s

Answer: D. In column chromatography, as in TLC, the less polar compound travels most rapidly. This means that 1-naphthalenemethanol, with the highest Rf value, would travel most rapidly and would be the first to elute from the column.

If these compounds were separated by column chromatography with ether on silica gel, which would elute first? A. Benzyl alcohol B. Benzyl acetate C. p-Nitrophenol D. 1-Naphthalenemethanol

Vacuum distillation would be the best technique to separate two chemicals with such high boiling points because the decreased ambient pressure will allow them to boil at a low temperature.

If we are given a solution of bromobenzene, with a boiling point of 156°C, and camphor, with a boiling point of 204°C, which type of distillation should be used to separate them?

A solution of ether and methylene chloride, which have very close boiling points, can be separated by using fractional distillation.

If we are given a solution of ether, with a boiling point of 308 K, and methylene chloride, with a boiling point of 313 K, which type of distillation should be used to separate them?

Balance the equation: 4NH3 + 5O2 ---> 4NO + 6H2O MW NH3 = 14 + 3(1) = 17 g/mol MW O2 = 2(16) = 32 g/mol 34 g NH3 * 1 mol NH3 = 2 mol 17 g NH3 32 g O2 * 1 mol O2 = 1 mol O2 32 g O2 2 mol NH3/4 = 0.25 mol 1 mol O2/5 = 0.2 mol O2 < NH3 O2 is limiting reagent 32 g O2 * 1 mol O2 * 4 mol NO * 30 g NO = 24 g NO 32 g O2 5 mol O2 1 mol NO

If we have 34 g of NH3 and 32 g of O2, how many grams of NO will be produced? NH3 + O2 ---> NO + H2O

Assume there is 100 g of substance. 27% Cl = 27 g Cl 73% Hg = 73 g Hg Divide by atomic mass: 27g Cl * 1 mol Cl = 0.76 mol 35.45 g Cl 73 g Hg * 1 mol Hg = 0.36 mol 200.59 g Hg Comparing the mols of each substance, we find that there is a 1:2 molar ratio between Hg:Cl. So, the empirical formula would be HgCl2.

If we have a bag that contains 73% Hg and 27% Cl, what would be the empirical formula of the substance in the bag.

Position Momentum Momentum Position

If we want to assess the _________________ of an electron, the electron has to stop moving (thereby removing _______________). If we want to assess the ____________________ of an electron, the electron has to be moving (thereby changing its ___________________).

Balance the equation: Fe2O3 + 2Al ---> Al2O3 + 2Fe MW Fe2O3 = 2(56) + 3(16) = 160 g/mol MW Al = 27 g/mol 85 g Fe2O3 * 1 mol Fe2O3 * 2 mol Al * 27 g Al = 28.7 g Al 160 g Fe2O3 1 mol Fe2O3 1 mol Al

If you are given 85 g of iron (III) oxide, how many grams of aluminum do we need? Fe2O3 + Al ---> Al2O3 + Fe

180 g/60 g/mol = 3 mol carbonate 250 mL * 1 g/mL = 250 g = 0.25 kg water 3 mol carbonate/0.25 kg water = 12 m molality = 12 m

If you mix 180 g of carbonate in 250 mL of water (density = 1 g/mL) what is its concentration in molality? (Note: Molar mass of carbonate is 60 g/mol)

Molality: Find kg water: 250 mL * 1 g/mL = 250 g = 0.25 kg H2O Find moles of glucose: 180 g/ 180 g/mol = 1 mol Find molality: 1 mol (glucose)/ 0.25 kg (water) = 4 m molality = 4 m

If you mix 180 g of glucose in 250 mL of water (density = 1 g/mL) what is its concentration in molality? (Note: Molar mass of glucose is 180 g/mol)

Electron Photocurrent Photoelectric effect

If you shine a light onto a metal, the photon will be able to knock a ______________ loose, causing a __________________ to flow. This is known as the _____________________ effect.

The amount of heat added did not exceed the amount needed to completely melt the block of ice: Q = mL m = 200 g L = 333 J/g Q = 200 g * 333 J/g = 6.66 x 10^4 Heat used is 5.46 x 10^4 J Therefore, no heat was applied to change the temperature of the resulting liquid and T remains constant. ΔS = Q(rev)/T Q(rev) = 5.46 x 10^4 J T = 273 K ΔS = 5.46 x 10^4 J/273 K = 200 J/K ΔS = 200 J/K

If, in a reversible process, 5.46 x 10^4 J of heat is used to change a 200 g block of ice to water at a temperature of 273 K, what is the change in entropy of the system? (Note: Heat of fusion for ice is 333 J/g)

Net dipole moment polar nonpolar

In a Lewis structure, if the orientation of the bond diploes do not cancel out the molecules have a ___________________ and are __________. In a Lewis structure if the individual bond dipole moments cancel each other out the result is a ______________ molecule.

If the volume decreases (compression), it means that work is being done on the system. Thus, work will be positive.

In a PV diagram, if the curve shows a decrease in volume, what does that say about the work being exerted?

If the volume increases (expansion), it means that work is being done by the system. Thus, work will be negative.

In a PV diagram, if the curve shows an increase in volume, what does that say about the work being exerted?

Nonpolar covalent

In a bond between carbon and hydrogen, carbon has an electronegativity value of 2.55 while hydrogen has an electronegativity value of 2.11. The electronegativity value of these atoms tells us that this bond is _____________________.

X-axis: The amount of time it takes for the analytes to pass through the detector. Y-axis: The amount of specific analyte that passes through the detector at a specific point in time.

In a gas chromatogram, what does the x-axis and y-axis represent?

The alcohol

In a molecule with two double bonds adjacent to each other and an alcohol, which functional group would take precedence in naming.

=

In a neutral atom, the # of protons _________ # of electrons.

Find moles MnO4-: M = mol/L M = 0.02 L = 20 mL = 0.02 L mol = 0.02 * 0.02 = 0.0004 mol Find moles Fe(2+): 1 mol MnO4- : 5 mol Fe(2+) 0.0004 mol MnO4- * (5 mol Fe(2+)/1 mol MnO4-) = 0.002 mol Fe(2+) Find [Fe(2+)]: M = mol/L mol = 0.002 mol L = 10 mL = 0.01 L M = 0.002/0.01 = 0.2 M [Fe(2+)] = 0.2 M

In a redox titration, 20 mL of a 0.02 M solution of K+MnO4- is needed to neutralize 10.0 mL of Fe(2+). What was the concentration of the acidic solution? MnO4- + 5Fe(2+) + 8H+ ---> Mn(2+) + 5Fe(3+) + 4H2O

Find normality for Ba(OH)2: Normality = equivalents * M equivalents = 2 OH- M = 0.0154 M N = 2 * 0.0154 M = 0.308 N Find Normality for HCl: N1V1 = N2V2 0.308 * 0.0274 = N2 * 0.0200 N2 = (0.308 * 0.0274)/0.0200 = 0.0422 N Find [HCl]: M = N/equivalents equivalents = 1 H+ M = 0.0422/1 = 0.0422 N [HCl] = 0.0422 M

In a titration, 27.4 mL of a 0.0154 M solution of Ba(OH)2 is needed to neutralize 20.0 mL of HCl. What was the concentration of the acid solution?

Find normality of NaOH: N = M * equivalents equivalents = 1 OH- M = 0.100 M N = 0.100 * 1 = 0.100 Find normality of HCl: N1V1 = N2V2 V1 = 48.6 mL = 0.0486 L N1 = 0.100 M V2 = 20.0 mL = 0.0200 L 0.0486 * 0.100 = N2 * 0.0200 (0.0486 * 0.100)/0.0200 = N2 N2 = 0.243 Find [HCl] M = N * equivalents equivalents = 1 H+ N = 0.243 M = 0.243/1 = 0.243 M [HCl] = 0.243 M

In a titration, 48.6 mL of a 0.100 M solution of NaOH is needed to neutralize 20.0 mL of HCl. What was the concentration of the acid solution?

[H3O+] [OH-]

In acid-base calculations: pH = -log_______ pOH = -log_______

Answer: B. To understand this question, make sure you understand all the terms. An adiabatic process means that there is no exchange of heat; in other words, Q = 0. When a gas is compressed, positive work is being done on the gas (instead of by the gas), so the value of work done on the gas will be positive (W > 0). Based on this, we can determine how the internal energy of the gas changes by using the first law of thermodynamics (ΔU = Q + W) If Q = 0, and W is positive, then ΔU is positive.

In an adiabatic compression process, the internal energy of the gas: A. increases, because the work done on the gas is negative. B. increases, because the work done on the gas is positive. C. decreases because the work done on the gas is negative. D. decreases because the work done on the gas is positive.

Answer: B. When the ink randomly intersperses throughout the water, the final state is more disordered than the initial state, so the entropy change of the system is positive. When the oil separates from the water, the final state is just as ordered as the initial state (because the oil and the water are still completely separate), so the entropy change is zero. You can also answer this question by noticing the reversibility of the two experiments. Experiment A has a positive entropy change because it is irreversible, while experiment B has no entropy change because the reaction is reversible. According to the second law of thermodynamics, the overall entropy change of a system and its surroundings can never be negative in a thermodynamic process that moves from one equilibrium state to another.

In experiment A, a student mixes ink with water and notices that the two liquids mix evenly. In experiment B, the student mixes oil with water; in this case, the liquids separate into two different layers. The entropy change is: A. positive in experiment A and negative in experiment B B. positive in experiment A and zero in experiment B C. negative in experiment A and positive in experiment B D. zero in experiment A and negative in experiment B.

Volatile: have a low melting point or are solids that are sublimable or liquids that are vaporizable.

In gas chromatography, the injected substance must be ______________________ in order for the process to work.

-e -al One

In naming a organic molecule, aldehydes are named by replacing the _____ of the parent alkane/alkene/alkyne with the suffix ______. When aldehyde is at carbon ___________, which is usually the case, we do not need to include its number in the chemical name.

Prefix Highest priority Suffix

In naming an organic molecule, a substituent's name will be placed at the beginning of the compound name as a _______________, followed by the name of the parent chain. The only exception to this rule is the _________________ ________________ functional group, which determines the ___________ of the compound and must be part of the parent chain.

-e -ol hydroxyl- alcohol

In naming an organic molecule, if alcohol is the highest priority functional group, then you replace the ____ in the name of the corresponding alkane/alkene/alkyne with ______. If alcohol is not the highest priority functional group, then it is named as a substituent with the prefix _________________. In the common way of naming alcohols, the name of the alkyl group is followed by the word _________________.

Substituents

In naming an organic molecule, if there are 2 or more chains of equal length, the chain with the most ____________________ gets priority as the parent chain.

-e -one lowest possible number

In naming an organic molecule, ketones are named by replacing the ____ in the name of the parent alkane/alkene/alkyne with the suffix ________. If ketone is the highest priority group, the carbonyl carbon of the ketone must have the ____________ ______________ ____________.

Lower-numbered

In naming an organic molecule, the double or triple bond is named like a substituent, and is indicated by the ______________________ carbon involved in the bond.

Commas Hyphens

In naming an organic molecule, the numbers corresponding to the functional groups are separated from each other with ____________, and numbers are separated from words with _____________.

Substituents Closest Low

In naming an organic molecule, the parent chain must be numbered to identify the location of the ____________________. The carbon numbered 1 will be the one ___________ to the highest priority functional group. If functional groups all have the same priority, make the numbers assigned to the substituted carbon to be as _______ as possible.

Nitrogen atom Prefixes N- Nitrogen Numbered -e/-oic acid -amide

In naming organic molecules, amides have two parts to it: -The first term is the substituents that are attached to the ________________. These substituents are included as ____________ and are labeled with _____, indicating that this group is bonded to a parent molecule via ________________. These substituents are not ________________. -The second term consists of the parent chain, with ______________ being replaced by _____________ as the suffix.

Alkyl Alkyl Oxygen -oate -e/-oic acid

In naming organic molecules, naming esters has two parts to them: -The first term is the ________ name of the esterifying group. First term is based off the ________ that is bonded to the single bonded _______________. -Second term is the name of the parent chain, with ____________ replacing __________ as the suffix.

The exact amount of energy

In order to get move an electron to a higher energy level, it must receive _____________________ energy in order to reach that level.

Answer: C. Affinity chromatography, using the target for the biological effector or a specific antibody, would work best in this case. It will specifically bind the protein of interest and keep it in the column.

In order to separate a biological effector from solution, which chromatographic technique would be the most effective? A. Thin-layer chromatography B. Ion-exchange chromatography C. Affinity chromatography D. Size-exclusion chromatography

Capillary action in the stationary phase

In paper chromatography and TLC, what is the main mechanism of action that pushes the mobile phase through the stationary phase?

If we poured in too much of our mobile phase, then it may interfere with our sample, causing it to bleed all over the stationary phase.

In paper chromatography and TLC, why is it important to only pour in a small amount of our mobile phase into the reaction chamber?

Partial negative (-δ) Partial positive (+δ)

In polar covalent bonds, the atom that has a greater pull of the electrons has a _____________________ charge while the atom with a weaker pull of the electrons will have a _______________________ charge.

Principle Azimuthal/Angular Magnetic Spin

In quantum numbers: -n represents the ________________________ quantum number. -l represents the _________________________ quantum number. -ml represents the ______________________ quantum number -ms represents the ______________________ quantum number

s-subshell d-subshell s-subshell d-subshell

In regards to the electron configuration of transition metals, the __________ subshell becomes higher in energy than the __________ subshell. Because of this, when a transition metal becomes a cation, valence electrons are first removed from the _____________ subshell before removing electrons from the ______________ subshell.

Polar: Mobile phase Nonpolar: Stationary phase

In reverse-phase paper chromatography, TLC, and simple column chromatography, which phase is typically polar and which phase is typically nonpolar?

Large molecules. The stationary phase of this chromatography are microporous beads, in which small molecules get trapped in, while large molecules just move around the beads.

In size-exclusion chromatography, do large molecules elute first, or do small molecules elute first? Explain your reasoning.

Formaldehyde Acetaldehyde Propionaldehyde

In the case of methanal, ethanal, and propanal, they are almost always referred to by their common names: ____________________, __________________, and __________________, respectively.

Answer: C. Start with the atoms that have oxidation states of which you are certain. Potassium is a Group IA metal, and therefore must have an oxidation state of +1. Hydrogen is almost always +1, unless it is paired with a less electronegative element (which is not the case here). Oxygen is generally -2. Because there are four oxygens, they create a total negative charge of -8 which is partially balanced by two hydrogens (+2) and potassium (+1). Therefore, phosphorous has a +5 charge, making it the highest oxidation state.

In the compound KH2PO4, which element has the highest oxidation number? A. K B. H C. P D. O

Positive heat implies that heat is being added to the system. Negative heat implies that heat is being removed from the system.

In the equation for the First Law of Thermodynamics, what is the difference of heat being positive vs. heat being negative.

Positive work implies that work is being done on the system, and thus internal energy is added to the system. Negative work implies that work is being done by the system, and thus internal energy is removed from the system.

In the equation for the First Law of Thermodynamics, what is the difference of work being positive vs. work being negative?

Bronsted-Lowry Acid: H2O Bronsted-Lowry Base: HCO3- Conjugate Acid: H2CO3 Conjugate Base: OH-

In the following equation, identify which substance is: -Bronsted-Lowry Acid -Bronsted-Lowry Base -Conjugate Acid -Conjugate Base

Bronsted-Lowry Acid: H2O Bronsted-Lowry Base: NH3 Conjugate Acid: NH4+ Conjugate Base: OH-

In the following equation, identify which substance is: -Bronsted-Lowry Acid -Bronsted-Lowry Base -Conjugate Acid -Conjugate Base

Bronsted-Lowry Acid: H2SO4 Bronsted-Lowry Base: H2O Conjugate Acid: H3O+ Conjugate Base: HSO4-

In the following equation, identify which substance is: -Bronsted-Lowry Acid -Bronsted-Lowry Base -Conjugate Acid -Conjugate Base

The hydrogen chloride molecule (hydrochloric acid) is the Bronsted-Lowry Acid, since it donates a H+ proton to water. The water molecule is the Bronsted-Lowry Base, since it accepts a H+ proton from the hydrochloric acid. The conjugate acid is the hydronium (H3O+) ion, since it was formed as a result of water accepting the H+ proton. The conjugate base is the chloride (Cl-) ion, since it was formed as a result of HCl donating a proton to water.

In the following equation, identify which substance is: -Bronsted-Lowry Acid -Bronsted-Lowry Base -Conjugate Acid -Conjugate Base

Lewis Acid: Ag+ Lewis Base: 2NH3

In the following equation, identify which substance is: -Lewis Acid -Lewis Base

Lewis Acid: BF3, since it is accepting an electron pair Lewis Base: F-, since it is donating an electron pair

In the following equation, identify which substance is: -Lewis Acid -Lewis Base

Lewis Acid: HCl Lewis Base: H2O

In the following equation, identify which substance is: -Lewis Acid -Lewis Base

Lewis Acid: SO3 Lewis Base: O(2-)

In the following equation, identify which substance is: -Lewis Acid -Lewis Base

Find the oxidation states: 0 +1 -2 -1 +1 -1 +1 +5 -2 +1 -2 3Cl2 + 6NaOH ---> 5NaCl + NaClO3 + 3H2O 0 +1 -2 -1 +1 -1 +1 +5 -6 +2 -2 Cl is oxidized in NaClO3 and reduced in 5NaCl, thus it is the element undergoing disproportionation. Cl has an oxidation state of +5 in NaClO3, and -1 oxidation state in NaCl.

In the following reaction which element undergoes disproportionation? What are the element's oxidation states in these products? 3Cl2 (g) + 6NaOH (aq) ---> 5NaCl (aq) + NaClO3 (aq) + 3H2O (l)

Find the oxidation states: +2 -2 +1 0 +4 -2 +1 -2 S2O3(2-) + 2H+ ---> S + SO2 + H2O +4 -6 +1 0 +4 -4 +2 -2 S is oxidized in SO2 and reduced in S, thus it is the element undergoing disproportionation. S has an oxidation state of +4 in SO2, and 0 oxidation state in S.

In the following reaction which element undergoes disproportionation? What are the element's oxidation states in these products? S2O3(2-) (aq) + 2H+ (aq) ---> S (s) + SO2 (g) + H2O (l)

Answer: B. A limiting reagent is by definition a reactant. Because Au and H2S are products, they cannot act as limiting reagents, eliminating (C) and (D). Next, note that the given equation is unbalanced and the first step is to balance it: Au2S3 + 3H2 ---> 2Au + 3H2S The problem states that 2 moles of gold(III) sulfide and 5 moles of hydrogen gas are available. To use up both moles of gold (III) sulfide, 6 moles of hydrogen gas are needed because there is a 1:3 ratio between these reactants. Since only 5 moles of hydrogen gas are present, that will have to be the limiting reagent.

In the following reaction: Au2S3 + H2 ---> Au + H2S If 2 moles of Au2S3 is reacted with 5 moles of hydrogen gas, what is the limiting reagent? A. Au2S3 B. H2 C. Au D. H2S

Group 2 Group 15 Group 18 decreasing

In the periodic trend of electron affinity, elements in Group ______, Group _____, and Group _____ defy this trend by _________________ in electron affinity from the preceding element.

Group 13 Group 16 Decreasing

In the periodic trend of ionization energy, elements in Group ______ and Group _____ defy this trend by _________________ in ionization energy from the preceding element.

Answer: B. 30 g H2O * 1 mol H2O * 1 mol glucose * 180 g glucose = 50 g glucose 18g H2O 6 mol H2O 1 mol glucose

In the process of photosynthesis, carbon dioxide and water combine with energy to form glucose and oxygen, according to the following equation: CO2 + H2O ---> C6H12O6 + O2 What is the theoretical yield of glucose if 30 grams of water are reacted with excess carbon dioxide and energy, according to the equation above? A. 30.0 g B. 50.0 g C. 300.1 g D. 1801 g

Answer: A. Equation is balanced MW Na2S: Na: 23 x 2 = 46 S: 32 32 + 46 = 78 g/mol MW AgNO3: Ag: 107.9 N: 14 O: 16 x 3 = 48 107.9 + 48 + 14 = 169.9 g/mol Na2S: 39.05 g * 1 mol = 0.5 mol 78 g/mol AgNO3: 85.5 g * 1 mol = 0.5 mol/2 = 0.25 mol 169.9 g Na2S is excess reagent 0.5 - 0.25 = 0.25 mol 0.25 mol * 78 g/mol = 19.5 g

In the reaction shown, if 39.05 g of Na2S are reacted with 85.5 g of AgNO3, how much of the excess reagent will be left over once the reaction has gone to completion? Na2S + 2AgNO3 ---> Ag2S + 2NaNO3 A. 19.5 g Na2S B. 26.0 g Na2S C. 41.4 g AgNO3 D. 74.3 g AgNO3

Answer: A. In this Lewis diagram, the phosphate molecule has an overall formal charge of -3. The four oxygen atoms would each be assigned a formal charge of -1. Given the overall charge of -3 and the -1 charge on each oxygen, the phosphorous must have a formal charge of +1.

In the structure shown, which atom(s) have the most positive charge? A. The phosphorous atom has the most positive charge B. All atoms share the charge equally C. The four oxygen atoms share the highest charge D. The oxygen atom at the peak of the trigonal pyramidal geometry has the most positive charge

Buffer regions

In the titration curve shown, what are the highlighted regions referred to as?

The equivalence point

In the titration curve shown, what is the red arrow pointing to?

Expose the plate to UV light, which will show compounds that are UV sensitive. Another method is to stain the spots with iodine, phosphomolybdic acid, or vanillin. But this can irreversibly destroy the compounds in the sample.

In thin-layer chromatography, after the mobile phase has ran far enough through the stationary phase, it is observed that the components of the sample used for the chromatography cannot be seen by the naked eye. What are some methods that allows us to visibly see the results of TLC?

You surround the chamber with paper that will absorb the mobile phase, which will allow for the vapors from the mobile phase to be present throughout the chamber. This is done in order to prevent the stationary phase from drying up while the mobile phase travels up it.

In thin-layer chromatography, how do you saturate the developing chamber containing the mobile phase? What is the importance of saturating the chamber?

If we leave the developing chamber open, then the solvent will evaporate and escape the chamber.

In thin-layer chromatography, what is the importance of sealing the developing chamber once we have placed our stationary phase in it?

As the name suggests, gas chromatography is simply the same technique of mobile and stationary phases performed with a gaseous eluent (instead of liquid). The stationary phase is usually a crushed metal or polymer.

In what way is gas chromatography distinct from all of the other techniques we have discussed?

Answer: A. Acetone = 66.7% Ethanol = 50% Propane = 80% Methanol = 37.5%

In which of the following compounds is the percent composition of carbon by mass closest to 62%? A. Acetone B. Ethanol C. Propane D. Methanol

Great Ion-dipole interactions

Ionic compounds in aqueous solutions are ________________ conductors of electricity since the lattice arrangement is disrupted by the ______________________________ between the ionic components and the water molecules, allowing free movement for the cations and anions.

Answer: C. Ionization energy increases from left to right, so the first ionization energy of lithium is lower than that of beryllium. Second ionization energy is always larger than first ionization energy, so beryllium's second ionization energy should be the highest value. This is because removing an additional electron from Be+ requires one to overcome a significantly larger electrostatic force.

Ionization energy contributes to an atom's chemical reactivity. Which of the following shows an accurate ranking of ionization energies from lowest to highest? A. first ionization energy of Be < second ionization energy of Be < first ionization energy of Li B. first ionization energy of Be < first ionization energy of Li < second ionization energy of Be C. first ionization energy of Li < first ionization energy of Be < second ionization energy of Be D. first ionization energy of Li < second ionization energy of Be < first ionization energy of Be

Endothermic process Exothermic process

Ionization energy is calculated from an ___________________ process, since the removal of electrons requires energy to execute. Electron affinity is calculated from an ___________________ process, since the addition of an electron dispels energy in the form of heat.

Chiral Has chiral centers, lacks symmetry

Is the following molecule chiral, achiral, or meso?

Chiral No plane of symmetry

Is the following molecule chiral, achiral, or meso?

Meso Has chiral centers, but has a plane of symmetry.

Is the following molecule chiral, achiral, or meso?

(E)

Is the following molecule designated as (E) or as (Z)?

Achiral No chiral centers

Is the following molecule, chiral, achiral, or meso?

Meso Has chiral centers, but has a plane of symmetry.

Is the following molecule, chiral, achiral, or meso?

Achiral since it does not have 4 unique substituents

Is the shown molecule chiral, achiral, or meso?

Chiral since the carbon has 4 unique substituents.

Is the shown molecule chiral, achiral, or meso?

Chiral, the carbon has 4 unique substituents

Is the shown molecule chiral, achiral, or meso?

Achiral, since none of the carbons have 4 unique substituents,

Is this molecule chiral, achiral, or meso?

Achiral, since none of the carbons have 4 unique substituents.

Is this molecule chiral, achiral, or meso?

Chiral, since the carbon bonded to bromine has 4 unique substituents. Substituents: -H -Br -CH2OCH2 -CH2CH2CH2

Is this molecule chiral, achiral, or meso?

Answer: B. 200 ppb of Pb(2+) is equivalent to 200 grams of Pb(2+) in 10^9 grams solution; given the extremely low concentration of lead, the mass of the water can be assumed to be approximately 10^9 grams, as well. To solve, set up a dimensional analysis question. The units needed at the end are moles/L (molarity), so convert from grams of lead to moles of lead and grams of water to liters of water: g (Pb(2+))/10^9 g (H2O) * density (H2O)/molar mass (Pb(2+)) 200 g Pb(2+) * 10^3 g H2O = 1 mol Pb(2+) = 200 x 10^3 mol 10^9 g H2O..................1L....................207.2 g Pb(2+)......207.2 x 10^9 L = 9.65 x 10^-7 mol/L Note that the denominator was rounded to a smaller number, meaning the estimated answer is slightly larger than the actual value.

Lead is a toxic element that can cause many symptoms, including mental retardation in children. If a body of water is polluted with lead ions at 200 ppb (parts per billion), what is the concentration of lead expressed in molarity? (Note: the density of water is 1 g/mL, and ppb = grams per 10^9 grams of solution) A. 9.7 x 10^-10 M Pb(2+) B. 9.7 x 10^-7 M Pb(2+) C. 6.2 x 10^-7 M Pb(2+) D. 6.2 x 10^-6 M Pb(2+)

[H3O+] = 2.2 x 10^-3 M [H3O+][OH-] = Kw [2.2 x 10^-3][OH-] = 1.0 x 10^-14 [OH-] = (1.0 x 10^-14)/(2.2 x 10^-3) = 4.5 x 10^-12 [OH-] < [H3O+] = acidic [OH-] = 4.5 x 10^-12 and the solution is acidic.

Lemon juice has a concentration of H3O+ ions at 2.2 x 10^-3 M. Find the concentration of OH- ions in the solution, and classify whether the solution is acidic or basic.

Answer: A. Hydride ions are composed of a hydrogen nucleus with two electrons, thereby giving it a negative charge and a considerable tendency to donate electrons. LiAlH4 is therefore a strong reducing agent. Strong reducing agents tend to have metals or hydrides; strong oxidizing agents tend to have oxygen or a similarly electronegative element.

Lithium aluminum hydride (LiAlH4) is often used in laboratories because of its tendency to donate hydride ions. Which of the following roles would lithium aluminum hydride likely play in a reaction? A. Strong reducing agent only B. Strong oxidizing agent only C. Both a strong reducing agent and strong oxidizing agent D. Neither a strong reducing agent nor a strong oxidizing agent

Answer: B. The periodic table is organized into periods (rows) and groups (columns). Groups (columns) are particularly significant because they represent sets of elements with the same valence electron configuration, which in turn will dictate many of the chemical properties of those elements. Although (A.) is true, the fact that both ions are positively charged does not explain the similarity in chemical properties; most metals produce positively charged ions. (C.) is not true because lithium and sodium do have relatively low atomic weights, so do several other elements that do not share the same properties, eliminating (D.).

Lithium and sodium have similar chemical properties. For example, both can form ionic bonds with chloride. Which of the following best explains this similarity? A. Both lithium and sodium ions are positively charged B. Lithium and sodium are in the same group of the periodic table C. Lithium and sodium are in the same period of the periodic table D. Both lithium and sodium have low atomic weights

van der Waals force

London dispersion forces are also a type of ____________________________.

Answer: C. All four descriptions of metal are true, but the most significant property that contributes to the ability of metals to conduct electricity is the fact that they have valence electrons that can move freely. Malleability, (A.), is the ability to shape a material with a hammer, which does not play a role in conducting electricity. The low electronegativity and high melting points of metal, (B.) and (D.), also do not play a major role in the conduction of electricity.

Metals are often used for making wires that conduct electricity. Which of the following properties of metals explains why? A. Metals are malleable B. Metals have low electronegativities C. Metals have valence electrons that can move freely D. Metals have high melting points

The reaction is already balanced: A. MW CO = 12 + 16 = 28 g/mol MW H2 = 2 g/mol 356 g CO * 1 mol CO * 1 mol CO = 12.71 mol CO 28 g CO 1 mol CO 65 g H2 * 1 mol H2 * 1 mol H2 = 16.25 mol H2 2 g H2 2 mol H2 CO < H2 CO is limiting reagent 356 g CO * 1 mol CO * 1 mol CH3OH * 32 g CH3OH = 406.86 g 28 g CO 1 mol CO 1 mol CH3OH B. 16.25 mol H2 - 12.71 mol CO = 3.54 mol H2 3.54 mol H2 * 2 mol H2 = 7.08 mol H2 7.08 mol H2 * 2 g H2 = 14.16 g H2 1 mol H2

Methanol CH3OH which is used as a fuel in racing cars and fuel cells, can be made by the reaction of carbon monoxide and hydrogen. CO + 2H2 ---> CH3OH Suppose 356 g of CO and 65 g of H2 are mixed and allowed to react. A. What mass of methanol can be produced? B. What mass of the excess reagent remains after the limiting reactant has been consumed?

Easier Decreases Harder Increases

More electron shielding makes a valence electron less attracted to the nucleus, which makes it ____________ for it to be removed. Therefore, more electron shielding _______________ ionization energy. Less electron shielding makes a valence electron more attracted to the nucleus, which makes it ______________ to remove it. Therefore, less electron shielding ______________ ionization energy.

Answer: C. The suffix -amide in nicotinamide indicates that this compound contains an amide functional group. The prefix diphospho- indicates there are two phosphate groups, as well. Even if we did not know the prefix phospho- from this chapter, we should recognize that nucleotides, mentioned in the name of the compound, contains a sugar, a phosphate group, and a nitrogenous base.

NADH is a coenzyme that releases high-energy electrons into the electron transport chain. It is known as nicotinamide adenine dinucleotide or diphosphopyridine nucleotide. What functional groups exist in this molecule? I. Phosphate II. Amide III. Anhydride A. I only B. II only C. I and II only D. I, II, and III

-Carbonic acid -Carbonate

Name the acid and it's anion: -H2CO3 -CO3(2-)

-Chromic acid -Chromate

Name the acid and it's anion: -H2CrO4 -CrO4(2-)

-Sulfuric acid -Sulfate

Name the acid and it's anion: -H2SO4 -SO4(2-)

Hypobromite

Name the common polyatomic ion: BrO-

Perbromate

Name the common polyatomic ion: BrO4-

Oxalate

Name the common polyatomic ion: C2O4(2-)

Acetate

Name the common polyatomic ion: CH3COO-

Cyanide

Name the common polyatomic ion: CN-

Hypochlorite

Name the common polyatomic ion: ClO-

Chlorite

Name the common polyatomic ion: ClO2-

Chlorate

Name the common polyatomic ion: ClO3-

Perchlorate

Name the common polyatomic ion: ClO4-

Dichromate

Name the common polyatomic ion: Cr2O7(2-)

Chromate

Name the common polyatomic ion: CrO4(2-)

Dihydrogen phosphate

Name the common polyatomic ion: H2PO4-

Nitrite

Name the common polyatomic ion: NO2-

Peroxide

Name the common polyatomic ion: O2(2-)

Hydroxide

Name the common polyatomic ion: OH-

Thiocyanate

Name the common polyatomic ion: SCN-

Sulfite

Name the common polyatomic ion: SO3(2-)

3-methyl-2-oxopentanoic acid

Name the following compound.

-Iron (II) or Ferrous -Chlorate -Hydride -Oxide -Hydrogen carbonate or Bicarbonate -Sulfate -Phosphate -Ammonium -Borate -Hypochlorite

Name the following ions: Fe(2+) ClO3- H- O(2-) HCO3- SO4(2-) P(3-) NH4+ BO3(3-) ClO-

Nitride Iron (III) or Ferric Dihydrogen phosphate Nitrate Perchlorate Copper (I) or cuprous Chromate Cyanide Nitrite Permanganate

Name the following ions: N(3-) Fe(3+) H2PO4- NO3- ClO4- Cu+ CrO4(2-) CN- NO2- MnO4-

Thiocyanate Acetate Sulfide Sulfite Copper (II) or Cupric Fluoride Chlorite Dichromate Hydrogen sulfate or bisulfate

Name the following ions: SCN- CH3COO- S(2-) SO3(2-) Cu(2+) F- ClO2- Cr2O7(2-) HSO4-

Left: 1-butene/but-1-ene Right: 2-butene/but-2-ene

Name the following molecules

Nitrate

Name the following polyatomic ion: NO3-

5-nonanone

Name the molecule

Succinic anhydride

Name the molecule

2-pentanone

Name the molecule.

3-(5-oxohexyl)cyclohexanone

Name the molecule.

Butyl methanoate

Name the molecule.

Ethyl propanoate

Name the molecule.

Methyl butanoate

Name the molecule.

N-ethyl-N-methylbutanamide

Name the molecule.

-H2 -N2 -O2 -F2 -Cl2 -Br2 -I2

Name the seven common diatomic molecules found in nature.

Answer: B. Electron affinity is related to several factors, including atomic size and filling of the valence shell. As atomic radius increases, the distance between the nucleus and the outermost electrons increases, thereby decreasing the attractive forces between protons and electrons. As a result, increased atomic radius will lead to lower electron affinity. Because atoms are in a low-energy state when their outermost valence electron shell is filled, atoms needing only one or two electrons to complete this shell will have high electron affinities. In this example, (B.) and (D.) need only one more electron to have a noble gas-like electron configuration; because (B.) is smaller, it will have the highest electron affinity.

Of the four atoms depicted here, which has the highest electron affinity?

Answer: B. Racemic mixtures like omeprazole contain equimolar amounts of two enantiomers and thus have no observed optical activity. Each of the two enantiomers causes rotation in opposite directions, so their effects cancels out. Esomeprazole only contains one of the two enantiomers and thus should cause rotation of plane-polarized light.

Omeprazole is a proton pump inhibitor commonly used in gastroesophageal reflux disease. When omeprazole, a racemic mixture, went off-patent, pharmaceutical companies began to manufacture esomeprazole, the (S)-enantiomer of omeprazole, by itself. Given 1 M solutions of omeprazole and esomeprazole, which solution(s) would likely exhibit optical activity. A. Omeprazole only B. Esomeprazole only C. Both oemprazole and esomeprazole D. Neither omeprazole nor esomeprazole

By evaporating the solvent.

Once a desired product has been successfully extracted, how can we isolate the product from the solvent?

Answer: B. The mass percent of a solute equals the mass of the solute divided by the mass of the total solution times 100%. Plug in the values given for sucrose, the volume of water and the density of water to determine the %mass of sucrose. %mass = mass solute/total mass of solution * 100% %mass sucrose = 100 g sucrose/(292.5 g water + 100 g sucrose) * 100% = 25.5%. Keep in mind that in rounding while calculating, the denominator was estimated to be larger than the actual value, thus giving an answer that is slightly lower than the actual value. Thus the correct answer is (B.), 25.5%, (A.) results if rounding error is not taken into account. While these answers are very close, the mass of water must be slightly less than 300 g, given the density value, so percent composition of sucrose must be slightly higher than 25%. If the solute's mass is not added to the solvent's, the calculated value is 34.2%, which is (D.). (C.) neglects both the addition step and rounding error.

One hundred grams of sucrose are dissolved in a cup of hot water at 80°C. The cup of water contains 300.00 mL of water What is the percent composition by mass of sugar in the resulting solution? (Note: Sucrose = C12H22O11, density of water at 80°C = 0.975 g/mL) A. 25.0% B. 25.5% C. 33.3% D. 34.2%

Answer: C. What you are shown is a net ionic equation. If two moles of FeSCN are created, two moles of Fe(3+) must be used because the mole ratio is 1:1. Iron sulfate has the formula Fe2(SO4)3 because sulfate has a charge of -2 and iron has a charge of +3 (based on the net ionic equation). Therefore, one mole of iron sulfate is needed to make two moles of iron for the reaction. The molar mass of iron sulfate is: 2 * 55.8 g/mol + 3 * 32.1 g/mol + 12 * 16.0 g/mol = 399.9 g/mol This most closely matches answer (C.). The most common error would be to calculate the amount of iron, which would be 111.6 g (A.).

One way to test for the presence of iron in solution is by adding potassium thiocyanate to the solution. The product when this reagent reacts with iron is FeSCN2+, which creates a dark red color in solution via the following net ionic equation: Fe3+ + SCN- ---> FeSCN(2+) How many grams of iron sulfate would be needed to produce 2 moles of FeSCN(2+)? A. 110 g B. 220 g C. 400 g D. 500 g

Step 1: Find molar mass of both reactants KClO3 = 39 + 35.5 + 3(16) = 122.5 g/mol P4 = 4(31) = 124 g/mol Step 2: Convert to mol: KClO3: 12 g * 1 mol = 0.1 mol 122.5 g P4: 0.33 mol * 1 mol = 0.33 mol 124 g Step 3: Divide each reagent by their stoichiometric coefficient KClO3: 0.1 mol/10 mol = 0.01 mol P4: 0.33 mol/3 mol = 0.11 mol P4 > KClO3 KClO3 is the limiting reagent Step 4: Find remaining P4 0.1 mol KClO3 = x mol P4 10 3 x mol P4 = 0.1 mol KClO3 * 3 10 x = 0.03 mol P4 Step 5: Subtract 0.33 mol by 0.03 mol: 0.33 - 0.03 = 0.3 mol P4

Phosphporus pentoxide is a powerful desicant, and can be used in organic chemistry to obtain anhydrides: 10KClO3 + 3P4 ---> 3P4O10 + 10KCl Find the limiting reagent if there is 12 g of potassium chlorate and 41 g of white phosphorus in the reaction tube and how much of the other reactant will remain unreacted.

Answer: C. Potentiometry refers to carrying out an oxidation-reduction titration with a voltmeter present to get precise readings of the reaction's electromotive force (emf) to determine the endpoint. This is analogous to using a pH meter in an acid-base titration because it uses technology to get precise readings for plotting a titration curve. Indicators, as in (A.) and (B.), can be used in both acid-base and redox titrations, but provide a qualitative (rather than quantitative) analysis of the titration. Oxidizing and reducing agents are used in redox titrations, not acid-base titrations, eliminating (D.).

Potentiometry in an oxidation-reduction titration is analogous to performing an acid-base titration with a(n): A. acidic indicator B. basic indicator C. pH meter D. oxidizing agent

Octahedral

Predict the molecular geometry of UBr6.

Answer: A. We can use the name acetylformic acid to figure out what our functional groups are. The prefix acet- refers to a two-carbon unit with one carbon in the carbonyl group - think acetic acid, acetic anhydride, or acetaldehyde. The carbonyl carbon is the point of attachment to another functional group. Formic acid is an acetyl group directly attached to formic acid, as shown in (A.). (B.) shows acetic acid, or vinegar; (C.) shows glucose; and (D.) shows formic acid.

Pyruvic acid, one of the end products of glycolysis, is commonly called acetylformic acid. Based on its common name, the structure of pyruvic acid must be:

1. C 2. Ge 3. Ca 4. K

Rank the following elements by decreasing first ionization energy: -Calcium (Ca) -Carbon (C) -Germanium (Ge) -Potassium (K)

1. Xe 2. Nb 3. Ta 4. Pr

Rank the following elements by increasing atomic radius: -Niobium (Nb) -Praseodymium (Pr) -Tantalum (Ta) -Xenon (Xe)

1. Ba 2. Y 3. Cu 4. S

Rank the following elements by increasing electron affinity: -Barium (Ba) -Copper (Cu) -Sulfur (S) -Yttrium (Y)

triple bond double bond σ bond π bond

Rank the following orbitals in decreasing order of strength: σ bond π bond double bond triple bond

1. London dispersion forces 2. Dipole-Dipole interactions 3. Hydrogen bonding

Rate the following intermolecular forces from weakest to strongest: -Dipole-dipole interactions -Hydrogen bonding -London dispersion forces

1. Metallic bond 2. Ionic bond 3. Polar covalent bond 4. Nonpolar covalent bond

Rate the following intramolecular forces from the strongest to the weakest: -Ionic bond -Nonpolar covalent bond -Polar covalent bond -Metallic bond

Answer: B. Osmotic pressure is given by the formula ∏=iMRT. Entering the values from the question stem gives: ∏ = (1000 x 10^-3 mOsm/L)(0.082 atm/molK)(298 K) = 24.2 atm Notice that the concentration of seawater is given for all solutes, which represents i x M. It is also given in mOsm/L which is converted into moles/L by multiplying 10^-3. Also, the question asks for the minimum pressure required, which means that the correct answer choice must be slightly above the calculated pressure in order for reverse osmosis to proceed.

Reverse osmosis is a process that allows fresh water to be obtained by using pressure to force an impure water source through a semipermeable membrane that only allows water molecules to pass. What is the minimum pressure that would be required to purify seawater at 25°C that has a total osmolarity of 1000 mOsm/L? A. 23.5 atm B. 24.5 atm C. 24000 atm D. 24500 atm

Since we are finding thermal equilibrium: Q(Cu) + Q(H2O) = 0 Q(Cu) = m*c*ΔT m = 0.5 kg = 500 g c = 0.387 J/g*K Ti = 90°C = 363 K Q(H2O) = m*c*ΔT m = 2 kg = 2000 g c = 4.184 J/g*K Ti = 20°C = 293 K Q(Cu) = 500 g * 0.387 J/g*K * (Tf - 363 K) Q(H2O) = 2000 g * 4.184 J/g*K * (Tf - 293 K) Q(Cu) = 193.5 J/K * (Tf - 363 K) Q(H2O) = 8368 J/K * (Tf - 293 K) Q(Cu) = 193.5 J/K * Tf - 70240.5 J Q(H2O) = 8368 J/K * Tf - 2451824 J (193.5 J/K * Tf - 70240.5 J) + (8368 J/K * Tf - 2451824 J) = 0 8561.5 J/K * Tf - 2522064.5 J = 0 8561.5 J/K * Tf = 2522064.5 J Tf = 294.58 K or 21.58°C

Say that we have a 2 kg container filled with water, sitting at a temperature of 20°C. If we were to drop a 0.5 kg block of copper with a temperature of 90°C into the water, what would be the temperature of thermal equilibrium that both the copper and water will reach. (Specific heat of copper = 0.387 J/g*K) (Specific heat of water = 4.184 J/g*K)

Hexane is nonpolar, while both phenol and acetic acid are polar and acidic. Phenol is a weaker acid compared to acetic acid, but since we have two acidic substances, we cannot use a strongly basic substance like NaOH to separate the two acids. Instead, we separate them by using a weaker base, like sodium bicarbonate (NaHCO3). Since sodium bicarbonate is a weak base, it will probably will not interact with very weak acids like phenol, but it will be able to deprotonate acetic acid. Thus the aqueous phase will consist of acetic acid, while the organic phase will consist of phenol and hexane. Once the acetic acid has been properly extracted, since the aqueous phase here is denser than the organic phase, we can now focus on separating the phenol from hexane. Since there aren't any other acidic species in the solution, it is safe to use NaOH to separate the phenol from the hexane. The aqueous phase now consists of phenol while the organic phase consists of hexane. We then extract the aqueous phase, which is denser than the organic phase, and are left with our organic phase of hexane.

Say that we have a solution containing hexane, phenol, and acetic acid. How do we extract all three of these compounds from this solution?

The difference between the two energy levels will equal the amount of energy needed to move the electron. E1 = -2.18 x 10^-18 J E2 = E1/(2^2) = -2.18 x 10^-18 J/4 = -5.45 x 10^-19 J -2.18 x 10^-18 J = -21.8 x 10^-19 J 21.8 x 10^-19 - 5.45 x 10^-19 16.35 x 10^-19 J = 1.64 x 10^-18 J eV: E1 = -13.6 eV E2 = E1/(2^2) = -13.6 eV/4 = -3.4 eV 13.6 eV - 3.4 eV 10.2 eV

Say that we want to cause an electron to go from the first energy level (n=1) to the second energy level (n=2). How much energy do we need to give the electron in order to do that (Give the value in both joules and electron volts).

The difference between the two energy levels will equal the amount of energy needed to move the electron. E1 = -2.18 x 10^-18 J E3 = E1/(2^2) = -2.18 x 10^-18 J/9 = -2.42 x 10^-19 J -2.18 x 10^-18 J = -21.8 x 10^-19 J 21.8 x 10^-19 - 2.42 x 10^-19 19.38 x 10^-19 J = 1.94 x 10^-18 J eV: E1 = -13.6 eV E3 = E1/(3^2) = -13.6 eV/9 = -1.51 eV 13.6 eV - 1.51 eV 12.09 eV

Say that we want to cause an electron to go from the first energy level (n=1) to the third energy level (n=3). How much energy do we need to give the electron in order to do that (Give the value in both joules and electron volts).

Insulate the container, so that there is no transfer of heat into or out of the system. Move the piston up and down extremely fast, to make sure no heat escapes the container.

Say that you have a cylindrical container full of a gas, a piston is at the top of the container, and the container is sitting on a heating plate. With this set up, how can we make an adiabatic process in a lab?

By having the piston move freely. This allows the volume of the chamber to increase and decrease in accordance with heat being added or removed, while maintaining a constant pressure.

Say that you have a cylindrical container full of a gas, a piston is at the top of the container, and the container is sitting on a heating plate. With this set up, how can we make an isobaric process in a lab?

Have the piston fixated in a certain position. This allows for the heat to increase and decrease the pressure in the system without actually changing the volume.

Say that you have a cylindrical container full of a gas, a piston is at the top of the container, and the container is sitting on a heating plate. With this set up, how can we make an isochoric process in a lab?

Either push or pull the piston very slowly. This way, the heat always has time to enter or exit the system accordingly, allowing for the temperature to remain constant.

Say that you have a cylindrical container full of a gas, a piston is at the top of the container, and the container is sitting on a heating plate. With this set up, how can we make an isothermal process in a lab?

Smallest E3>E2>E1

Since the quantities of energy associated with electrons at certain energy levels are negative, the highest energy level has the __________________ negative value of energy. E__ > E__ > E__

Poor Lattice arrangement

Solid ionic compounds are ___________________ conductors of electricity since the charged particles are rigidly set in place by the _______________________ of the crystalline solid.

s subshell d subshell d subshell

Some transition metals will demote an electron from the __________ subshell to the _____________ subshell in order to stabilize the ___________ subshell, such is the case with transition metals such as manganese and copper.

Valence electrons Electron Negative Electron Positive Electronegative Single bonds Electrons Single bonds Terminal Central Multiple bonds Central

Steps for determining the dot structure of a molecule: 1. Find the total number of ______________ ______________in the molecule. (Add a ______________ for every ______________ charge, and subtract a ________________ for every _______________ charge). 2. Choose the least __________________ atom to act as the central atom. 3. Draw _____________ ____________ between the central atom and the terminal atoms. Subtract 2 ____________ for each ____________ _____________ you form. 4. Assign leftover electrons to the ______________ atoms. 5. If necessary, assign remaining leftover electrons to ____________ atom. (If it does not have a full octet, create ____________ __________)(If ___________ atom has an octet or exceeds octet, you are done).

Longest Highest order Number Substituents Number Priority

Steps for naming an organic chemistry molecule: 1. Identify the _________________ carbon chain containing the ___________ ___________ functional group. 2. ___________ the chain 3. Name the ________________ 4. Assign a _____________ to each substituent based on their __________________. 5. Complete the name

Dot structure Electron clouds Electrons Electron clouds Lone pairs

Steps to finding the molecular geometry of a molecule: 1. Draw the ___________ ______________ to show valence electrons. 2. Count the number of ___________ __________ (pairs of ___________) surrounding the central atom 3. Predict the geometry of the ____________ ___________ around the central atom 4. Ignore any ___________ ___________ and predict the geometry of the molecule/ion

σ-bonds lone pairs σ lone

Steric number is calculated as the number of __________________ plus the number of ___________________. SN = #______ + #_______

Answer: D. Saying that substance B has a higher internal energy cannot explain the phenomenon because the internal energy is irrelevant; the heat involved in the process is related only to the specific heat, the heat of fusion, and the heat of vaporization. All of the other choices could explain the phenomenon. The heat required to melt the solid is determined by the heat of fusion, (C). The heat required to bring the liquid to its boiling point is determined by the specific heat, (A). The heat required to boil the liquid is determined by the heat of vaporization (B).

Substances A and B have the same freezing and boiling points. If solid samples of both substances are heated in the exact same way, substance A boils before substance B. Which of the following would not explain this phenomenon? A. Substance B has a higher specific heat B. Substance B has a higher heat of vaporization C. Substance B has a higher heat of fusion D. Substance B has a higher internal energy

Answer: B. Because the electron is moving into the n=1 shell, the only subshell available is the 1s subshell, which eliminates (C.) and (D.). There will be some energy change, however, as the electron must lose energy to return to the minimum-energy ground-state. That will require emitting radiation in the form of a photon.

Suppose an electron falls from n=4 to its ground state, n=1. Which of the following effects is most likely? A. A photon is absorbed B. A photon is emitted C. The electron moves into a p-orbital D. The electron moves into a d-orbital

Answer: B. Because methylene chloride is denser than brine (salt water), the organic layer will settle at the bottom of the funnel. Methylene chloride is nonpolar, so it cannot mix with brine, eliminating (C).

Suppose an extraction with methylene chloride (ρ = 1.33 g/mL) is performed, with the desired compound initially in brine (ρ = 1.04 g/mL). In a separatory funnel, which layer will be the organic layer? A. The top layer B. The bottom layer C. No layers are observed; methylene chloride and brine are miscible D. More information is needed to answer the question

ΔV = βVΔT β = 1.8 x 10^-4 K^-1 V = 1 mL ΔT = 275°C - (-25°C) = 300 K ΔV = 1.8 x 10^-4 K^-1 * 1 mL * 300 K ΔV = 0.054 mL

Suppose that a thermometer with 1 mL of mercury is taken from a freezer with a temperature of -25°C and placed near an oven at 275°C. If the coefficient of volume expansion of mercury is 1.8 x 10^-4 K^-1, by how much will the liquid expand?

Answer: A. The MCAT covers the topics in this chapter qualitatively more often than quantitatively. It is critical to be able to distinguish the fundamental principles that determine electron organization, which are usually known by the names of the scientists who discovered or postulated them. The Heisenberg uncertainty principle, (B.), refers to the inability to know the momentum and position of a single electron simultaneously. The Bohr model, (C.), was an early attempt to describe the behavior of the single electron in a hydrogen atom. The Rutherford model, (D.), described a dense, positively charged nucleus. The element shown here, phosphorous, is often used to demonstrate Hund's rule because it contains a half-filled p subshell. Hund's rule explains that electrons fill empty orbitals first before doubling up electrons in the same orbital.

Suppose that an atom fills its orbitals as shown. Such an electron configuration most clearly illustrates which of the following laws of atomic physics? A. Hund's rule B. Heisenberg uncertainty principle C. Bohr model D. Rutherford model

Step 1: Find molecular weight of both solute and solvent Cd(NO3)2 = 112 g + 2(14) g + 6(16) g = 236 g Cd(NO3)2 H2O = 16 g + 2 g = 18 g H2O Step 2: Find grams of solute and solvent in the solution Since the aqueous solution is a 12% (wt-wt) solution, 12% of the mass of the solution would be the solute. We have 100 grams of the solution, therefore 12% of the mass would be 12 grams. This would be the mass of the solute in the solution. The remaining mass would be the mass of the solvent. 100 g - 12 g = 88g. This would be the mass of the solvent. Cd(NO3)2 = 12 g H2O = 88 g Step 3: Find moles of each substance 12 g Cd(NO3)2 * 1 mol Cd(NO3)2 = 0.05 mol 236 g Cd(NO3)2 88 g H2O * 1 mol H2O = 4.89 mol 18 g H2O Step 4: Find mole fraction by dividing moles of substance of interest by the total moles of the solution 0.05 mol Cd(NO3)2/ (4.89 mol H2O + 0.05 mol Cd(NO3)2) = 0.01

Suppose we have a 12% (wt-wt) 100 gram aqueous solution of cadmium nitrate. What is the mole fraction of cadmium nitrate in the solution.

Strong acid + strong base Strong acid + weak base Weak acid + strong base Weak acid + weak base

The 4 kinds of neutralization reactions among acids and bases include: ____________ acid + ____________ base ___________ acid + _____________ base ___________ acid + _____________ base ___________ acid + ____________ base

Position Momentum Same time Position/Momentum Momentum/Position

The Heisenberg uncertainty principle is a principle of quantum mechanics that states that the ________________ and ________________ of a particle cannot be accurately measured at the ____________________. If you know the _____________________ of a particle, then you do not know the _________________ of the particle, and vice versa.

pH or pOH

The Henderson Hasselbalch equation is used to estimate the ________________ of a buffer solution.

pH = pKa + log([A-]/[HA]) pOH = pKb + log([B+]/[HB])

The Henderson Hasselbalch equation: pH = _______ + log(________/_________) pOH = _______ + log(_______/________)

Answer: C. Among the functional groups presented, carboxylic acids have the highest priority, and their compounds end with an -oic acid. (A.) denotes an alcohol, (B.) a ketone, and (D.) an alkyne, all of which have lower priorities than carboxylic acids. The MCAT does not test nomenclature of halides or ethers, but note that these must have lower priority than a carboxylic acid because they are less oxidized groups.

The IUPAC name for the following structure ends in what suffix? A. -ol B. -one C. -oic acid D. -yne

Answer: B. We know right away that (C.) and (D.) will be incorrect because a hydroxyl group is of higher priority than a methyl group. We also know that we will number the carbon chain so that the hydroxyl group receives the lowest possible position. Therefore, this molecule is 2-methyl-2-butanol, which starts with the prefix 2-methyl.

The IUPAC name for the following structure starts with what prefix? A. 3-methyl- B. 2-methyl- C. 2-hydroxy- D. 3-hydroxy-

AgI (s) <---> Ag+ (aq) + I- (aq) I 1 x 10^-5 0 C -x +x +x E 1 x 10^-5 + x x Ksp = [Ag+] * [I-] 8.5 x 10^-17 = (1 x 10^-5 + x) * x since Ksp < 1 x 10^-4, assume x<<1 8.5 x 10^-17 = 1 x 10^-5 * x 1 x 10^-5 1 x 10^-5 8.5 x 10^-12 = x x = [I-] [I-] = 8.5 x 10^-12 M

The Ksp of AgI in aqueous solution is 8.5 x 10^-17. If a 1 x 10^-5 M of AgNO3 is saturated with AgI, what will be the final concentration of the iodide ion?

Ba(OH)2 <---> Ba(2+) + 2OH- I 0 0 C -x +x +2x E x 2x IP = [Ba(2+)] * [2OH-]^2 IP = [x] * [2x]^2 Ba(2+) = 0.05 M IP = 0.05 * [2x]^2 If Ba(2+) = x, 2OH- = 2x, and Ba(2+) = 0.05, then 2OH- = 2(0.05) IP = 0.05 * [2 * 0.05]^2 IP = 0.05 * [0.1]^2 IP = 0.05 * 0.01 IP = 5 x 10^-4 M Ksp = 5.0 x 10^-3 IP < Ksp, means dissolution.

The Ksp of Ba(OH)2 is 5.0 x 10^-3. Assuming that barium hydroxide is the only salt added to form a solution, calculate the ion product of a solution with a concentration of Ba(2+) of 0.05 M. Then predict the behavior of the given solution (dissolution, equilibrium, precipitation).

Ba(OH)2 <---> Ba(2+) + 2OH- I 0 0 C -x +x +2x E x 2x IP = [Ba(2+)] * [2OH-]^2 IP = [x] * [2x]^2 Ba(2+) = 0.1 M IP = 0.1 * [2x]^2 If Ba(2+) = x, 2OH- = 2x, and Ba(2+) = 0.1, then 2OH- = 2(0.1) IP = 0.1 * [2 * 0.1]^2 IP = 0.1 * [0.2]^2 IP = 0.1 * 0.04 IP = 0.004 M Ksp = 5.0 x 10^-3 IP < Ksp, means dissolution.

The Ksp of Ba(OH)2 is 5.0 x 10^-3. Assuming that barium hydroxide is the only salt added to form a solution, calculate the ion product of a solution with a concentration of Ba(2+) of 0.1 M. Then predict the behavior of the given solution (dissolution, equilibrium, precipitation).

Ba(OH)2 <---> Ba(2+) + 2OH- I 0 0 C -x +x +2x E x 2x IP = [Ba(2+)] * [2OH-]^2 IP = [x] * [2x]^2 Ba(2+) = 0.5 M IP = 0.5 * [2x]^2 If Ba(2+) = x, 2OH- = 2x, and Ba(2+) = 0.5, then 2OH- = 2(0.5) IP = 0.5 * [2 * 0.5]^2 IP = 0.5 * [1]^2 IP = 0.5 M Ksp = 5.0 x 10^-3 IP > Ksp, means precipitate.

The Ksp of Ba(OH)2 is 5.0 x 10^-3. Assuming that barium hydroxide is the only salt added to form a solution, calculate the ion product of a solution with a concentration of Ba(2+) of 0.5 M. Then predict the behavior of the given solution (dissolution, equilibrium, precipitation).

octet rule Less than More than d-orbital odd number

The __________ _________ states that atoms lose, gain or share electrons in order to acquire a full set of eight valence electrons. Exceptions: -Elements that are stable with ________ ________ 8 electrons (such as hydrogen, helium, lithium, beryllium, and boron) -Elements that are able to have ___________ _________ 8 electrons due to having a _______________ (any element beyond period 2) -Molecules that have an ___________ ___________ of valence electrons (such as with NO molecules)

Bond order

The ____________ __________ of a molecule is the total number of bonds divided by the number of attaching atoms.

Lower Higher

The ____________ the pKa, the stronger the acid. The ____________ the pKb, the stronger the base.

Coordination number

The _____________ _____________ of a molecule is the number of atoms bonded to the central atom.

solubility precipitate saturation

The _____________ of a solute is a measure of the amount of solute that can be dissolved in a solvent at a certain temperature. A ____________ is an insoluble solid that separates from a solution; generally the result of mixing two or more solutions or of a temperature change. The _____________ of a solution is the maximum amount of solute that can be dissolved in a particular solvent at a given temperature.

endpoint

The _____________ of a titration is the point in which we start getting an excess of titrant in our titrand solution, and the indicator changes to its final color.

Greater Smaller

The _____________ the effective nuclear charge, the greater the ionization energy, the _______________ the effective nuclear charge, the smaller the ionization energy.

Stronger.

The __________________ the intermolecular forces of attraction, the more energy is required to break these forces, thus the higher the boiling and melting point will be.

Equivalence point

The ____________________ is the point in a titration in which the amount of titrant added is just enough to completely neutralize the titrand solution. (moles base = moles acid)

Azimuthal/Angular Quantum number Spin Quantum number Principle Quantum number Magnetic Quantum number

The ____________________ quantum number indicates the shape that an orbital will take. The ____________________ quantum number indicates the "spin" of an electron (though the electron is not actually spinning. The ___________________ quantum number indicates the main energy level that the electrons of an atom occupy. The __________________ quantum number indicates the orientation of an orbital around a nucleus.

Lyman series Balmer series Paschen series Brackett series

The ______________________ is a set of spectral lines that appear in the UV region when a hydrogen atom undergoes a transition from energy levels n>1 to n=1. The _____________________ is a set of spectral lines that appear in the visible light region when a hydrogen atom undergoes a transition from energy levels n>2 to n=2. The ______________________ is a set of spectral lines that appear in the infrared region when a hydrogen atom undergoes a transition from energy levels n>3 to n=3. The ___________________ is a set of spectral lines that appear in the infrared region when a hydrogen atom undergoes a transition from energy levels n>4 to n=4.

Half-equivalence points

The _______________________ of a polyvalent titration is found at the center of the buffer regions, where half of a given species has been protonated or deprotonated.

Solubility Product Constant (Ksp) Ion Product (IP)

The _________________________________ is the product of concentration of ions in a saturated solution. The ________________ is the product of concentration of ions in a solution that has not reached saturation.

Parent chain Hydrogens

The carbonyl carbon on aldehydes and ketones are not connected to any leaving groups; they are only connected to either the ______________________ or, in the case of aldehydes, ______________ atoms.

Increases Decreases

The closer the electron is to the nucleus, the greater the attractive force will be. Thus, less distance between the nucleus and valence electron ______________ ionization energy. The further the electron is to the nucleus, the weaker the attractive force will be. Thus, more distance between the nucleus and valence electron __________________ ionization energy.

concentration chemical identity

The colligative properties of a solution are the physical properties of a solution that depend on the __________________ of dissolved particles, but not on the _______________ _____________ of the particles.

Answer: B. Form- is a prefix shared by the common names of methanoic acid (formic acid) and methanal (formaldehyde).

The common names for the aldehydes and carboxylic acids that contain only one carbon start with what prefix? A. Para- B. Form- C. Meth- D. Acet-

Ions Ions Greater Ions Weaker

The electrical conductivity of aqueous solutions is determined by the presence and concentration of _______________ in the solution. The more ___________ that is present, the _______________ the electrical conductivity of the solution. The less _______________ that is present, the _________________ the electrical conductivity of the solution.

Ionization energy

The energy required to remove an electron from one mole of gaseous atoms to produce one mole of gaseous ions is known as the ________________ __________________ of an atom. X + energy -----> X+ + e-

Answer: C. CaS will cause the most negative contribution to ΔS°(soln) through hydration effects because the Ca(2+) and S(2-) ions have the highest charge density compared to the other ions. All of the other ions have charges of +1 or -1, whereas Ca(2+) and S(2-) each have charges with a magnitude of 2.

The entropy change when a solution forms can be expressed by the term ΔS°(soln). When water molecules become ordered around an ion as it dissolves, the ordering would be expected to make a negative contribution to ΔS°(soln). An ion that has more charge density will have a greater hydration effect, or ordering of water molecules. Based on this information, which of the following compounds will have the most negative contribution to ΔS°(soln). A. KCl B. LiF C. CaS D. NaCl

Answer: B. The entropy of a system can decrease as long as the entropy of its surroundings increases by at least as much. On the other hand, the entropy of an isolated system increases for all (irreversible) processes. This adheres to the second law of thermodynamics, which says that energy will be dispersed and entropy of the universe will remain constant or increase during all processes.

The entropy of a system can: A. never decrease. B. decrease when the entropy of the surroundings increases by at least as much. C. decrease when the system is isolated and the process is irreversible. D. decrease during an adiabatic reversible process.

Weak Weak Acid Base Base Acid

The equivalence point of a titration reaction between a ________ acid and _____________ base is dependent on the relative strength of the acid and base. If the ___________ is stronger than the _____________, pH<7 If the _____________ is stronger than the ______________, pH>7

Strong Strong

The equivalence point of a titration reaction between a ___________ acid and ____________ base would have a pH = 7

Strong Weak

The equivalence point of a titration reaction between a ___________ acid and _____________ base would have a pH<7.

Answer: B. The electron moves from a higher energy level to a lower energy level; this can only occur if the extra energy is dissipated through the emission of a photon. If the electron moved from B to A, it would absorb a photon and increase the atom's total energy level; however, the opposite is occurring, so (A.) and (D.) can be eliminated. The work function is the amount of energy required to eject an electron from a material; when moving from A to B, the electrical potential energy of the atom decreases, meaning that more energy will be required to free the electron from the atom, eliminating (C.).

The figure shown illustrates an electron with initial energy of -10 eV moving from point A to point B. What change accompanies the movement of the electron? A. Absorption of a photon B. Emission of a photon C. Decrease in the atom's work function D. Increase in the atom's total energy

balanced

The first step in any stoichiometric problem is to always ensure that the chemical reaction you are dealing with is _____________________.

Buffer region

The flat portion before the curve of a titration curve is known as the __________________.

Ionic compound

The following are all characteristics of a ____________ compound: -High melting point and boiling point due to electrostatic attractions -Very soluble in water/polar solvents -Good conductors of heat and electricity in molten state -Solid state is in a crystal lattice arrangement to minimize repulsive forces -Large electronegativity differences between atoms

Non-polar covalent bonds

The following are characteristics of _______________________ bonds: -Often occurs between atoms that are of the same element, or atoms of different elements with a low enough electronegativity difference. -Electronegativity difference between bonded atoms are small (<0.5 Pauling Scale) -Electrons are shared between atoms equally

Oxidizing Agents General Chemistry

The following are common ____________ agents in ________________ chemistry: -O2 -H2O2 -H2SO4 -Halogens (F2, Cl2, Br2, I2) -HNO3 -NaClO

Reducing Agents General Chemistry

The following are common _____________ agents in ______________ chemistry: -CO -Pure Metals (Fe, Cu, etc.) -B2H6 -C

Reducing Agents Organic Chemistry

The following are common ______________ agents in __________________ chemistry: -Hydrazine -Lindlar's Catalyst -NaBH4 -Zn(Hg) -LiAlH4

Reducing Agents Biochemistry

The following are common _______________ agents in ________________ chemistry: -NADH -FADH2

Oxidizing Agents Biochemistry

The following are common _______________ agents in ___________________ chemistry: -NAD+ -FADH

Oxidizing Agents Organic Chemistry

The following are common __________________ agents in ______________ chemistry: -CrO3 -Pyridinium Chlorochromate -KMnO4 -NaCr2O7

Buffers

The following are common examples of ______________: - Acetic acid & sodium acetate CH3COOH + CH3COO-Na+ - Ammonia & ammonium chlorate NH3 + NH4+Cl-

Answer: D. A strong oxidizing agent will be easily reduced, meaning that it will have a tendency to gain electrons. Atoms usually gain electrons if they are one or two electrons away from filling up a valence shell. (A.) has a full 4s-orbital, meaning that it can only gain an electron if it gains an entire subshell. (B.) has a stable, half-full 3d-orbital, so it is unlikely to pick up electrons unless it can gain five. (C.) has only a single electron in the outer shell, which is more likely lost upon ionization. (D.) would fill up its 4p-orbital by gaining one electron, so it is easily reduced.

The following electronic configurations represent elements in their neutral form. Which element is the strongest oxidizing agent? A. 1s(2)2s(2)2p(6)3s(2)3p(6)4s(2) B. 1s(2)2s(2)2p(6)3s(2)3p(6)4s(2)3d(5) C. 1s(2)2s(2)2p(6)3s(2)3p(6)4s(2)3d(10)4p(1) D. 1s(2)2s(2)2p(6)3s(2)3p(6)4s(2)3d(10)4p(5)

Energy Wavelength Energy of the photon Planck's constant (6.626 x 10^-34 J*s/4.14 x 10^-15 eV*s) Speed of light (3 x 10^8 m/s) Wavelength of the photon

The following equation is used to determine the _________________ of a photon when we are given ________________________. E(photon) = h*(c/λ) E(photon) = _____________________ h = _____________________ c = ______________________ λ = ________________________

Energy Frequency Energy of the photon Planck's constant (6.626 x 10^-34 J*s/4.14 x 10^-15 eV*s) frequency of the photon

The following equation is used to determine the __________________ of a photon when we are given ___________________: E(photon) = hν E(photon) = ______________________ h = _______________________ ν = __________________________

Energy Energy level E(n) = The energy of an electron at an energy level of n E1 = Rydberg unit of energy (-2.18 x 10^-18 J) n = the energy level of the valence electron

The following equation is used to determine the __________________ of an electron at a certain ____________________: E(n) = E1/(n^2) E(n) = _______________________ E1 = _________________________ n = __________________________

Heisenberg uncertainty principle Δx = uncertainty in position Δp = uncertainty in momentum h = Planck's constant (6.626 x 10^-34 J*s/4.14 x 10^-15 eV*s)

The following equation is used to determine the ________________________________: Δx * Δp > h/4π Δx = ______________________ Δp = _______________________ h = ____________________________

Kinetic energy Kinetic energy Mass Velocity

The following equation is used to find _____________________________: KE = (1/2)mv^2 KE = ________________________ m = ______________ v = _________________

Atomic radii r(n) = atomic radius of an atom at a particular energy level n = energy level the electron is at r1 = atomic radius of the smallest orbit in the Bohr model (5.3 x 10^-11 m)

The following equation is used to find the _________________________ of an atom using the Bohr model: r(n) = n^2 * r1 r(n) = _____________________ n = ______________________ r1 = __________________________

Kinetic energy of the photoelectron ejected Energy of the photon Work function of the metal

The following equation is used to find the __________________________________ of a photoelectron: KE(photoelectron) = E(photon) - E° KE(photoelectron) = ______________________________ E(photon) = __________________________ E° = _________________________________

Answer: D. The solubility of AgBr can be determined using the Ksp value given in the equation. Some amount of AgBr will dissolve; this is the molar solubility x for these conditions. When AgBr dissociates, there will be x amount of silver(I) formed and x amount of bromide - which is added to the 0.0010 M Br- already present from NaBr. AgBr <---> Ag+ + Br- Ksp = [Ag+][Br-] 5.35 x 10^-13 = (x)(0.0010 M + x) Given the Ksp of 5.4 x 10^-13, x will be negligible compared to 0.0010 M. Thus, the math can be simplified to: 5.35 x 10^-13 = (x)(0.0010) x = 5.35 x 10^-13 = 5.35 x 10^-10 0.0010 Therefore, x, the molar solubility, is 5.35 x 10^-10, which looks like (C.). However, the units of the answer choices are g/L, not mol/L, and the result must be multiplied by the molar mass (187.8 g/mol): 5.35 x 10^-10 mol/L x 187.8 g/mol = 1 x 10^-7 which is close to (D.). Note that a very accurate approximation was reached by rounding down the first number and rounding up the second, balancing the error.

The following equilibrium exists when AgBr (Ksp = 5.35 x 10^-13) is in solution: AgBr (s) <---> Ag+ (aq) + Br- (aq) What is the solubility of AgBr in a solution of 0.0010 M NaBr? A. 5.35 x 10^-13 g/L B. 1.04 x 10^-12 g/L C. 5.35 x 10^-10 g/L D. 1.04 x 10^-7 g/L

Linear momentum p = linear momentum m = mass v = velocity

The following is an equation used for ________________________: p=mv p = _____________________ m = _____________________ v = ____________________

Rydberg unit of energy (E1) in joules

The following is the value of ___________________: -2.18 x 10^-18 J

The speed of light (c)

The following is the value of ____________________: 3 x 10^8 m/s

Rydberg's constant

The following is the value of _____________________: 1.09 x 10^7 1/m

Rydberg unit of energy (E1) in electron volts

The following is the value of _______________________: -13.6 eV

Planck's constant in J*s (h)

The following is the value of __________________________: 6.626 x 10^-34 J*s

The atomic radius of the smallest orbit in the Bohr model (r1)

The following is the value of ____________________________: 5.3 x 10^-11 m

Planck's constant in eV*s (h)

The following is the value of ______________________________: 4.14 x 10^-15 eV*s

Answer: A. These compounds are nonsuperimposable mirror images. To make analysis a bit easier, we can rotate the second structure 180° and compare it to the third structure. The two structures show to have opposite stereochemistry at every chiral center, meaning that they are enantiomers. (B.) is incorrect because diastereomers are stereoisomers that are not mirror images of each other. (C.) is incorrect because meso compounds must contain a plane of symmetry, which neither of these molecules has. (D.) is incorrect because structural isomers are compounds with the same molecular formula but different atomic connections. The connectivity in these two molecules is the same, which means that they are stereoisomers, not structural isomers.

The following molecules are considered to be: A. Enantiomers B. Diastereomers C. Meso compounds D. Structural isomers

Answer: C. The relative configuration is retained because the bonds of the stereocenter are not broken; thus the position of groups around the chiral carbon are maintained. The absolute configuration is also retained because both the reactant and product are (R).

The following reaction results in: A. retention of relative configuration and a change in the absolute configuration B. a change in the relative and absolute configurations C. retention of the relative and absolute configurations D. retention of the absolute configuration and a change in the relative configuration.

Chair conformation Cyclohexane

The following shows the ____________________ conformation of _______________________.

Twist-boat conformation Cyclohexane

The following shows the ____________________ conformation of _________________________.

Boat conformation Cyclohexane

The following shows the _____________________ conformation of __________________________

λ = wavelength of photon R = Rydberg's constant (1.097 x 10^7 1/m) n(f) = final (ground) energy state n(i) = initial (excited) energy state

The folowing equation is used to determine the wavelength of the photon that is emitted when an electron transitions from a higher energy state to a lower energy state: 1/λ = R * [(1/(n(f)^2)) - (1/(n(i)^2))] λ = ___________________________ R = _____________________________ n(f) = __________________________ n(i) = ____________________________

Answer: C. The four bonds point to the vertices of a tetrahedron, which means that the angle between two bonds is 109.5°, a characteristic of sp3 orbitals. Hence, the carbon atom of CH4 is sp3-hybridized.

The four C-H bonds of CH4 point towards the vertecies of a tetrahedron. This indicates that the hybridization of the carbon atom in methane is: A. sp B. sp2 C. sp3 D. sp3d

Reflective Heat Electricity

The freely moving electrons in metals are responsible for their _______________ properties, and their ability to effectively conduct __________ and ___________.

Answer: B. The purpose of a buffer is to resist changes in the pH of a reaction. Buffers are not generally used to affect the kinetics of a reaction, so (C.) and (D.) are incorrect. (A.) is correct only in specific circumstances where the pH of the buffer solution itself is neutral. Many natural buffers systems maintain pH in acidic or basic ranges.

The function of a buffer is to: A. maintain a neutral pH B. resist changes in pH when small amounts of acid or bases are added C. slow down reactions between acids and bases D. speed up reactions between acids and bases

Increases Greater Decreased Increase

The further right you go on the periodic table, the nuclear charge of the elements ________________ by 1, which causes a ____________ attractive force of the valence electron to the nucleus, which also leads to a ______________ distance between the valence and nucleus, resulting in an _____________ in ionization energy.

Answer: B. Each of these is the mobile phase of the system, in which the solutes are dissolved and move. The stationary phase in gas chromatography is usually a crushed metal or polymer; the stationary phase in paper chromatography is paper.

The gas eluent in gas chromatography and the liquid eluent in paper chromatography are examples of what component of these systems? A. Stationary phase B. Mobile phase C. Column D. Fraction

Mass

The gram equivalent weight of an acid/base is the ______________ of the acid/base that produces 1 acid/base equivalent.

Bond strength Bond length

The greater the bond energy, the stronger the ________________, and the smaller the ________________.

Higher Heteroatoms Hydrogen

The greater the oxidation state the carbon has, the ____________ the priority it has in a molecule. Oxidation state increases with more bonds to ____________________. Oxidation state decreases with more bonds to _____________________.

Stronger Stronger

The higher the Ka, the ____________ the acid. The higher the Kb, the _____________ the base.

Oxidized Suffix

The highest priority functional group is the functional group with the most ________________ carbon. This functional group will provide the _____________ of the molecule's name.

Answer: D. The carbon and nitrogen atoms are connected by a triple bond in CN- (:C==N:-). A triple-bonded atom is sp hybridized; one s-orbital hybridizes with one p-orbital to form two sp-hybridized orbitals. The two remaining unhybridized p-orbitals take part in the formation of two π bonds.

The hybridizations of the carbon and nitrogen atoms in CN- are: A. sp3 and sp3, respectively B. sp3 and sp, respectively C. sp and sp3, respectively D. sp and sp, respectively

moles molar coefficient lowest value

The ideal way of finding the limiting reagent and excess reagent is to find the number of __________ in each reactant and divide by the _________________________ in the balanced reaction. The substance with the _______________________ is the limiting reagent.

Ionic

The intermolecular forces present in a ___________ compound include London dispersion forces and ion-ion attraction between ions.

Polar covalent with hydrogen bonding

The intermolecular forces present in a __________________ compound includes London dispersion forces and hydrogen bonding.

Nonpolar Covalent

The intermolecular forces present in a ___________________ compound includes only London dispersion forces.

s p s d s f

The key feature of representative elements is that they have their valence electrons in the orbital of either _____ or ______ subshells. The key feature of nonrepresentative elements is that they have their valence electrons in the orbital of either ________ and ________ subshells (for transition metals) or _______ and _______ subshells (for lanthanide and actinide elements).

reactants products

The law of the conservation of charge say for a chemical reaction, the total charge of the _______________ consumed must equal total charge of _____________ generated.

Reactants Products

The law of the conservation of mass says for a chemical reaction, the mass of _________________ consumed must equal to mass of ____________ generated.

Moles of substance of interest Total moles of the solution Moles of substance of interest Total moles of the solution

The mole fraction is equal to the _____________________ divided by _________________________. Mole fraction = ________________/________________

stronger

The more electrons a molecule has, the _________________ the London dispersion forces will be.

Stoichiometric coefficients

The numbers placed in front of each compound to indicate the relative number of moles of a given species in a reaction are called _________________ __________________.

Mercury

The one metal that is not solid at room temperature is ______________, which is liquid at room temperature.

Answer: B. The carbon bond in hydrogen cyanide (H -- C == N:) is triple-bonded, and because triple bonds require two unhybridized p-orbitals, the carbon must be sp-hybridized; sp-hybridized orbitals have 50% s character and 50% p character.

The p character of the bonds formed by the carbon atom in HCN is: A. 25% B. 50% C. 67% D. 75%

Center of the buffer region

The pKa of a titration curve is found in the _____________ of the _______________________ of the curve.

Reduction reaction Oxidation reaction

The part of a redox reaction where a chemical species gains an electron is known as the _______________ reaction. The part of the redox reaction where a chemical species loses electrons is known as the _______________ reaction.

Most stable ---> Least stable 1. A. (chair conformation) 2. C. (twist boat conformation) 3. B. (boat conformation)

The picture shows the three conformations of cyclohexane. Order the three conformations from most stable/lowest energy to least stable/highest energy.

Non-polarized light Plane-polarized light Non-polarized light Plane-polarized light A solution of a compound Plane-polarized light Compound Plane-polarized light Light

The process of determining optical activity of a stereoisomer: 1. ____________________________ passes through polarized filter 2. ________________________ is produced by _______________________ passing through the filter 3. _________________________ passes through a polarimeter tube containing a ______________________________. ___________________________ rotates a certain amount of degrees when it hits the __________________. 4. ____________________________ hits the analyzer, which we rotate to see the angle the ___________ is now.

Answer: D. The first step will most likely be endothermic because energy is required to break molecules apart. The second step is also endothermic because the intermolecular forces in the solvent must be overcome to allow incorporation of solute particles. The third step will most likely be exothermic because polar water molecules will interact with the dissolved ions, creating a stable solution and releasing energy.

The process of formation of a salt solution can be better understood by breaking the process into three steps: 1. Breaking the solute into its individual components 2. Making room for the solute in the solvent by overcoming intermolecular forces in the solvent 3. Allowing solute-solvent interactions to occur to form the solution Which of the following correctly lists the enthalpy of changes for these three steps, respectively? A. Endothermic, exothermic, endothermic B. Exothermic, endothermic, endothermic C. Exothermic, exothermic, endothermic D. Endothermic, endothermic, exothermic

Answer: C. Electronegativity describes how strong an attraction an element will have for electrons in a bond. A nucleus with a larger effective nuclear charge will have a higher electronegativity; Z(eff) increases toward the right side of a period. A stronger nuclear pull will also lead to increased first ionization energy, as the forces make it more difficult to remove an electron. The vertical arrow can be explained by the size of the atoms. As sized decreases, the positive charge becomes more effective at attracting electrons in a chemical bond (higher electronegativity), and the energy required to remove an electron (ionization energy) increases.

The properties of atoms can be predicted, to some extent, by their location within the periodic table. Which property or properties increase in the direction of the arrows shown? I. Electronegativity II. Atomic radius III. First ionization energy A. I only B. I and II only C. I and III only D. II and III only

0 Charge +1 +2 -1 +1 -1 -2 -1 +2 0 Charge

The rules of oxidation numbers: 1. The oxidation number of a free element (N2, S8, He, etc) is ____. 2. The oxidation number for a monoatomic ion (Na+, Cl-, Fe(3+), etc.) is equal to the __________ of the ion. 3. The oxidation number for each Group 1 element (Li, Na, K, etc.) in a compound is __________. 4. The oxidation number for each Group 2 element (Be, Mg, Ca, etc.) in a compound is _______. 5. The oxidation number for each Group 17 element (F, Cl, Br, etc.) in a compound is _________. 6. The oxidation number of a hydrogen atom is usually ________, but in the presence of less electronegative elements in a compound (such as Group 1 and Group 2 elements) its oxidation number is ________. 7. In most compounds, oxygen has an oxidation number of ________. Exceptions to this rule is if oxygen is part of a peroxide (where each oxygen has an oxidation number of _______), and if it is in a compound with a more electronegative element (such as OF2, where its oxidation number will be _______). 8. The sum of oxidation numbers of all atoms in a neutral compound will equal ______. The sum of oxidation numbers of atoms in a polyatomic ion (such as SO4(2-)) is equal to the __________ of the ion.

Answer: A. Dissolution is governed by enthalpy and entropy, which are related by the equation ΔG°(soln) = ΔH°(soln) - TΔS°(soln). The cooling of the solution indicates that heat is used up in this bond breaking reaction. In other words, dissolution is endothermic, and ΔH is positive. The reaction is occurring spontaneously, so ΔG must be negative. The only way that a positive ΔH can result in a negative ΔG is if entropy, ΔS, is a large, positive value as in (A.). Conceptually, that means that the only way the solid can dissolve is if the increase in entropy is great enough to overcome the increase in enthalpy. (B.) is incorrect because it is clearly stated in the question stem that KCl dissolves; further, all salts of Group 1 metals are soluble. (C.) is incorrect because ΔS°(soln) must be positive in order for KCl to dissolve. Finally, (D.) is incorrect because solute dissolution would cause the boiling point to elevate, not depress. It is also not a piece of evidence that could be found by simply observing the beaker's temperature change.

The salt KCl is dissolved in a beaker. To an observer holding the beaker, the solution begins to feel colder as the KCl dissolves. From this observation, one could conclude that: A. ΔS°(soln) is large enough to overcome the unfavorable ΔH°(soln) B. KCl is mostly insoluble in water C. ΔS°(soln) must be negative when KCl dissolves D. boiling point depression will occur in this solution

Molecular formula Empirical formula Structural formula

The three ways to represent a molecule: __________________________ __________________________ _________________________

indicator pH meter.

The two common ways of determining an acid-base titration include: -Estimate by using a _____________ -Graphing the pH of the titrand as a function of the amount of titrant added using a _________________

Inversely proportional Increases Decreases

The uncertainties of momentum and position of a particle in the Heisenberg uncertainty principle are ___________________________ to each other. So, if the value of one of the uncertainties _________________, the value of the the other uncertainty _____________________.

Joules (J)

The unit of measurement __________ is also equal to kg*(m^2/s^2)

Joules: E= -E1[(1/(n(i)^2) - 1/(n(f)^2)) E = -2.18 x 10^-18 * [1/(2^2) - 1/(4^2)] = -4.09 x 10^-19 J Electron volts: E = -13.6 * [1/4 - 1/16] = -2.55 eV In both cases, the value is negative, indicating that energy is absorbed. This is consistent with an electron moving from a lower to a higher shell.

The valence electron in a lithium atom jumps from energy level n=2 to n=4. What is the energy of this transition in joules? In eV? (Note: E1 = 2.18 x 10^-18 J = 13.6 eV)

Positive Negative Positive

The value for ionization energy is always ___________, the value for electron affinity is always _____________. (____________ electron affinities are considered to be inert).

Magnetic quantum number Azimuthal/Angular quantum number Principle quantum number Spin quantum number

The value of ___________________________ quantum number are dependent on the value of l. It equals any integral value that goes from -l to +l. The value of ___________________________ quantum number are dependent on the value of n. Values will be a range between 0 to n-1. The value of ____________________________ quantum number is a positive integer, meaning the value of it will equal to 1, 2, 3, etc. The value of ____________________________ quantum number are either +1/2 or -1/2.

3 Carboxyl group Amino group Side chain

There are ________ equivalence points in the titration of an acidic/basic amino acid. One corresponds to the titration of the ____________________. One corresponds to the titration of the ____________________. One corresponds to the titration of the ___________________.

π σ π σ interconverted

Though double bonds are stronger than single bonds overall, ______ bonds are weaker than __________ bonds. Therefore, it is possible to break only the _____ bond of a double bond and leave the __________ bond intact. This happens alot in organic chemistry, such as with cis-trans isomers being ____________________ between conformations.

decreases greater decrease

Through the removal of the first electron, electron shielding ______________, causing the valence electron to feel a ___________ attractive force to the nucleus than before. This ______________ in electron shielding causes a massive increase in ionization energy to remove a second electron.

Cyclopentane Envelope conformation

To reduce strain on itself, ____________________ takes on a _________________ conformation, as shown here.

False. Small changes, such as protonation and deprotonation, change in oxidation state or bond order, and others may cause dramatic changes in light absorption in a material.

True or False: Small changes in chemical structure only minimally impact light absorption and emission patterns.

Answer: D. When atomic orbitals combine, they form molecular orbitals. When two atomic orbitals with the same sign are added head-to-head or tail-to-tail, they form bonding molecular orbitals. When two atomic orbitals with opposite signs are added head-to-head or tail-to-tail, they form the antibonding molecular orbitals. Atomic orbitals can also hybridize, forming sp3, sp2, or sp orbitals.

Two atomic orbitals may combine to form: I. A bonding molecular orbital II. An antibonding molecular orbital III. Hybridized orbitals A. I only B. III only C. I and II only D. I, II, and III

covalent-nonpolar bond covalent-polar bond ionic bond

Two atoms of very similar or same electronegativity that there is very little or no difference in electronegativity are able to form a ________________________ bond. Two atoms that have somewhat differing electronegativities that there is a significant difference in electronegativity are able to form a ______________________ bond. Two atoms that have vastly differing electronegativities that there is a large difference in electronegativity are able to form a __________________________ bond.

Answer: B. Benzene and toluene are both organic liquids and have very similar properties. They are both nonpolar and are almost exactly the same size. Raoult's law states that ideal solution behavior is observed when solute-solute, solvent-solvent, and solute-solvent interactions are all very similar. Therefore, benzene and toluene in the solution will be predicted to behave as a nearly identical solution.

Two organic liquids, pictured in the figure, are combined to form a solution. Based on their structures, will the solution closely obey Raoult's Law? A. Yes; the liquids differ due to the additional methyl group on toluene and, therefore, will not deviate from Raoult's Law. B. Yes, the liquids are very similar and, therefore, will not deviate from Raoult's Law. C. No, the liquids differ due to the additional methyl group on toluene and, therefore, will deviate from Raoult's Law. D. No, the liquids both contain benzene rings, which will interact with each other and cause deviation from Raoult's Law.

Answer: D. The photoelectric effect occurs when a photon of sufficiently high energy strikes an atom with a sufficiently low work function. This means that a photon with higher energy is more likely to produce the effect. Because ultraviolet light has a higher frequency and lower wavelength than visible light, it also carries more energy according to the equation E=hν. All light travels at the speed of light, eliminating (B.). As mentioned earlier, ultraviolet light has a shorter wavelength than visible light, eliminating (A.). The visibility of a wave plays no role in its ability to cause the photoelectric effect, eliminating (C.).

Ultraviolet light is more likely to induce a current in a metal than visible light. This is because photons of ultraviolet light: A. have a longer wavelength B. have a higher velocity C. are not visible D. have a higher energy

Endothermic Exothermic

Unlike most dissolutions, dissolutions of gases into liquids are not ________________, they are ________________.

Answer: A. This is a question best answered by dimensional analysis. Keeping in mind that molar mass is measured in grams of a substance per moles of that substance, only (A) comes out with the units of grams of oxygen. (B) has the units of grams per mole of oxygen, not grams per oxygen. (C) has the units of moles per gram of oxygen. (D) has the units of mol^2 per grams of oxygen.

Using a given mass of KClO3, how would one calculate the mass of oxygen produced in the following reaction, assuming it goes to completion? 2KClO3 ---> 2KCl + 3O2 A. (g KClO3 consumed) (3 moles O2) (molar mass O2) (molar mass KClO3) (2 moles KClO3) B. (g KClO3 consumed) (molar mass O2) (molar mass KClO3) (2 moles KClO3) C. (molar mass KClO3) (2 moles KClO3) (g KClO3 consumed) (molar mass O2) D. (g KClO3 consumed) (3 moles O2) (molar mass KClO3) (2 moles KClO3) (molar mass O2)

Steric number

We can find the hybridization of an atom by finding the _______________________ of it.

Process functions are thermodynamic properties that describes the path taken to get from one state to another. Heat (Q) Work (W)

What are process functions? List the different types of process functions.

-Melting point -Boiling point -Solubility -Odor -Color -Density

What are some of the physical properties of a molecule?

Chiral column chromatography Chiral gas chromatography

What are some separation methods that are useful for separating enantiomers?

Linear thermal expansion Volumetric thermal expansion

What are some ways in which heat can alter the physical properties of matter.

State functions are thermodynamic properties of a system that are a function of only the current equilibrium state of a system. Pressure (P) Density (ρ) Temperature (T) Volume (V) Enthalpy (H) Internal energy (U) Gibbs free energy (G) Entropy (S)

What are state functions? List the different types of state functions.

-Principle quantum number (n) -Azimuthal/Angular quantum number (l) -Magnetic quantum number (ml) -Spin quantum number (ms)

What are the 4 quantum numbers of an atom?

Nonmetals

What are the brown colored elements in the periodic table classified as?

CuBr (s) <-----> Cu+ (aq) + Br- (aq) I 0 0 C -x +x +x E x x Ksp = [Cu+] * [Br-] 6.27 x 10^-9 = x^2 sqrt(6.27 x 10^-9) = sqrt(x^2) x = 7.91 x 10^-5 M x = [Cu+], [Br-], and [CuBr] Molar solubility of [Cu+], [Br-], and [CuBr] = 7.91 x 10^-5 M 3g CuBr/143.4 g/mol CuBr = 0.02 moles 0.02 moles/1 L = 0.02 M 0.02 M = Molar Solubility from IP 7.91 x 10^-5 M = Molar Solubility from Ksp 0.02 M > 7.91 x 10^-5 M IP > Ksp The solution is super-saturated

What are the concentrations of each of the ions in a saturated solution of CuBr, given that the Ksp of CuBr is 6.27 x 10^-9 at 25°C? If 3 g CuBr is dissolved in water to make 1 L of solution at 25°C, would the solution be saturated, unsaturated, or supersaturated?

Assume we have 100 g of carbohydrate: C: 40.9g * 1 mol = 3.4 mol 12 g H: 4.58 g * 1 mol = 4.6 mol 1 g O: 54.52 g * 1 mol = 3.4 mol 16 g Find simplest whole number ratio by dividing the number of moles for each compound by the smallest number out of all obtained in the previous step: 3.4/3.4 = 1 4.6/3.4 = 1.33 C = 1 H = 1.33 O = 1 Convert numbers obtained into whole numbers by multiplying them by an integer value: 1.33 x 3 = 4 1 x 3 = 3 C = 3 H = 4 O = 3 C3H4O3

What are the empirical and molecular formulas of a carbohydrate that contains 40.9% carbon, 4.58% hydrogen, and 54.52% oxygen and has a molar mass of 264 g/mol?

Nonrepresentative elements

What are the green and blue colored elements in the periodic table known as?

Metalloids

What are the green colored elements in the periodic table classified as?

Alkali metals

What are the highlighted elements in the periodic table classified as?

Alkaline earth metals

What are the highlighted elements in the periodic table classified as?

Halogens

What are the highlighted elements in the periodic table classified as?

Noble gases

What are the highlighted elements in the periodic table classified as?

Transition metals

What are the highlighted elements in the periodic table classified as?

Chalcogens

What are the purple-highlighted elements in the periodic table classified as?

At least one of the two liquids has a boiling point that is over 150°C. The difference in boiling points between the two liquids is less than 25°C.

What are the requirements needed for fractional vacuum distillation to be used?

The two liquids have a boiling point that is under 150°C. The difference in boiling points between the two liquids is at least 25°C.

What are the requirements needed for simple distillation to be used?

At least one of the two liquids has a boiling point that is over 150°C. The difference in boiling points between the two liquids is at least 25°C.

What are the requirements needed for simple vacuum distillation to be used?

.. (-) : O : I I N / \\ / \\ : O : : O : .. .. (-) : O : I I N // \ // \ : O : : O : .. : O : (-) II II N / \ / \ : O : : O : .. ..

What are the resonance structures of NO3-?

angle strain, torsional strain, and nonbonded strain

What are the three factors that determines the stability of the conformation of cycloalkanes?

-Conduction -Convection -Radiation

What are the three methods of which heat can transfer energy?

Ion-exchange: Column is given a charge, which attracts molecules with the opposite charge. Size-exclusion: Small pores are used; smaller molecules are trapped, while larger molecules pass through the column. Affinity: Specific receptors or antibodies can trap the target in the column; the target must then be washed out using other solutions.

What are the three specialized types of column chromatography? What does each use in order to separate the sample?

Absorption Solubility

What are the two things that determines affinity in chromatography?

By adding or removing heat from the system By doing work on the system or having work be done by the system

What are the two ways to change the internal energy of a system?

Representative elements

What are the yellow colored elements in the periodic table known as?

Fluorescence is a special stepwise photon emission in which an excited electron returns to the ground state through one or more intermediate excited states. Each energy transition releases a photon of light. With smaller energy transitions than the initial energy absorbed, these materials can release photons of light in the visible range.

What causes fluoresence?

If the molecule and its mirror image are superimposable (they overlap).

What characteristics makes a molecule achiral?

If the molecule and its mirror image are non-superimposable.

What characteristics makes a molecule chiral?

The energy differences between ground-state electrons and higher-level electron orbits determine the frequencies of light a particular material absorbs (its absorption spectrum).

What determines the absorption spectrum of a single atom?

Answer: C. Atomic radius is determined by multiple factors. Of the choices given, the number of valence electrons does have an impact on the atomic radius. As one moves across a period (row), protons and valence electrons are added, and the electrons are more strongly attracted to the central protons. This attraction tightens the atom, shrinking the atomic radius. The number of electron shells is also significant, as demonstrated by the trend when moving down a group (column). As the electron shells are added that separate the positively charged nucleus from the outermost electrons, the electrostatic forces are weakened, and the atomic radius increases. The number of neutrons is irrelevant because it does not impact these attractive forces.

What determines the length of an element's atomic radius? I. The number of valence electrons II. The number of electron shells III. The number of neutrons in the nucleus A. I only B. II only C. I and II only D. I, II, and III

Vacuum distillation uses a vacuum adaptor to lower ambient pressure in the system. Fractional distillation uses a fractionating column which allows for further separation of two liquids with similar boiling points. It essentially allows for the distillation process to be done multiple times.

What differentiates vacuum distillation from simple distillation? What differentiates fractional distillation from simple distillation?

All halides are water soluble except for fluorides and halides formed with Ag+ (silver ion), Pb(2+) (lead ion), or Hg2(2+) (mercury ion)

What do the rules of solubility say in regards to halides?

All metal oxides are insoluble except for those formed with alkali metals, NH4+ (ammonium), Ca(2+) (calcium ion), Sr(2+) (strontium ion), or Ba(2+) (barium ion).

What do the rules of solubility say in regards to metal oxides?

All salts containing nitrate (NO3-) anions and acetate (CH3COO-) anions are water soluble.

What do the rules of solubility say in regards to salts containing nitrate (NO3-) and acetate (CH3COO-)?

All salts containing SO4(2-) are water soluble except for those formed with Ca(2+) (calcium ion), Sr(2+) (strontium ion), Ba(2+) (barium ion), Pb(2+) (lead ion).

What do the rules of solubility say in regards to sulfates?

There are more microstates for a disordered macrostate than there are microstates for an ordered macrostate. (There are more possibilities for microstates in a macrostate where energy is mixed than there are possibilities for microstates in a macrostate where energy is separated to one side over the other).

What does entropy say about the macrostates and microstates of a system?

It means that the enantiomer is levorotatory.

What does it mean if an enantiomer is labeled with a (-) on it?

It refers to how polar substances dissolves best in polar solvents, while nonpolar substances dissolves best in nonpolar solvents.

What does the concept of like dissolves like mean?

The entropy of the universe is constantly increasing. ΔS(universe) = ΔS(system) + ΔS(surroundings) > 0

What does the second law of thermodynamics claim about the entropy of the universe?

The threshold frequency depends on the chemical composition of a material (that is, the identity of the metal).

What does the threshold frequency depend upon?

The accumulation of moving electrons creates a current during the photoelectric effect.

What electrical phenomenon results from the application of the photoelectric effect?

Acyl halide

What functional group is being shown here?

Alcohol/Hydroxyl

What functional group is being shown here?

Alkyne

What functional group is being shown here?

Amide

What functional group is being shown here?

Amine

What functional group is being shown here?

Anhydride

What functional group is being shown here?

Aromatic ring/Benzene

What functional group is being shown here?

Ester

What functional group is being shown here?

Ether

What functional group is being shown here?

Ketone

What functional group is being shown here?

Sulfide

What functional group is being shown here?

Thiol

What functional group is being shown here?

The common ion will lead to an increase in our products, causing our equilibrium to shift in favor of the reactants, which will cause us to have more of our solute at equilibrium, and decrease the solubility of our solute.

What happens when a solute is placed in a solvent that shares a common ion?

There will be no net flow of heat between the two objects, since they are in thermodynamic equilibrium.

What happens when two objects at the same temperature make thermal contact with one another.

The ionic attraction between Cl- and Ag+ will be greater than the ion-dipole interactions between the water molecules and the ions, which causes the Cl- and Ag+ ions to form an ionic bond with one another, resulting in the formation of AgCl precipitate.

What happens when we add an aqueous solution of AgNO3 to an aqueous solution of NaCl?

An aqueous solution that resists changes in pH upon the addition of an acid or base. It is made either from a weak acid and its salt, which consists of the conjugate base and a cation, or from a weak base and its salt, which consists of the conjugate acid and an anion.

What is a buffer solution? How is a buffer solution made?

(Absorbant) The medium in which a sample is placed on for chromatography. (Carrier) The substance that pushes the sample through the stationary phase in chromatography.

What is a stationary phase? What is a mobile phase?

The strengths of adhesion of the components in the analyte for the stationary and mobile phase. The property of how well a component of the analyte dissolves into the mobile phase. The property of how well a component of the analyte sticks to the stationary phase.

What is affinity in chromatography? What is solubility in chromatography? What is absorption in chromatography?

A thermodynamic process in which the pressure of the system does not change.

What is an isobaric thermodynamic process?

A thermodynamic process in which the volume of the system does not change.

What is an isochoric thermodynamic process?

A thermodynamic process in which the temperature in the system does not change, and by extension, the internal energy of the system does not change.

What is an isothermal thermodynamic process?

It is the amount of spontaneous dispersal of energy at a certain temperature in a system. The disorderliness of energy in a system.

What is entropy?

Heating above a phase transition temperature without the occurrence of the phase change. It occurs when the gas bubbles in a liquid are unable to overcome the surface tension and ambient pressure to break through the surface of the liquid.

What is superheating?

Answer: A. The first task in naming a compound is identifying the longest carbon chain. In this case, the longest chain has seven carbons, so the parent alkane ends in -heptane. (B.) and (D.) can therefore be eliminated. Then, we must make sure that the carbons are numbered so that the substituents' position numbers are as small as possible. This compound has two methyl groups; minimizing their position numbers requires us to number the chain from right to left. These methyl groups are attached to carbons 2 and 5, so the correct IUPAC name is 2,5-dimethylheptane. (C.) is incorrect because the position numbers of the substituents are not minimized.

What is the IUPAC name for the following structure? A. 2,5-dimethylheptane B. 2-ethyl-5-methylhexane C. 3,6-dimethylheptane D. 5-ethyl-2-methylhexane

Answer: D. We begin by finding the longest carbon chain; because there are no non-alkyl groups, we don't need to worry about any other group priorities. We then number our carbons such that the lowest possible combination of numbers is given the various substituents. Then substituents are organized alphabetically, not numerically - eliminating (C.).

What is the IUPAC name for the following structure? A. 4-isopropyl-2-methylhexane B. 3-isopropyl-5-methylhexane C. 2,2,5-trimethyl-3-ethylhexane D. 3-ethyl-2,2,5-trimethylhexane

Answer: D. This question requires the application of the acid dissociation constant. Weak acids do not dissociate completely; therefore, all three species that appear in the balanced equation will be present in the solution. Hydrogen ions and conjugate base anions dissociate in equal amounts, so [H+] = [XO2-]. If the initial concentration of HXO2 was 2 M and some amount x dissociates, we will have x amount of H3O+ and XO2- at equilibrium, with 2 M - x amount of HXO2 at equilibrium. HXO2 + H2O <-----> H3O+ + XO2- 2M 0 0 -x +x +x 2M - x x x Ka = [H3O+][XO2-]/[HXO2] = [x][x]/[2-x] x<<1 Ka = x^2/2 3.2 x 10^-5 = x^2/2 x^2 = 6.4 x 10^-5 = 8 x 10^-3 M

What is the [H3O+] of a 2 M aqueous solution of a weak acid HXO2 with Ka = 3.2 x 10^-5? A. 6.4 x 10^-5 M B. 1.3 x 10^-4 M C. 4.0 x 10^-3 M D. 8.0 x 10^-3 M

Answer: C. NaOH is a strong base; as such, there will be 1.2 x 10^-5 M OH- in solution. Based on this information alone, the pOH must be between 4 and 5, and the pH must be between 9 and 10. pOH = -log[1.2 x 10^-5] = 4.92 pH = 14 - 4.92 = 9.08

What is the approximate pH of a 1.2 x 10^-5 M aqueous solution NaOH? A. 4.92 B. 7.50 C. 9.08 D. 12.45

Answer: A. The first pKa in this curve can be estimated by eye. It is located halfway between the starting point (when no base had yet been added) and the first equivalence point (the first steep portion of the graph, around 15 mL). This point is at approximately 7-8 mL on the x-axis, which corresponds to a pH of approximately 1.9. Notice that this region experiences very little change in pH, which is the defining characteristic of a buffer region.

What is the approximate value of pKa1 in the titration curve shown? A. 1.9 B. 2.9 C. 3.8 D. 4.1

Answer: B. The value of the second pKa is found at the midpoint between the first and second equivalence points. In this curve, that corresponds to pH = 4.1. Just like the first pKa, it is in the center of a flat buffering region.

What is the approximate value of pKa2 in the titration curve shown? A. 3.6 B. 4.1 C. 5.5 D. 7.2

P = X * P° Density of water at 100°C is close to 1 g/mL, and vapor pressure of 1 atm at the same temperature. 180g C3H6O3/90 g/mol C3H6O3 = 2 mol C3H6O3 0.18 L H2O = 180 mL H2O * 1 g/mL = 180 g H2O 180 g H2O/18 g/mol H2O = 10 mol H2O 10 mol + 2 mol = 12 mol solution 10 mol H2O/12 mol solution = 0.83 P = (0.83) * (1 atm H2O) = 0.83 atm 1 atm - 0.83 atm = 0.17 atm

What is the change in vapor pressure when 180 g of glyceraldehyde (C3H6O3) are added to 0.18 L of water at 100°C?

Answer: B. Carbon monoxide, CO, has a triple bond between carbon and oxygen, with the carbon and oxygen each retaining one lone pair. In polar covalent bonds, the difference in electronegativity between the bonded atoms is great enough to cause electrons to move disproportionately towards the more electronegative atom but not great enough to transfer electrons completely. This is the case for CO. Oxygen is significantly more electronegative than carbon, so electrons will be disproportionately carried on the oxygen, leaving the carbon atom with a slight positive charge.

What is the character of the bond in carbon monoxide? A. Ionic B. Polar covalent C. Nonpolar covalent D. Coordinate covalent

A constant that characterizes how a specific material's length changes as temperature changes. A constant that characterizes how a specific material's volume changes as temperature changes. The coefficient of volumetric expansion is equal to three times the coefficient of linear expansion. (β = 3α)

What is the coefficient of linear expansion? What is the coefficient of volumetric expansion? What is the relationship between the two?

pH = 3.82 pH = -log[H3O+] 3.82 = -log[H3O+] -3.82 = log[H3O+] Take antilog: 10^-3.82 = [H3O+] = 1.5 x 10^-4 [H3O+] = 1.5 x 10^-4 M

What is the concentration of H3O+ ions if a given solution has a pH of 3.82?

-HCl ---> Cl- -CH3COOH ---> CH3COO- -H2SO4 ---> HSO4- -CH3OH ---> CH3O- -HI ---> I- -HNO3 ---> NO3- -HClO4 ---> ClO4-

What is the conjugate base of the following acids? -HCl -CH3COOH -H2SO4 -CH3OH -HI -HNO3 -HClO4

The difference in thermal/kinetic energy between two objects that determines the direction heat will flow.

What is the definition of temperature at a macroscopic level?

The average kinetic energy of the particles in a certain substance.

What is the definition of temperature at a molecular level?

1000 g/L

What is the density of water in g/L?

TLC involves small spotting and is used to identify unknown compounds at a small scale. Preparative TLC involves splitting of large spots into individual bands, and serves as a large scale means of purification.

What is the difference between TLC and preparative TLC?

A macrostate describes the general conditions of a system, like if it is organized or disorganized. A microstate describes the specific details within a given macrostate.

What is the difference between a macrostate and a microstate?

A natural process would describe the transfer of energy from a hot object to a cold object. An unnatural process would describe the transfer of energy from a cold object to a hot object.

What is the difference between a natural thermodynamic process and an unnatural thermodynamic process?

A reversible process would result in a net change in entropy of zero between the system and its surroundings. An irreversible process would result in a net increase in entropy between the system and its surroundings.

What is the difference between a reversible thermodynamic process and an irreversible thermodynamic process?

State functions are variables independent from the path taken to achieve a particular equilibrium and are properties of a given system at equilibrium; they may be dependent on one another. Process functions define the path (or how the system got to its state) through variables such as Q (heat) or W (work).

What is the difference between a state function and a process function?

Isolated system: A system in which matter and energy cannot enter or exit the system. Closed system: A system in which energy can freely enter and exit, but matter cannot. Open system: A system in which matter and energy can freely enter and exit the system.

What is the difference between an open system, a closed system, and an isolated system?

The aqueous layer is the layer that contains water and other charged species, like ions. The organic layer is the layer that does not contain water, but contains uncharged species or neutral compounds.

What is the difference between the organic layer and the aqueous layer in extractions?

: O : II .. II .. H ---- O ---- S ----O ----H .. II .. II : O :

What is the dot structure of H2SO4?

: O : (-) II II N // \\ // \\ : O : : O :

What is the dot structure of NO3-?

.. : F : I .. I .. : F ---- Si ---- F : .. I .. I : F : ..

What is the dot structure of SiF4?

.. : F : I .. I .. : F ----- Xe(+) -----F : .. .. .. / \ / \ : F : : F : .. ..

What is the dot structure of XeF5+?

Germanium's atomic number is 32, thus has a nuclear charge of 32. There are 2 inner electrons on the n=1 valence, 8 inner electrons on the n=2 valence, and 18 inner electrons on the n=3 valence, thus it's electron shielding is 2 + 8 + 18 = 28 Z(eff) = 32 - 28 = +4 Z(eff) = +4

What is the effective charge of germanium?

Lithium's atomic number is 3, thus has a nuclear charge of 3 There are 2 inner electrons shielding the valence electrons Z(eff) = 3 - 2 = +1 Z(eff) = +1

What is the effective charge of lithium?

[He] 2s(2) 2p(8)

What is the electron configuration of a sodium (Na+) cation?

[Ar] 4s(2) 3d(4)

What is the electron configuration of an iron (II) (Fe(2+)) cation?

[Ar] 4s(2) 3d(10) 4p(1)

What is the electron configuration of gallium?

[He] 2s(2) 2p(4)

What is the electron configuration of oxygen?

[Ne] 3s(1)

What is the electron configuration of sodium

0%, since the solution consists of half of one enantiomer and half of the other enantiomer? 100%, since the solution does not have both enantiomers. 40% favoring R enantiomers, subtract 30% of S enantiomers from 70% of R enantiomers, and you will have an enantiomeric excess of 40%.

What is the enantiomeric excess of a racemic mixture? What is the enantiomeric excess of a solution containing only one enantiomer? What is the enantiomeric excess of a solution consisting of 70% of R enantiomers and 30% of S enantiomers?

n=1 E(n) = E1/(n^2) E1 = -2.18 x 10^-18 J -2.18 x 10^-18 J/1.6 x 10^-19 J = -13.6 eV E(n) = -13.6 eV/(1^2) E(n) = -13.6 eV n=2 E(n) = E1/(n^2) E1 = -2.18 x 10^-18 J -2.18 x 10^-18 J/1.6 x 10^-19 J = -13.6 eV E(n) = -13.6 eV/(2^2) E(n) = -3.4 eV n=3 E(n) = E1/(n^2) E1 = -2.18 x 10^-18 J -2.18 x 10^-18 J/1.6 x 10^-19 J = -13.6 eV E(n) = -13.6 eV/(3^2) E(n) = -1.51 eV

What is the energy of a valence electron at an energy level of n=1 in electron volts? What is the energy of a valence electron at an energy level of n=2 in electron volts? What is the energy of a valence electron at an energy level of n=3 in electron volts?

n=1 E(n) = E1/(n^2) = -2.18 x 10^-18 J/(1^2) E(n) = -2.18 x 10^-18 J n=2 E(n) = E1/(n^2) = -2.18 x 10^-18 J/(2^2) E(n) = -5.45 x 10^-19 J n=3 E(n) = E1/(n^2) = -2.18 x 10^-18 J/(3^2) E(n) = -2.42 x 10^-19 J

What is the energy of a valence electron at an energy level of n=1 in joules? What is the energy of a valence electron at an energy level of n=2 in joules? What is the energy of a valence electron at an energy level of n=3 in joules?

ΔU = Q + W ΔU = change in internal energy of a system Q = heat added to the system W = work done on the system (-W if work is done by the system)

What is the equation that describes the First Law of Thermodynamics.

Answer: C. Q = mcΔT Q = 100W/10 min (Watts is equal to Joules per second, so Joules will equal Watts times seconds) 100 W * 10 min * 60 s = 60000 J Q = 60000 J m = 3 kg c = 500 J/kg*K Ti = 20°C 60000 J = 3 kg * 500 J/kg*K * (Tf - 20°C) 60000 J = 1500 J/K * (Tf - 20°C) 40°C = Tf - 20°C Tf = 60°C

What is the final temperature of a 3 kg wrought iron fireplace tool that is left in front of an electric heater, absorbing heat energy at a rate of 100 W for 10 minutes? Assume the tool is initially at 20°C and that the specific heat of wrought iron is 500 J/kg*K. A. 40°C B. 50°C C. 60°C D. 70°C

y=1/x or P=1/v

What is the function in an isothermal curve in a PV diagram?

H = 1 x 2 = 2 g/mol S = 32.1 g/mol O = 16 x 4 = 64 g/mol 64 + 32.1 + 2 = 98.1 g/mol GEW = Molar mass/n GEW = 98.1 g/mol/2 = 49.05 6/mol

What is the gram equivalent weight (GEW) of sulfuric acid?

H2SO4 has 2 H+ groups, thus has the ability to produce 2 acid equivalents. So divide the molar mass by the number of equivalents: 98 g/mol/2 mol = 49 g Gram equivalent weight = 49 g

What is the gram equivalent weight of H2SO4? Molar mass H2SO4 = 98 g/mol.

Answer: B. Gram equivalent weight is the weight (in grams) that releases 1 acid or base equivalent from a compound. Because H3PO4 contains 3 protons, we find the gram equivalent weight by dividing the mass of one mole of the species by 3. The molar mass of phosphoric acid is 98 g/mol, so gram equivalent weight is 32.7 g.

What is the gram equivalent weight of phosphoric acid? A. 24.5 g B. 32.7 g C. 49.0 g D. 98.0 g

Answer: C. When n=3, l=0, 1, or 2. The highest value for l in this case is 2, which corresponds to the d subshell. Although the 3d block appears to be part of the fourth period, it still has the principal quantum number n=3. In general, the subshells within an energy shell increases in energy as follows: s < p < d < f (although there is no 3f subshell).

What is the highest-energy orbital of elements with valence electrons in the n = 3 shell? A. s-orbital B. p-orbital C. d-orbital D. f-orbital

Answer: A. This molecule features an anhydride. The only other groups are hydrocarbon chains, which will provide part of the name of the parent root. Keep in mind that when a carbonyl group is present with a leaving group, the larger functional group (carboxylic acid, anhydride, ester, amide) takes priority over the carbonyl group alone. This molecule is propanoic anhydride.

What is the highest-priority functional group in this molecule? A. Anhydride B. Carbonyl C. Ketone D. Alkyl chain

Answer: C. For any value of n, there will be a maximum of 2n^2 electrons; that is, two per orbital. This can also be determined from the periodic table. There are only two elements (H and He) that have valence electrons in the n=1 shell. Eight elements (Li to Ne) have valence electrons in the n=2 shell. This is the only equation that matches the pattern.

What is the maximum number of electrons allowed in a single atomic energy level in terms of the principal quantum number n? A. 2n B. 2n + 2 C. 2n^2 D. 2n^2 + 2

C6H6 H = 1 g/mol C = 12 g/mol 1 g/mol * 6 = 6 g/mol H6 12 g/mol * 6 = 72 g/mol C6 72 + 6 = 78 g/mol MW = 78 g/mol C6 = 72 g/mol H6 = 6 g/mol %mass = 72 g/mol/78 g/mol * 100% = 92.3%

What is the molar mass of benzene? What percent of benzene is carbon?

H2O: Molar mass H2O = 2(atomic mass H) + (atomic mass of O) Molar mass H2O = 2(1) + 16 = 18 g/mol Glucose (C6H12O6): Molar mass glucose = 6(atomic mass C) + 12(atomic mass H) + 6(atomic mass O) Molar mass glucose = 6(12) + 12(1) + 6(16) = 180 g/mol

What is the molar mass of water? What is the molar mass of glucose?

H = 1 g/mol O = 16 g/mol MW = 16 + 2(1) = 18 g/mol MW = 18 g/mol O = 16 g/mol H2 = 2 g/mol %mass = 16 g/mol/18 g/mol * 100% = 88.89%

What is the molar mass of water? What percent of water is oxygen?

Zn(OH)2 (s) <---> Zn(2+) (aq) + 2OH- (aq) I 0 0.1 C -x +x +2x E x 0.1 + 2x Ksp = [Zn(2+)] * [2OH-]^2 4.1 x 10^-17 = [x] * [01. + 2x]^2 Since Ksp < 10^-4, we assume x<<1 4.1 x 10^-17 = x * [0.1]^2 4.1 x 10^-17 = x * 0.01 0.01 0.01 4.1 x 10^-15 = x [Zn(2+)] = x = [Zn(OH)2] Molar Solubility Zn(OH)2 = 4.1 x 10^-15 M

What is the molar solubility of Zn(OH)2 (Ksp = 4.1 x 10^-17) in a 0.1 M solution of NaOH?

Answer: C. The simplest approach is to determine the molar mass of the empirical formula. B2H5 has a molar mass of 26.6 g/mol. A molecular formula is always a multiple of the empirical formula; doubling this quantity will result in the molar mass in the question stem. Therefore, the compound must be B4H10.

What is the molecular formula of a compound with an empirical formula of B2H5 and a molar mass of 53.2 g/mol? A. B2H5 B. B3H7 C. B4H10 D. B6H15

Trigonal Planar

What is the molecular geometry of BF3?

Linear

What is the molecular geometry of CO2?

T-shaped

What is the molecular geometry of ClF3?

Bent/Angular

What is the molecular geometry of H2O?

Linear

What is the molecular geometry of I3-?

Trigonal Pyramidal

What is the molecular geometry of NH3?

Bent/Angular

What is the molecular geometry of SO2?

Square Planar

What is the molecular geometry of XeF4?

Shape: Linear Angles: 180°

What is the molecular geometry of this molecule? What are the bond angles of this molecule?

Shape: Octahedral Angles: 90°

What is the molecular geometry of this molecule? What are the bond angles of this molecule?

Shape: Tetrahedral Angles: 109.5°

What is the molecular geometry of this molecule? What are the bond angles of this molecule?

Shape: seesaw Angles: 90° (axial-equatorial) 120° (equatorial-equatorial)

What is the molecular geometry of this molecule? What are the bond angles of this molecule?

Answer: B. This reaction is a classic example of a neutralization reaction, in which an acid and a base react to form a salt and, usually, water. Although this reaction also fits the criteria for a double displacement reaction, (C), in which two molecules essentially exchange ions with each other, neutralization is a more specific description.

What is the most specific characterization of the reaction shown? Ca(OH)2 (aq) + H2SO4 (aq) ---> CaSO4 (aq) + H2O (l) A. Single-displacement B. Neutralization C. Double-displacement D. Oxidation-reduction

2-butyne

What is the name of this molecule?

Acetic acid

What is the name of this molecule?

Acetic anhydride

What is the name of this molecule?

Butanal

What is the name of this molecule?

Cyclohexene

What is the name of this molecule?

Diethyl amine

What is the name of this molecule?

Diethyl ether

What is the name of this molecule?

Diethyl sulfide

What is the name of this molecule?

Ethanol

What is the name of this molecule?

Ethyl acetate

What is the name of this molecule?

Toulene/Methylbenzene

What is the name of this molecule?

acetyl chloride

What is the name of this molecule?

butyric acid (butanoic acid)

What is the name of this molecule?

isobutyl

What is the name of this substituent?

methyl

What is the name of this substituent?

n-butyl

What is the name of this substituent?

n-propyl

What is the name of this substituent?

neopentyl

What is the name of this substituent?

sec-butyl

What is the name of this substituent?

tert-butyl

What is the name of this substituent?

NaCl (aq) + AgNO3 (aq) ---> AgCl (s) + NaNO3 (aq) Na+ + Cl- + Ag+ + NO3- ---> AgCl + Na+ + NO3- Ag+ (aq) + Cl- (aq) ---> AgCl (s)

What is the net ionic equation of the combination of an aqueous solution of NaCl and an aqueous solution of AgNO3?

Find oxidation numbers -3 +1 +6 -2 0 +3 -2 +1 -2 (NH4)2Cr2O7 (aq) ---> N2 (g) + Cr2O3 (s) + 4H2O (g) +2 +12 -14 +6 -6 Divide into half reactions: 2NH4+ ---> N2 + 8H+ Cr2O7 + 8H+ ---> Cr2O3 + 4H2O Combine half reactions: 2NH4+ + CrO7(2-) ---> N2 + Cr2O3 + 4H2O

What is the net ionic equation of the following reaction? (NH4)2Cr2O7 (aq) ---> N2 (g) + Cr2O3 (s) + 4H2O (g)

Find oxidation states: +1 -1 +1 -1 +1 -1 +1 -1 AgNO3 (aq) + HCl (aq) ---> HNO3 (aq) + AgCl (s) Separate compounds: Ag+ + NO3- + H+ + Cl- ---> H+ + NO3- + AgCl Cancel out spectator ions: Ag+ + Cl- ---> AgCl

What is the net ionic equation of the following reaction? AgNO3 (aq) + HCl (aq) ---> HNO3 (aq) + AgCl (s)

Find oxidation states: -4 +1 0 +4 -2 +1 -2 CH4 (g) + 2O2 (g) ---> CO2 (g) + 2H2O (l) Divide into half reactions: CH4 + 2H2O ---> CO2 + 8H+ + 8e- 2O2 + 8H+ + 8e- ---> 4H2O Combine half reactions: CH4 + 2O2 ---> CO2 + 2H2O

What is the net ionic equation of the following reaction? CH4 (g) + 2O2 (g) ---> CO2 (g) + 2H2O (l)

Find oxidation numbers: 0 0 +1 -1 H2 + F2 ---> 2HF Split into half reactions: H2 ---> 2H+ + 2e- F2 + 2e- ---> 2F- Combine half reactions: H2 + F2 ---> 2H+ + 2F-

What is the net ionic equation of the following reaction? H2 (g) + F2 (g) ---> 2HF (aq)

Zn + CuSO4 ---> Cu + ZnSO4 Zn + Cu(2+) + SO4(2-) ---> Cu + Zn(2+) + SO4(2-) Zn + Cu(2+) ---> Cu + Zn(2+)

What is the net ionic equation of the following reaction? Zn (s) + CuSO4 (aq) ---> Cu (s) + ZnSO4 (aq)

Mg(OH)2 has 2 base equivalents n = 2 2M * 2 = 4 N = 4

What is the normality of a 2M Mg(OH)2 solution?

Optical Activity

What is the one physical property that a pair of enantiomers will not share with one another?

Molecular Formula

What is the only property that is shared between structural isomers?

Answer: C. In NaClO (sodium hypochlorite), sodium carries its typical +1 charge, and oxygen carries its typical -2 charge. This means that the chlorine atom must carry a +1 charge in order to balance the overall charge of zero.

What is the oxidation number of chlorine in NaClO? A. -1 B. 0 C. +1 D. +2

Henderson-Hasselbalch equation: pH = pKa + log([A-]/[HA]) NH3 = A- NH4+ = HA pKa = -log[Ka] find Ka using Kb Ka * Kb = Kw Ka * 1.8 x 10^-5 = 1.0 x 10^-14 Ka = 1.0 x 10^-14/1.8 x 10^-5 = 5.6 x 10^-10 pKa = -log[5.6 x 10^-10] = 9.25 pH = 9.25 + log(0.24/0.20) pH = 9.25 + log(1.20) = 9.25 + 0.080 = 9.33 pH = 9.33 pH = 9.25 + log9

What is the pH of a buffer solution that is 0.24 M NH3 and 0.20 M NH4+Cl-? (Kb of NH3 = 1.8 x 10^-5)

Normality = N Volume = V CH3COOH: M(i) * V(i) = M(f) * V(f) M(f) = M(i) * V(i)/V(f) V(f) = 1 L + 500 mL M(f) = (0.05 M * 1 L)/1.5 L = 0.033 M CH3COO-: M(i) * V(i) = M(f) * V(f) M(f) = M(i) * V(i)/V(f) M(f) = (1 M * 0.5 L)/1.5 L = 0.33 M Use Henderson-Hasselbach equation: pH = pKa + log[A-]/[HA] pKa = -log[Ka] = -log[1.8 x 10^-5] = 4.74 pH = 4.74 + log(0.33/0.033) = 4.74 + 1 = 5.74 pH = 5.74

What is the pH of a solution made from 1 L of 0.05 M acetic acid (CH3COOH, Ka = 1.8 x 10^-5) mixed with 500 mL of 1 M acetate (CH3COO-)?

Answer: D. The question is asking for the pH, but because of the information given, we must first find the pOH and then subtract it from 14 to get the pH. Use the Henderson-Hasselbalch equation: pOH = pKb + log ([B-]/[BOH]) = 3.45 + log(70 mM/712 mM) = 3.45 - 1 = 2.45 If pOH = 2.45, the pH = 14 - 2.45 = 11.55

What is the pH of a solution with an ammonium concentration of 70 mM and an ammonia concentration of 712 mM? (Note: The pKb of ammonia is 3.45). A. 2.45 B. 4.45 C. 9.55 D. 11.55

[OH-] = 2.1 x 10^-3 M pOH = -log[OH-] pOH = -log[2.1 x 10^-3] pOH = 2.68 pH + pOH = 14 pH = 14 - pOH = 14 - 2.68 = 11.32 pH > 7 --> basic pH = 11.32 and the solution is basic

What is the pH of an aqueous NH3 solution with a concentration of OH- at 2.1 x 10^-3 M? Is the solution acidic, basic, or neutral?

pH = -log[H3O+] [H3O+] = 1.0 x 10^-7 pH = -log[1.0 x 10^-7] = 7 pH = 7.00

What is the pH of water?

K = 39.1 * 2 = 78.2 Cr = 52 * 2 = 104 O = 16 * 7 = 112 112 +104 + 78.2 = 294.2 104/294.2 * 100% = 35.4%

What is the percent composition of chromium in K2Cr2O7?

Balanced equation: Zn + CuSO4 ---> Cu + ZnSO4 32.7 g Zn * 1 mol Zn * 1 mol Cu * 63.5 g Cu = 31.75 g Cu 65.4 g Zn 1 mol Zn 1 mol Cu 28 g Cu/31.75 g Cu * 100% = 88.2%

What is the percent yield for a reaction in which 28 g of Cu is produced by reacting 32.7 g of Zn in excess CuSO4 solution.

Answer: B. We know from the IUPAC name that we have a butanedioic acid backbone - in other words, a four-carbon backbone with carboxylic acids at either end. Adding the hydroxyl groups at carbon 2 and 3 then yields the correct measure.

What is the proper structure for 2,3-dihydroxybutanedioic acid (tartaric acid)?

It allows water to cycle in and out of the condenser to keep the condenser cool. The cooled condenser allows for vaporized liquids to be cooled, which allows them to be condensed back into a liquid.

What is the purpose of the condenser in distillation?

To break surface tension in the distilling flask and prevent superheating.

What is the purpose of the following equipment in the process of distillation? -boiling chip -ebulliator -magnetic stirrer

It is filled with inert objects like glass beads in order to increase the surface area inside the column. When the mixture vaporizes, the vapor particles will condense on the surfaces of these objects and reflux back down until the rising heat causes the liquid to evaporate again. Each time the condensate evaporates, the vapor consists of a higher proportion of the compound with the lower boiling point. By the time the top of the column is reached, only the desired product drips down into the receiving flask.

What is the purpose of the fractionating column in fractional distillation?

It allows us to lower the ambient pressure of the entire system, which allows us to lower the temperature needed to vaporize the liquids. These are typically used for compounds with considerably high boiling points.

What is the purpose of the vacuum adaptor in distillations?

When internal energy increases, temperature increases. When internal energy decreases, temperature decreases.

What is the relationship between internal energy and temperature?

The entropy of a system and its surroundings will never decrease; it will always either remain zero or increase.

What is the relationship between the entropy of a system and its surroundings for any thermodynamic process?

Sodium acetate (CH3COONa) pH > 7 basic A salt formed from a weak acid and a strong base will result in a basic solution.

What is the salt formed from an acid-base neutralization reaction between CH3COOH and NaOH? Would the pH of the solution be acidic, basic, or neutral? Explain why.

Ammonium chloride (NH4Cl) pH < 7 acidic A salt formed from a strong acid and a weak base will result in an acidic solution.

What is the salt formed from an acid-base neutralization reaction between HCl and NH3? Would the pH of the solution be acidic, basic, or neutral? Explain why.

Sodium Chloride (HCl) pH = 7 neutral A salt formed from a strong base and a strong acid will result in a neutral solution.

What is the salt formed from an acid-base neutralization reaction between HCl and NaOH? Would the pH of the solution be acidic, basic, or neutral? Explain why.

It allows us to define a universal temperature scale.

What is the significance of the Zeroth Law of Thermodynamics?

1 cal/(g)(K) or 4.184 J/(g)(K)

What is the specific heat of water? (In both cal/g*K & J/g*K).

Titrant: Strong acid Titrand: Strong base

What is the titrant and the titrand/analyte of the shown titration curve?

Titrant: Strong acid Titrand: Weak base

What is the titrant and the titrand/analyte of the shown titration curve?

Titrant: Strong base Titrand: Strong acid

What is the titrant and the titrand/analyte of the shown titration curve?

Titrant: Strong base Titrand: Weak acid

What is the titrant and the titrand/analyte of the shown titration curve?

Titrant: Weak acid Titrand: Strong base

What is the titrant and the titrand/analyte of the shown titration curve?

Titrant: Weak acid Titrand: Weak base

What is the titrant and the titrand/analyte of the shown titration curve?

Titrant: Weak base Titrand: Strong acid

What is the titrant and the titrand/analyte of the shown titration curve?

Titrant: Weak base Titrand: Weak acid

What is the titrant and the titrand/analyte of the shown titration curve?

8.314 J/molK or 0.082 atm/molK

What is the value of the ideal gas constant (R)?

P = X * P° 58 g butane /58 g/mol butane = 1 mol butane 172 g hexane/86 g/mol hexane = 2 mol hexane 1 mol butane/3 mol solution = 1/3 butane 2 mol hexane/3 mol solution = 2/3 hexane P(butane) = 1/3 * 172 kPa = 57.3 kPa P(hexane) = 2/3 * 17.6 kPa = 11.7 kPa 57.3 kPa + 11.7 kPa = 69 kPa

What is the vapor pressure at room temperature of a mixture containing 58 g butane (C4H10) and 172 g hexane (C6H14)? (Note: The vapor pressures of pure butane and pure hexane are 172 kPa and 17.6 kPa, respectively, at 25°C).

n=1 r(n) = n^2 * r1 = 1^2 * 5.3 x 10^-11 m r(n) = 5.3 x 10^-11 m mvr = nh/2π v = nh/2πmr v = 1 * 6.626 x 10^-34 J*s/2π * 9.11 x 10^-31 kg * 5.3 x 10^-11 m v = 2184124.128 m/s v = 2.18 x 10^6 m/s n=2 r(n) = n^2 * r1 = 2^2 * 5.3 x 10^-11 m r(n) = 2.12 x 10^-10 m mvr = nh/2π v = nh/2πmr v = 2 * 6.626 x 10^-34 J*s/2π * 9.11 x 10^-31 kg * 2.12 x 10^-10 m v = 1092062.064 m/s v = 1.09 x 10^6 m/s n=3 r(n) = n^2 * r1 = 3^2 * 5.3 x 10^-11 m r(n) = 4.77 x 10^-10 m mvr = nh/2π v = nh/2πmr v = 3 * 6.626 x 10^-34 J*s/2π * 9.11 x 10^-31 kg * 4.77 x 10^-10 m v = 728041.3761 m/s v = 7.28 x 10^5 m/s

What is the velocity of an electron at an energy level of n=1? What is the velocity of an electron at an energy level of n=2? What is the velocity of an electron at an energy level of n=3?

Answer: B. To determine the wavelength of the light ray, first calculate its frequency from the photoelectric effect equation: KE = E(photon) - E° E(photon) = KE + E° = 50 J + 16 J = 66 J E(photon) = hc/λ λ = hc/E(photon) = 6.626 x 10^-34 J*s * 3 x 10^8 m/s/66J λ = 3.0 x 10^-27

What is the wavelength of a photon that causes an electron to be emitted from a metal with a kinetic energy of 50 J? (Note: Work function of the metal is 16 J, and h = 6.626 x 10^-34) A. 1.0 x 10^-34 m B. 3.0 x 10^-27 m C. 3.0 x 10^-26 m D. 1.0 x 10^35 m

2-1 1/λ = R * (1/(n(f)^2) - 1/(n(i)^2)) 1/λ = 1.097 x 10^7 1/m * (1/(1^2) - 1/(2^2)) 1/λ = 8227500 1/m λ = 1.22 x 10^-7 m = 122 x 10^-9 m = 122 nm Ultraviolet 3-1 1/λ = R * (1/(n(f)^2) - 1/(n(i)^2)) 1/λ = 1.097 x 10^7 1/m * (1/(1^2) - 1/(3^2)) 1/λ = 9751111.111 1/m λ = 1.03 x 10^-7 m = 103 x 10^-9 m = 103 nm Ultraviolet 4-1 1/λ = R * (1/(n(f)^2) - 1/(n(i)^2)) 1/λ = 1.097 x 10^7 1/m * (1/(1^2) - 1/(4^2)) 1/λ = 10284375 1/m λ = 9.72 x 10^-8 m = 97.2 x 10^-9 m = 97 nm Ultraviolet 4-3 1/λ = R * (1/(n(f)^2) - 1/(n(i)^2)) 1/λ = 1.097 x 10^7 1/m * (1/(3^2) - 1/(4^2)) 1/λ = 533263.8889 1/m λ = 1.875 x 10^-6 m = 1875 x 10^-9 m = 1875 nm Infrared

What is the wavelength of an electron in hydrogen going from an energy level of n=2 to n=1? Is the light emitted categorized as visible light, infrared light, or ultraviolet light? What is the wavelength of an electron in hydrogen going from an energy level of n=3 to n=1? Is the light emitted categorized as visible light, infrared light, or ultraviolet light? What is the wavelength of an electron in hydrogen going from an energy level of n=4 to n=1? Is the light emitted categorized as visible light, infrared light, or ultraviolet light? What is the wavelength of an electron in hydrogen going from an energy level of n=4 to n=3? Is the light emitted categorized as visible light, infrared light, or ultraviolet light?

sp hybrid orbitals

What kind of hybrid orbitals are shown in the picture?

sp2 hybrid orbitals

What kind of hybrid orbitals are shown in the picture?

sp3 hybrid orbitals

What kind of hybrid orbitals are shown in the picture?

Dipole-dipole interactions

What kind of intermolecular force is being demonstrated in the picture?

Hydrogen bonding

What kind of intermolecular force is being demonstrated in the picture?

London dispersion forces

What kind of intermolecular force is being demonstrated in the picture?

dipole-dipole interactions

What kind of intermolecular force is being demonstrated in the picture?

hydrogen bonding

What kind of intermolecular force is being demonstrated in the picture?

london dispersion forces

What kind of intermolecular force is being demonstrated in the picture?

structural (constitutional) isomers

What kind of isomers are in the shown image?

Adiabatic compression

What kind of thermodynamic process is being shown in the PV diagram?

Adiabatic expansion

What kind of thermodynamic process is being shown in the PV diagram?

Isobaric compression

What kind of thermodynamic process is being shown in the PV diagram?

Isochoric cooling

What kind of thermodynamic process is being shown in the PV diagram?

Isochoric heating

What kind of thermodynamic process is being shown in the PV diagram?

Isothermal compression

What kind of thermodynamic process is being shown in the PV diagram?

Isothermal expansion

What kind of thermodynamic process is being shown in the PV diagram?

Polyvalent titration

What kind of titration is being shown in this graph?

Kinetic energy of molecules (motion of molecules) Potential energy of molecules (intra and intermolecular forces) Thermal energy

What kinds of energies make up the internal energy of a system?

If the enantiomer rotates plane-polarized light to the right.

What makes an enantiomer dextrorotatory?

If the enantiomer rotates plane-polarized light to the left.

What makes an enantiomer levorotatory?

The two solvents must be immiscible and must have different polarity or acid-base properties that allow a compound of interest to dissolve more easily in one than the other.

What must be true about the two solvents used for an extraction to work?

Each of these methods separates compounds using charge and polarity.

What properties of molecules do thin-layer chromatography, paper chromatography, and standard column chromatography take advantage of to separate compounds?

All CO3(2-) (carbonates) are insoluble except those formed with alkali metals and ammonium.

What to the rules of solubility say in regards to carbonates (CO3(2-))?

All hydroxides are insoluble except for those formed with alkali metals, NH4+ (ammonium), Ca(2+) (calcium ion), Sr(2+) (strontium ion), or Ba(2+) (barium ion)

What to the rules of solubility say in regards to hydroxides?

All PO4(3-) (phosphates) are insoluble except those formed with alkali metals and ammonium.

What to the rules of solubility say in regards to phosphates (PO4(3-))?

All S(2-) (sulfides) are insoluble except those formed with alkali metals and ammonium.

What to the rules of solubility say in regards to sulfides (S(2-))?

All SO3(2-) (sulfites) are insoluble except those formed with alkali metals and ammonium.

What to the rules of solubility say in regards to sulfites (SO3(2-))?

Polar covalent

What type of bond is shown between the oxygen and hydrogen atom?

Ionic

What type of bond is shown between the sodium and chloride atoms?

Nonpolar covalent

What type of bond is shown between the two carbon atoms?

Condensation

What type of phase change involves transitioning from a gas to a liquid?

Deposition

What type of phase change involves transitioning from a gas to a solid?

Boiling/evaporation/vaporization

What type of phase change involves transitioning from a liquid to a gas?

Freezing/solidification/crystallization

What type of phase change involves transitioning from a liquid to a solid?

Sublimation

What type of phase change involves transitioning from a solid to a gas?

Melting/fusion

What type of phase change involves transitioning from a solid to a liquid?

Decomposition Reaction

What type of reaction is the following considered? (NH4)2Cr2O7 (aq) ---> N2 (g) + Cr2O3 (s) + 4H2O (g)

Disproportionation Reaction

What type of reaction is the following considered? 2H2O2 (aq) ---catalase---> 2H2O (l) + O2 (g)

Double-Displacement/Metathesis Reaction

What type of reaction is the following considered? AgNO3 (aq) + HCl (aq) ---> HNO3 (aq) + AgCl (s)

Combustion Reaction

What type of reaction is the following considered? CH4 (g) + 2O2 (g) ---> CO2 (g) + 2H2O (l)

Combination Reaction

What type of reaction is the following considered? H2 (g) + F2 (g) ---> 2HF (aq)

Extractions A. Separation funnel B. Stopcock C. Empty flask

What type of separation/purification uses the shown apparatus? What are the labeled components of the apparatus?

Fractional distillation A. Heating plate (heat source) B. Oil bath C. Distilling flask D. Fractioning column E. Thermometer F. Condenser G. Water inlet H. Water outlet I. Vacuum adaptor J. Receiving flask K. Ice bath (should be there)

What type of separation/purification uses the shown apparatus? What are the labeled components of the apparatus?

Simple distillation (specifically vacuum distillation) A. Heating plate (heat source) B. Oil bath C. Distilling flask D. Thermometer E. Condenser F. Water inlet G. Water outlet H. Vacuum adaptor I. Receiving flask J. Ice bath

What type of separation/purification uses the shown apparatus? What are the labeled components of the apparatus?

Strong acid-strong base titration

What type of titration reaction is shown in the titration curve above?

Strong acid-weak base titration

What type of titration reaction is shown in the titration curve above?

Strong base-strong acid titration

What type of titration reaction is shown in the titration curve above?

Strong base-weak acid titration

What type of titration reaction is shown in the titration curve above?

Answer: D. Hexane is less polar than ether and, therefore, is less likely to displace polar compounds absorbed into silica gel. This would decrease the distance these polar compounds would travel, decreasing Rf values.

What would be the effect on the Rf values if thin-layer chromatography (TLC) were run with hexane rather than ether as the eluent? A. No effect B. Increase tenfold C. Double D. Decrease

The light will bend in favor of the enantiomer of greater concentration, but not to the full degree of the angle.

What would be the optical activity of a mixture of unequal distribution of enantiomers?

Hydrolysis

When a salt ion is split into an acid and base by the presence of a water molecule, this reaction is referred to as a ______________ reaction.

When a solute is added to a solvent, the boiling point of the solution will be higher than the pure solvent. It is higher since the addition of a solute requires more energy to get the vapor pressure to equal the ambient pressure.

When a solute is added to a solvent, does the solution have a higher or lower boiling point. Explain why.

When a solute is added to a solvent, the freezing point of the solution will be lower than the pure solvent. It is lower since the solute particles interferes with the formation of lattice arrangement of solvent molecules in their solid state, meaning energy must be removed from the solution to solidify the solution.

When a solute is added to a solvent, does the solution have a higher or lower freezing point. Explain why.

Ionization Protonated Left-side Reactants

When a weak acid undergoes equilibria, it does not undergo complete _______________. Weak acids are not able to donate their protons well, so it will stay mostly _______________. Because of this, equilibrium will lean towards the __________-side of the reaction, in favoring the ________________.

Answer: B. Formation of complex ions between silver ions and ammonia will cause more molecules of solid AgCl to dissociate. The equilibrium is driven toward dissociation because the Ag+ ions are exxentially being removed from solution when they complex with ammonia. This rationale is based upon Le Chatelier's principle, stating that when a chemical equilibrium experiences a change in concentration, the system will shift to counteract that change.

When ammonia, NH3, is used as a solvent, it can form complex ions. For example, dissolving AgCl in NH3 will result in the complex ion [Ag(NH3)](2+). What effect would the formation of complex ions have on solubility of a compound like AgCl in NH3? A. The solubility of AgCl will increase because complex ion formation will cause more ions to exist in solutions, which interact with AgCl to cause it to dissociate. B. The solubility of AgCl will increase because complex ion formation will consume Ag+ ions and cause the equilibrium to shift away fro, solid AgCl. C. The solubility of AgCl will decrease because Ag+ ions are in complexes, and the Ag+ ions that are not complexed will associate with Cl- to form solid AgCl. D. The solubility of AgCl will decrease because complex ion formation will consume Ag+ ions and cause the equilibrium to shift toward the solid AgCl.

Bicarbonate ions (HCO3-) Carbonic acid (H2CO3) Acidic Carbonic acid (H2CO3) Bicarbonate ions (HCO3-) Basic

When an acidic substance enters our blood stream, the ________________ neutralizes the H3O+ ions, forming ________________ and water. This prevents the blood from becoming _________________. When a basic substance enters our blood stream, the ______________________ neutralizes the OH- ions, forming _________________ and water. This prevents the blood from becoming __________________.

ground energy state excited energy state temporary excited energy state ground energy state photon light photon

When an electron undergoes absorption, it will go from a ___________ energy state to a ___________ energy state. Absorption is only _________________; it will not stay in the _____________ state forever. It will eventually return to the _______________ state, which is referred to as emission. When an electron undergoes emission, it releases a ______________, which causes the electron to emit ______________. The ______________ will have a certain wavelength related to the energy levels it drops from.

Endpoint

When an indicator of a titration changes color, does it mean that the buffer solution has reached its _________________________.

Answer: B. Iron is a transition metal. Transition metals can often form more than one ion. Iron, for example, can be Fe(2+) or Fe(3+). The transition metals, in these various oxidation states can often form hydration complexes with water. Part of the significance of these complexes is that, when a transition metal can form a complex, its solubility within the related solvent will increase. The other ions given might dissolve readily in water, but because none of them are transition metals, they will not likely form complexes.

When dissolved in water, which of the following ions is most likely to form a complex ion with H2O? A. Na+ B. Fe(2+) C. Cl- D. S(2-)

For a closed system, entropy increases when thermal energy is distributed into the system at a given temperature. Likewise, entropy decreases when thermal energy is distributed out of a system at a given temperature.

When does entropy increase for a closed system? When does it decrease?

s subshell p subshell d subshell f subshell

When l=0, it indicates the presence of a _______ subshell. When l=1, it indicates the presence of a ________ subshell. When l=2, it indicates the presence of a ________ subshell. When l=3, it indicates the presence of a ________ subshell.

The work is represented by the area underneath the curve. W = P*ΔV

When looking at a PV diagram, how do you determine the work done by the gas? What is the equation that represents this area?

Molecule's name Suffix Suffix

When naming a double or triple bond, the number may precede the ____________________, or it may be placed near the _________. (2-butene or but-2-ene). If there are multiple double/triple bonds, the numbering is generally separate from the __________. (1,3-butadeine).

Di- Tri- Tetra- Directly before

When naming an organic molecule, if there are multiple substituents of the same type, we use the prefixes ______, ______, _________, and so on to indicate this. The prefixes are included _______________ ________________ the substituent's name.

Alphabetical Number Ignored Alphabetizing Not ignored Alphabetizing

When naming an organic molecule, the name of the molecule always begins with the names of the substituents in ____________________ order, with each substituent preceded by its ________________. Hyphenated prefixes like di-, tri-, tetra-, n-, and tert- are _____________ while ________________. Non-hyphenated prefixes like iso-, neo-, or cyclo- are _____________________ while ____________________.

Stable formal charges full octets Induction Aromaticity

When stability of the various resonance structures differs, the true electron density will favor the most ____________ form. Some resonance structures are favored because they do not have _______________ or form __________________ on highly electronegative atoms, like oxygen and nitrogen. Stabilization of positive and negative charges through _____________ and _________________ can also favor certain resonance structures.

An anhydride

Which functional group consists of 2 carbonyl groups connected by an ether group, with an R-group on the opposite sides of each carbonyl group?

An acyl halide

Which functional group consists of a carbonyl group in which the carbon is attached to a halide on one side, and an R-group on the other side?

An aldehyde

Which functional group consists of a carbonyl group in which the carbon is attached to a hydrogen on one side, and an R-group on the other side?

A carboxylic acid

Which functional group consists of a carbonyl group in which the carbon is attached to a hydroxyl group on one side, and an R-group on the other side?

An amide

Which functional group consists of a carbonyl group in which the carbon is attached to a nitrogen on one side, and an R-group on the other side?

An ester

Which functional group consists of a carbonyl group in which the carbon is attached to an oxygen attached to an R-group on one side, and an R-group on the other side?

A ketone

Which functional group consists of a carbonyl group in which the carbon is attached to two R-groups?

An alkyl halide

Which functional group consists of a halogen attached to an R-group?

A thiol

Which functional group consists of only an -SH group attached to an R-group?

-Aldehydes -Ketones -Carboxylic acids -Acyl halides -Anhydrides -Ester -Amide

Which functional groups contains a carbonyl group?

More stable: Bonding orbital Higher energy: Antibonding orbital

Which is more stable: a bonding orbital or an antibonding orbital? Which has higher energy?

Answer: B. Acetic anhydride is the common name for ethanoic anhydride. Methyl formate is the common name for methyl methanoate; we can infer this from the common root form- and the ester suffix -oate (which is sometimes shortened to -ate for pronunciations purposes). Formic acid is the common name for methanoic acid, but this is a carboxylic acid - not a derivative.

Which of the following are common names for carboxylic acid derivatives? I. Acetic anhydride II. Formic acid III. Methyl formate A. I and II only B. I and III only C. II and iII only D. I, II, and III

Answer: C. Aldehydes and carboxylic acids are characterized by their positions at the ends of carbon backbones and are thus considered terminal groups. As a result, the carbons to which they are attached are usually designated carbon 1. Ketones are internal by definition because there must be a carbon on either side of the carbonyl.

Which of the following are considered terminal functional groups? I. Aldehydes II. Ketones III. Carboxylic acids A. I only B. III only C. I and III only D. I, II, and III

Ag - B Al - A K - A P - A Pb - A Li - A Pu - B U - B Cu - B Cf - B Zn - B B - A N - A Np - B O - A He - A

Which of the following are representative elements (A) and which are nonrepresentative (B)? -Ag -Al -K -P -Pb -Li -Pu -U -Cu -Cf -Zn -B -N -Np -O -He

Answer: D. The only answer choice without unpaired electrons in its ground state is helium. Recall from the chapter that a diamagnetic substance is identified by the lack of unpaired electrons in its shell. A substance without unpaired electrons, like helium, cannot be magnetized by an external magnetic field and is actually slightly repelled. Elements that come at the end of a block (Group IIA, the group containing Zn, and the noble gases, most notably) have only paired electrons.

Which of the following atoms only has paired electrons in its ground state? A. Sodium B. Iron C. Cobalt D. Helium

Answer: D. The effective nuclear charge refers to the strength with which the protons in the nucleus can pull on electrons. This phenomenon helps to explain electron affinity, electronegativity, and ionization energy. In (A.), the nonionized chlorine atom, the nuclear charge is balanced by the surrounding electrons: 17 p+/17 e-. The chloride ion, (B.), has a lower effective nuclear charge because there are more electrons than protons: 17 p+/18 e-. Next, elemental potassium (C.), has the lowest effective nuclear charge because it contains additional inner shells that shield its valence electron from the nucleus. (D.), ionic potassium, has a higher effective nuclear charge than any of the other options do because it has the same electron configuration as Cl- (and the same amount of shielding from inner shell electrons as neutral Cl) but contains two extra protons in its nucleus; 19 p+/18 e-.

Which of the following atoms or ions has the largest effective nuclear charge? A. Cl B. Cl- C. K D. K+

Answer: B. Soluble hydroxides of Group IA and IIA metals are strong bases, eliminating (A.) and (D.). (B.) and (C.) are both weak bases; however, methylamine contains an alkyl group, which is electron-donating. This increases the electron density on the nitrogen in methylamine, making it a stronger (Lewis) base. Therefore, ammonia is the weakest base.

Which of the following bases is the weakest? A. KOH B. NH3 C. CH3NH2 D. Ca(OH2)

Answer: B. This answer requires an understanding of the trends that cause higher or lower bond energies. Bonds of high energy are those that are difficult to break. These bonds tend to have more shared pairs of electrons and, thus, cause a stronger attraction between the two atoms in the bonds. This stronger attraction also means that the bond length of a higher-energy, high-order bond such as a triple bond is shorter than that of its lower-energy counterparts such as single or double bonds.

Which of the following best describes an important property of bond energy? A. Bond energy increases with increasing bond length B. The more shared electron pairs comprising a bond, the higher the energy of that bond C. Single bonds are more difficult to break than double bonds D. Bond energy and bond length are unrelated

Answer: C. First recall that ammonium is NH4+ while ammonia is NH3. Ammonium is formed by the association of NH3, an uncharged molecule with a lone pair on the nitrogen, with a positively charged hydrogen cation. In other words, NH3 is a Lewis base, while H+ is a Lewis acid. This type of bonding between a Lewis acid and base is a coordinate covalent compound.

Which of the following best describes the number and character of the bonds in an ammonium cation? A. Three polar covalent bonds B. Four polar covalent bonds, of which none are coordinate covalent bonds C. Four polar covalent bonds, of which one is a coordinate covalent bond D. Four polar covalent bonds, of which two are coordinate covalent bonds

Answer: C. The limitations placed by the Heisenberg uncertainty principle are caused by limitations inherent in the measuring process: if a particle is moving, it has momentum, but trying to measure that momentum necessarily creates uncertainty in the position. Even if we had an exact definition of the meter, as in(A.), or perfect measuring devices, as in (B.), we still wouldn't be able to measure position and momentum simultaneously and exactly.

Which of the following best explains the inability to measure position and momentum exactly and simultaneously according to the Heisenberg uncertainty principle? A. Imprecision in the definition of the meter and kilogram B. Limits on accuracy of existing scientific instruments C. Error in one variable is increased by attempts to measure the other D. Discrepancies between the masses of nuclei and of their component particles

Answer: A. Because there is essentially only empty space between the Sun and the Earth, the only means of heat transfer is by radiation - electromagnetic waves that propagate across space. When a metal spoon is placed in a pot of hot soup, the molecules in the soup collide with those on the surface of the spoon, thereby transferring heat by conduction. Finally, fire warms the air above it, and the warmed air is less dense than the surrounding air, so it rises. A rising column of warm air means that heat is being transported in the air mass, which is simply the process of convection. The smoke particles ride along with the upward moving air current and create a plume of smoke.

Which of the following choices correctly identifies the following three heat transfer processes? I. Heat transferred from the Sun to the Earth II. A metal spoon heating up when placed in a pot of hot soup III. A rising plume of smoke from a fire A. I. Radiation, II. Conduction III. Convection B. I. Conduction II. Radiation III. Convection C. I. Radiation II. Convection III. Conduction D. I. Convection II. Conduction III. Radiation

Answer: A. Mixtures have a higher vapor pressure than predicted by Raoult's Law have stronger solvent-solvent and solute-solute interactions than solute-solvent interactions. Therefore, particles do not want to stay in solution and more readily evaporate, creating a higher vapor pressure than an ideal solution. Two liquids that have different properties, like hexane (hydrophobic) and ethanol (hydrophilic, small) in (A.), would not have many interactions with each other and would cause positive deviation; i.e. higher vapor pressure. (B.) and (C.) are composed of liquids that are similar to one another and would not show significant deviation from Raoult's Law. (D.) contains two liquids that would interact very well with each other, which would actually cause a negative deviation from Raoult's Law - when attracted to one another, solutes and solvents prefer to stay in liquid form and have a lower vapor pressure than predicted by Raoult's Law.

Which of the following combinations of liquids would be expected to have a vapor pressure higher than the vapor pressure that would be predicted by Raoult's Law? A. Ethanol and hexane B. Acetone and water C. Isopropanol and methanol D. Nitric acid and water

Answer: D. Ionic compounds are composed of atoms held together by ionic bonds. Ionic bonds associate charged particles with large differences in electronegativity. Rather than forming molecules or being measured by molecular weight, as in (A) and (B), ionic compounds form large arrays of ions in crystalline solids and are measured with formula weights. In ionic bonds, electrons are not really shared but rather are donated from the less electronegative atom to the more electronegative atom.

Which of the following compounds best describes ionic compounds? A. Ionic compounds are formed from molecules containing two or more atoms B. Ionic compounds are formed of charged particles and are measured by molecular weight C. Ionic compounds are formed of charged particles that share electrons equally D. Ionic compounds are three-dimensional arrays of charged particles

Answer: A. Of the compounds listed, both (B) and (D) are covalent compounds and thus are measured in molecular weights, not formula weights. The formula weight of MgCl2 is much too high, eliminating (C). Only KCl fits the criteria.

Which of the following compounds has a formula weight between 74 and 75 grams per mole? A. KCl B. C4H10O C. MgCl2 D. BF3

Answer: C. This answer choice is an example of a meso compound - a compound that contains chiral centers but has an internal plane of symmetry. Owing this internal plane of symmetry, the molecule is achiral and, hence, optically inactive. (A.) and (B.) are enantiomers of each other and will certainly show optical activity on their own. (D.), because it contains a chiral carbon and no internal plane of symmetry, is optically active as well.

Which of the following compounds is optically inactive?

Answer: D. All single bonds are σ bonds; double and triple bonds each contain one σ bond and one or two π bonds, respectively. The compounds CH4, C2H2, and C2H4 all contain at least one single bond and therefore contain at least one σ bond.

Which of the following compounds posses at least one σ bond? A. CH4 B. C2H2 C. C2H4 D. All of the above contain at least one σ bond

Answer: C. By extracting with sodium hydroxide, benzoic acid will be converted to its sodium salt, sodium benzoate. Sodium benzoate, unlike its conjugate acid, will dissolve in an aqueous solution. The aqueous layer simply has to be acidified afterward to retrieve benzoic acid. (A) is incorrect because diethyl ether and tetrahydrofuran are both nonpolar and are miscible. Hydrochloric acid will not transform benzoic acid into a soluble salt, so (B) is incorrect. Finally, (D) is incorrect because protonated benzoic acid has limited solubility in water.

Which of the following compounds would be most effective in extracting benzoic acid from a diethyl ether solution? A. Tetrahydrofuran B. Aqueous hydrochloric acid C. Aqueous sodium hydroxide D. Water

Answer: D. The key to answering this question is to understand the types of intermolecular forces that exist in each of these molecules because larger intermolecular forces correspond to higher boiling points. Kr is a noble gas with a full octet, so the only intermolecular forces present are London dispersion forces, the weakest type of intermolecular forces. Acetone and isopropyl alcohol are both polar, so both have dipole-dipole interactions, which are stronger than dispersion forces. However, isopropyl alcohol can also form hydrogen bonds, increasing its boiling point. Finally, the strongest intermolecular force is ionic bonds, which exists in potassium chloride.

Which of the following correctly ranks the compounds below by ascending boiling point? I. Acetone II. KCl III. Kr IV. Isopropyl alcohol A. I < II < IV < III B. III < IV < I < II C. II < IV < I < III D. III < I < IV < II

Answer: D. This is a racemic mixture of 2-butanol because it consists of equimolar amounts of (R)-2-butanol and (S)-2-butanonl. The (R)-2-butanol molecule rotates the plane of polarized light in one direction, and the (S)-2-butanol rotates it by the same angle, but in opposite directions; as a result, no net rotation of polarized light is observed.

Which of the following does NOT show optical activity? A. (R)-2-butanol B. (S)-2-butanol C. A solution containing 1 M (R)-2-butanol and 2 M (S)-2-butanol D. A solution containing 2 M (R)-2-butanol and 2 M (S)-2-butanol

Answer: B. For an electron to gain energy, it must absorb energy from the photons to jump up to a higher energy level. There is a bigger jump between n=2 and n=6 than there is between n=3 and n=4.

Which of the following electronic transitions would result in the greatest gain in energy for a single hydrogen electron? A. An electron moves from n=6 to n=2 B. An electron moves from n=2 to n=6 C. An electron moves from n=3 to n=4 D. An electron moves from n=4 to n=3

Answer: B. This question requires knowledge of the trends of electronegativity within the periodic table. Electronegativity increases as one moves from the left to right for the same reasons that effective nuclear charge increases. Electronegativity decreases as one moves down the periodic table because there are more electron shells separating the nucleus from the outermost electrons. In this question, chlorine is the furthest toward the top-right corner of the periodic table.

Which of the following elements has the highest electronegativity? A. Mg B. Cl C. Zn D. I

Answer: B. This formula describes the number of electrons in terms of the azimuthal quantum number l, which ranges from 0 to n-1, with n being the principal quantum number.

Which of the following equations describes the maximum number of electrons that can fit a subshell? A. 2l + 2 B. 4l +2 C. 2l^2 D. 2l^2 +2

Answer: C. Melting point depresses upon solute addition, making (A.) and (B.) incorrect. Solute particles interfere with lattice formation, the highly organized state in which solid molecules align themselves. Colder-than-normal conditions are necessary to create the solid structure.

Which of the following explanations best describes the mechanism by which solute particles affect the melting point of ice? A. Melting point is elevated because the kinetic energy of the substance increases. B. Melting point is elevated because the kinetic energy of the substance decreases. C. Melting point is depressed because solute particles interfere with lattice formation. D. Melting point is depressed because solute particles enhance lattice formation.

Answer: A. The best way to approach this problem is to draw the structure of each of these molecules, then consider the electronegativity of each bond as it might contribute to an overall dipole moment. HCN is the correct answer because of the large differences in electronegativity aligned in a linear fashion. There is a strong dipole moment in the direction of nitrogen, without any other moments canceling out. HCN is the correct answer because of the large differences in electronegativity aligned in a linear fashion. There is a strong dipole moment in the direction of nitrogen, without any other moments canceling it out. Water, (B.), has two dipole moments, one from each hydrogen pointing in the direction of oxygen. The molecule is bent, and the dipole moments partially cancel out. There is a molecular dipole, but it is not as strong as in HCN. Sulfur dioxide (D.), has a similar bent configuration, and its dipole will again be smaller than that of HCN. Further, oxygen and sulfur do not have large differences in electronegativity, so even the individual bond dipoles are smaller than those in the other molecules. CCl4, (D.), has tetrahedral geometry. Although each of the individual C-Cl bonds is highly polar, the orientation of these bonds causes the dipoles to cancel each other out fully, yielding no overall dipole moment.

Which of the following has the largest dipole moment? A. HCN B. H2O C. CCl4 D. SO2

Answer: A. Beryllium has only two electrons in its valence shell. When it bonds to two hydrogens, it requires two unhybridized orbitals, meaning that its hybridizationmust be sp. Note that the presence of only single bonds does not mean that the hybridization must be sp3; this is a useful assumption for carbon, but does not apply to beryllium because of its smaller number of valence electrons. The two unhybridized p-orbitals around beryllium are empty in BeH2, which takes on the linear geometry characteristic of sp-hybridized orbitals.

Which of the following hybridizations does the Be atom in BeH2 assume? A. sp B. sp2 C. sp3 D. sp3d

Answer: D. An amphoteric species is one that can act either as an acid or a base, depending on the environment. Proton transfers are classic oxidation-reduction reactions, so (A.) and (B.) are true. (C.) is true because many amphoteric species, such as water and bicarbonate, can either donate or accept a proton. (D.) is false, and thus the correct answer because amphoteric species can be either polar or nonpolar in nature.

Which of the following is NOT a characteristic of an amphoteric species? A. Amphoteric species can act as an acid or a base, depending on its environment B. Amphoteric species can act as an oxidizing or reducing agent, depending on its environment C. Amphoteric species are sometimes amphiprotic D. Amphoteric species are always nonpolar

Answer: B. This block represents the alkaline earth metals, which form divalent cations, or ions with a +2 charge. All of the elements in Group IIA have two electrons in their outermost s subshell. Because loss of these two electrons would leave a full octet as the outermost shell, becoming a divalent cation is a stable configuration for all of the alkaline earth metals. Although some of these elements might be great conductors, they are not as effective as the alkali metals, eliminating (A.). (C.) is also incorrect because, although forming a divalent cation is a stable configuration for the alkaline earth metals, the second ionization energy is still always higher than the first. Finally, (D.) is incorrect because atomic radii increase when moving down a group of elements because the number of electron shells increases.

Which of the following is an important property of the group of elements shaded in the periodic table above? A. These elements are the best electrical conductors in the periodic table B. These elements form divalent cations C. The second ionization energy for these elements is lower than the first ionization energy D. The atomic radii of these elements decrease as one moves down the column

Answer: B. First, convert the concentration to 5 x 10^-3 M. Next, because sulfuric acid is a strong acid, we can assume that, for the majority of sulfuric acid molecules (although not all), both portions will dissociate. The concentration of hydrogen ions is therefore 2 x 5 x 10^-3, or 10^-2. The equation for pH is pH = -log[H+]. If [H+] = 10^-2 M, then pH = 2.

Which of the following is closest to the pH of a solution containing 5 mM H2SO4? A. 1 B. 2 C. 3 D. 4

Answer: D. A Bronsted-Lowry base is defined as a proton acceptor. Ammonia, fluoride, and water - (A.), (B.), and (C.), respectively - each accepts a proton. (D.), HNO2, is a far better Bronsted-Lowry acid, donating a proton to solution.

Which of the following is not a Bronsted-Lowry base? A. NH3 B. F- C. H2O D. HNO2

Answer: B. State functions are any that are independent of the path taken to achieve a given state and which are not themselves defined as a process, such as pressure, density, temperature, volume, enthalpy, internal energy, Gibbs free energy, and entropy. Heat and work are process functions that are pathway dependent.

Which of the following is not a state function? A. Internal energy B. Heat C. Temperature D. Entropy

Answer: C. Electronegative atoms bonded to hydrogen disproportionately pull covalently bonded electrons towards themselves, which leaves hydrogen with a partial positive character. That partial positive charge is attracted to nearby negative or partial negative charges, such as those on other electronegative atoms.

Which of the following is the best explanation of the phenomenon of hydrogen bonding? A. Hydrogen has a strong affinity for holding onto valence electrons B. Hydrogen can only hold two valence electrons C. Electronegative atoms disproportionately carry shared electron pairs when bonded to hydrogen D. Hydrogen bonds have ionic character

Answer: C. The reaction in this question shows a water molecule, which has two lone pairs of electrons on the central oxygen, combining with a free hydrogen cation. The resulting molecule H3O+ has formed a new bond between H+ and H2O. This bond is created via the sharing of one of oxygen's lone pairs with the free H+ ion. This represents the donation of a shared pair of electrons from a Lewis base (H2O) to a Lewis acid (H+, electron acceptor). This type of bond is called a coordinate covalent bond.

Which of the following is the best name for the new bond formed in the reaction shown? A. Nonpolar covalent B. Ionic bond C. Coordinate covalent bond D. Hydrogen bond

Answer: A. Remember that when electrons are removed from an element, forming a cation, they will be removed from the subshell with the highest n-value first. Zn has 30 electrons, so it would have an electron configuration of 1s(2)2s(2)2p(6)3s(2)3p(6)4s(2)3d(10). The 4s subshell has the highest principal quantum number, so it is emptied first, forming 1s(2)2s(2)2p(6)3s(2)3p(6)4s(0)3d(10). (B.) implies the electrons are pulled out of the d-orbital, (C.) presents the configuration of the uncharged zinc atom, and (D.) shows the configuration that would exist if four electrons were removed.

Which of the following is the correct electron configuration for Zn(2+)? A. 1s(2)2s(2)2p(6)3s(2)3p(6)4s(0)3d(10) B. 1s(2)2s(2)2p(6)3s(2)3p(6)4s(2)3d(8) C. 1s(2)2s(2)2p(6)3s(2)3p(6)4s(2)3d(10) D. 1s(2)2s(2)2p(6)3s(2)3p(6)4s(0)3d(8)

Answer: D. A net ionic equation represents each of the aqueous ions comprising the reactants and products as individual ions, instead of combining them as a formula unit. Thus, (A.) is not a net ionic reaction. The term net means that the correct answer does not include any spectator ions (ions that do not participate in the reaction). In this reaction, nitrate (NO3-) remains unchanged. Therefore, (B.) and (C.) are eliminated.

Which of the following is the correct net ionic reaction for the reaction of copper with silver(I) nitrate? A. Cu + AgNO3 ---> Cu(NO3)2 + Ag B. Cu + 2Ag+ + 2NO3- ---> Cu(2+) + 2NO3- + 2Ag C. 2Ag+ + 2NO3- ---> 2NO3- + 2Ag+ D. Cu + 2Ag+ ---> Cu(2+) + 2Ag

Answer: A. S: 32 g/mol O: 16 x 4 = 64 g/mol H: 1 x 2 = 2 g/mol 64 + 32 + 2 = 98.

Which of the following is the gram equivalent weight of H2SO4 with respects to protons? A. 49.1 g B. 98.1 g C. 147.1 g D. 196.2 g

Answer: A. The common name of ethanal is acetaldehyde, the common name of methanal is formaldehyde, and the common name of ethanol is ethyl alcohol. Isopropyl alcohol is the common name of 2-propanol. Propionaldehyde is the common name of propanol.

Which of the following lists the correct common names for ethanal, methanal, and ethanol, respectively. A. Acetaldehyde, formaldehyde, ethyl alcohol B. Ethyl alcohol, propionaldehyde, isopropyl alcohol C. Ethyl alcohol, formaldehyde, acetaldehyde D. Isopropyl alcohol, ethyl alcohol, formaldehyde

Answer: C. The definition of an empirical formula is a formula that represents a molecule with the simplest ratio, in whole numbers, of the elements comprising the compound. In this case, given the empirical formula CH, any molecule with carbon and hydrogen atoms in a 1:1 ratio would be accurately represented by this empirical formula. (C) has three carbon atoms and four hydrogen atoms. Both its molecular and empirical formulas would be C3H4 because this formula represents the smallest whole-number ratio of its constituent elements.

Which of the following molecules cannot be expressed by the empirical formula CH? A. Benzene B. Ethyne C. H..........................H .........\......................./ ...........C == C == C ........./......................\ .......H........................H D. .........\\.......................... ...........\\..........//.........\\ ..........................\------/

Answer: B. To answer this question, one must understand the contribution of resonance structures to average formal charge. In (B.), there are three possible resonance structures. Each of the three oxygen atoms carries a formal charge of -1 in two out of the three structures. This averages to approximately -2/3 charge on each oxygen atom, which is more negative than in the other answer choices. Both water and formaldehyde, (A.) and (D.), have no formal charge on the oxygen. Ozone, (C.), has a -1/2 on two of the three oxygens and a +1 charge on the central oxygen.

Which of the following molecules contains the oxygen atom with the most negative formal charge? A. H2O B. CO3(2-) C. O3 D. CH2O

Answer: C. If a substance is undergoing a phase change, any added heat will be used toward overcoming the heat of transformation of the phase change. During the phase change, the temperature will remain constant. Temperature is a measure of the kinetic energy of the molecules in a sample, so a change in kinetic energy (A) is essentially the same thing as a change in temperature. The heat transfer by radiation described in (B) will definitely change the temperature of the solid as long as it is not in the process of melting. (D) Describes heat transfer by convection, in which the warm gas will transfer heat to the cold gas until they both reach an intermediate temperature.

Which of the following processes is least likely to be accompanied by a change in temperature? A. The kinetic energy of a gas is increased through a chemical reaction B. Energy is transferred to a solid via electromagnetic waves C. A boiling liquid is heated on a hot plate D. A warm gas is mixed with a cool gas

Answer: B. The azimuthal quantum number l cannot be higher than n-1, ruling out (A.). The m(l) number, which describes the chemical's magnetic properties, can only be an integer value between -l and +l. It cannot be equal to +/- 1 if l = 0; this would imply that an s orbital has three subshells (-1, 0, +1) when we know it can only have one. This rules out (C.) and (D.).

Which of the following quantum number sets is possible? A. n=2, l=2, m(l)=1, m(s)=+1/2 B. n=2, l=1, m(l)=-1, m(s)=+1/2 C. n=2, l=0, m(l)=-1, m(s)=-1/2 D. n=2, l=0, m(l)=1, m(s)=-1/2

Answer: A. Answering this question is simply a matter of knowing nomenclature. Acids ending in -ic are derivatives of anions ending with -ate, while acids ending in -ous are derivatives of anions ending in -ite. ClO3-, (B.), is chlorate because it has more oxygen than the other commonly occurring ion, ClO2-, which is named chlorite. Therefore, HClO3 is chloric acid. HClO4, (C.), represents chlorous acid. HClO, (D.), represents hypochlorous acid.

Which of the following represents chloric acid? A. HClO3 B. ClO3- C. HClO2 D. HClO

Answer: B. When dealing with ions, you cannot directly approach electronic configurations based on the number of electrons they currently hold. First examine the neutral atom's configuration, and then determine which electrons are removed. Due to the stability of half-filled d-orbitals, neutral chromium assumes the electron configuration of [Ar] 4s(1)3d(5). Mn must lose one electron from its initial configuration to become the Mn+ cation. That electron would come from the 4s subshell according to the rule that the first electron removed comes from the highest energy shell. Fe must lose two electrons to become Fe(2+). They'll both be lost from the same orbital; the only way Fe(2+) could hold the configuration in the question stem would be if one d-electron and one s-electron were lost together.

Which of the following species is represented by the electron configuration 1s(2)2s(2)2p(6)3s(2)3p(6)4s(1)3d(5)? I. Cr II. Mn+ III. Fe(2+) A. I only B. I and II only C. II and III only D. I, II, and III

Answer: D. The Bohr model is based on a set of postulates originally put forward to discuss the behavior of electrons in hydrogen. In summary, these postulates state that the energy levels of the electron are stable and discrete, and they correspond to specific orbits, eliminating (A.). They also state that an electron emits or absorbs radiation only when making a transition from one energy level to another, eliminating (B.). Specifically, when an electron jumps from a lower-energy orbit to a higher energy one, it must absorb a photon of light of precisely the right frequency such that the photon's energy equals the energy difference between the two orbits, eliminating (C.). When falling from a higher energy orbit to a lower-energy one, an electron emits a photon of light with a frequency that corresponds to the energy difference between the two orbits. This is the opposite of (D.), which makes it the right answer.

Which of the following statements is inconsistent with the Bohr model of the atom? A. Energy levels of the electron are stable and discrete. B. An electron emits or absorbs radiation only when making a transition from one energy level to another. C. To jump from a lower energy to a higher energy orbit, an electron must absorb a photon of precisely the right frequency such that the photon's energy equals the energy difference between the two orbits. D. To jump from a higher energy to a lower energy orbit, an electron absorbs a photon of a frequency such that the photon's energy is exactly the energy difference between the two orbits.

Answer: C. The two greatest contributors are structures I and II. Resonance structures are representations of how charges are shared across a molecule. In reality, the charge distribution is a weighted average of contributing resonance structures. The most stable resonance structures are those that minimize charge on the atoms in the molecule; the more stable the structure, the more it will contribute to the overall charge distribution in the molecule. Structures I and II minimize formal charges, so will be the largest contributors to the resonance bybrid.

Which of the following structures contribute(s) most to NO2's resonance hybrid? -1 +1 0 .. .. .. I. : O --- N === O .. .. 0 0 -1 .. .. .. II. O === N --- O : .. .. -1 +2 -1 .. . .. III. : O --- N --- O : .. .. A. I only B. III only C. I and II only D. I, II, and III

Answer: C. The two greatest contributors are structures I and II. Resonance structures are representations of how charges are shared across a molecule. In reality, the charge distribution is a weighted average of contributing resonance structures. The most stable resonance structures are those that minimize charge on the atoms in the molecule; the more stable the structure, the more it will contribute to the overall charge distribution in the molecule. Structures I and II minimize formal charges, so will be the largest contributors to the resonance hybrid.

Which of the following structures contribute(s) most to NO2's resonance hybrid? -1 +1 .. . .. I. :O --- N === O .. .. +1 -1 .. . .. II O === N --- O: .. .. -1 +2 -1 .. . .. III. :O --- N --- O: .. .. A. I only B. III only C. I and II only D. I, II, and III

Answer: C. All of the listed types of forces describe interactions between different types of molecules. However, noble gases are entirely uncharged and do not have polar covalent bonds, ionic bonds, or dipole moments. Therefore, the only intermolecular forces experienced by noble gases are London dispersion forces. Although these interactions are small in magnitude, they are necessary for condensation into a liquid.

Which of the following types of intermolecular forces provides the most accurate explanation for why noble gases can liquefy? A. Hydrogen bonding B. Ion-dipole interactions C. Dispersion forces D. Dipole-dipole interactions

Answer: C. Typically, both single-displacement and double-displacement reactions have two reactants that swap either one or two components between two species. Combination reactions, on the other hand, have more reactants than products because the reactants combine together to form the product.

Which of the following types of reactions generally have the same number of reactants and products? I. Double-displacement reactions II. Single-displacement reactions III. Combination reactions A. I only B. II only C. I and II only D. II and III only

Answer: B. The equation to determine the change in boiling point of a solution is as follows: ΔT = iKbm. m is the molarity of the solution, and Kb is the boiling point elevation constant. In this case, the solvent is always water, so Kb will be the same for each solution. What is needed is the number of dissociated particles from each of the original species. This is referred to as the van't Hoff factor (i) and is multiplied by molality to give normality (the concentration of the species of interest - in this case, all particles). The normality values determine which species causes the greatest change in boiling point. Species #moles #particles dissolved i x m (normality) CaSO4 0.4 2 0.8 Fe(NO3)3 0.5 4 2.0 CH3COOH 1.0 Either 1 or 2 Either 1.0 or 2.0 C12H22O11 1.0 1 1.0 The choice is between iron(III) nitrate and acetic acid. The fact that acetic acid is a weak acid indicates that only a few particles will dissociate into H+ and acetate. Therefore, the normality of the acetic acid will be much closer to 1.0 than 2.0.

Which of the following will cause the greatest increase in the boiling point of water when it is dissolved in 1.00 kg H2O? A. 0.4 mol calcium sulfate B. 0.5 mol iron(III) nitrate C. 1.0 mol acetic acid D. 1.0 mol sucrose

Answer: D. It is more effective to perform four successive extractions with small amounts of ether than to perform one extraction with a large amount of ether.

Which of the following would be the best procedure for extracting acetaldehyde from an aqueous solution? A. A single extraction with 100 mL of ether B. Two successive extractions with 50 mL portions of ether C. Three successive extractions with 33.3 mL portions of ether D. Four successive extractions with 25 mL portions of ether

Answer: B. The best electrolytes dissociate readily and are ionic compounds with large amounts of cations and anions. This rules out (A) and (C). (D) has fewer total ions with a smaller total magnitude of charge and therefore is not as strong an electrolyte as (B).

Which of the following would make the strongest electrolytic solution? A. A nonpolar covalent compound with significant solubility B. An ionic compound of one cation with +3 charge and three anions with -1 charge C. A polar covalent compound with a small dissociation constant D. An ionic compound composed of two cations with +1 charge and one anion with -2 charge

Answer: A. This molecule is 3-ethyl-4-methylhexane, not 2,3-diethyl-pentane. When naming alkanes, one must locate the longest carbon chain (6 carbons, rather than 5 carbons), and the numbering system should give the alkyl groups the lowest possible numbers.

Which of the two possibilities shown correctly numbers the carbon backbone of this molecule? A. Numbering Scheme 1 B. Numbering Scheme 2 C. Numbering Schemes 1 and 2 are equivalent and correct D. Numbering Schemes 1 and 2 are equivalent and incorrect

Answer: D. All three choices can make a solution as long as the two components create a mixture that is homogeneous (uniform in appearance). Hydrogen in platinum is an example of a gas in a solid. Brass and steel are examples of homogeneous mixtures of solids. The air we breathe is an example of a homogeneous mixture of gases; while these are more commonly simply referred to as mixtures, they still fit the criteria of a solution.

Which phases of solvent and solute can form a solution? I. Solid solvent, gaseous solute II. Solid solvent, solid solute III. Gaseous solvent, gaseous solute A. I and II only B. I and III only C. II and III only D. I, II, and III

Gases are considered the worst heat conductors because of the amount of space that exists between gas particles. This space between individual molecules causes energy-transferring collisions to occur relatively infrequently.

Why are gases considered to be the worst heat conductors?

Metals are the best heat conductors because of the free movement of electrons in metals due to metallic bonds. This sea of electrons allows for rapid energy transfer in metals.

Why are metals considered to be the best heat conductors?

Answer: D. Ionic bonds are formed through unequal sharing of electrons. These bonds typically occur because the electron affinities of the two bonded atoms differ greatly. For example, halogens have high electron affinities because adding a single electron to their valence shells would create full valence shells. In contrast, the alkaline earth metals have very low electron affinities and are more likely to be electron donors because the loss of two electrons would leave them in full valence shells. (A.) states the opposite and is incorrect because the halogens have high electron affinity and the alkaline earth metals have low electron affinity. (B.) is incorrect because equal sharing of electrons is a classic description of covalent bonding, not ionic. (C.) is a true statement, but is not relevant to why ionic bonds form.

Why do halogens often form ionic bonds with alkaline earth metals? A. The alkaline earth metals have much higher electron affinities than the halogens. B. By sharing electrons equally, the alkaline earth metals and halogens both form full octets. C. Within the same row, the halogens have smaller atomic radii than the alkaline earth metals. D. The halogens have much higher electron affinities than the alkaline earth metals.

Answer: B. Bond strength is determined by the degree of orbital overlap; the greater the overlap, the greater the bond strength. A π bond is weaker than a single bond because there is significantly less overlap between the unhybridized p-orbitals of a π bond (due to parallel orientation) than between the s-orbitals or hybrid orbitals of a σ bond. sp3 hybridized orbitals can be quite stable, as evidenced by the number of carbon atoms with this hybridization forming stable compounds.

Why is a single bond stronger than a π bond? I. π bonds have greater orbital overlap II. s-orbitals have more overlap than p-orbitals III. sp3 hybridization is always unstable A. I only B. II only C. I and III only D. II and III only

It heats the mixture in the distilling flask more evenly, and for maintaining a constant temperature in the distilling flask.

Why is an oil bath used for the distilling flask during distillation?

Different enatiomers of the same molecule may have different or opposite physiological effects on the body. That is why enantiomers of drugs are separated in order to form the right drug.

Why is chirality important when developing a drug?

Nickel is used to separate genetically engineered proteins that have histidine tags.

Why is nickel a commonly used stationary phase in affinity chromatography?

Because water contains no ions, other than a few hydrogen and hydroxide ions that result from water's low level of autodissociation.

Why is water a poor conductor of electricity?

Because of how high the specific heat of water is, it is able to absorb a lot of heat, and barely change that much in temperature.

Why is water often considered a great heat exchanger?

Answer: A. The energies of the subshells within a principle quantum number are as follows: s < p < d < f.

Within one principal energy level, which subshell has the least energy? A. s B. p C. d D. f

Acid dissolves better in aqueous base because it will dissociate to form the conjugate base and, being more highly charged, will become more soluble. Note that like dissolves like applies to polarity; acids and bases dissolve more easily in solutions with the opposite acid-base characteristics.

Would acid dissolve better in aqueous acid or aqueous base? Why?

Both O and O(2-) have fully filled 1s and 2s orbitals. O has four electrons in the 2p subshell; two are paired, and the other two have their own orbital. O(2-) has six electrons in the 2p subshell, all of which are paired in the three p-orbitals.

Write out and compare an orbital diagram for a neutral oxygen (O) atom and an O(2-) ion.

This is a double displacement reaction. The solid species does not split. Cu+ + NO3- + Na+ + Cl- -----> CuCl + Na+ + NO3- Cu+ + Cl- -----> CuCl

Write the net ionic equation for the following reaction: CuNO3 (aq) + NaCl (aq) -----> CuCl (s) + NaNO3 (aq)

Balance the equation: 3Mg + 2AlCl3 -----> 2Al + 3MgCl2 3Mg + 2Al(3+) + 2Cl3- -----> 2Al + 3Mg(2+) + 3Cl2- 3Mg + 2Al(3+) -----> 2Al + 3Mg(2+)

Write the net ionic equation for the following reaction: Mg (s) + AlCl3 (aq) -----> Al (s) + MgCl2 (aq)

M1V1=M2V2 100 ppm * V1 = 25 ppm * 100 mL V1 = 25 ppm * 100 mL 100 ppm V1 = 25 mL To get the final solution, start with 25 mL of the 100 ppm stock solution, and then add 75 mL of pure water to get 100 mL solution with 25 ppm of chlorine.

You are working in a sewage treatment facility and are assaying chlorine in a water sample. You need to dilute the water sample from 100 ppm stock to 25 ppm and create 100 mL of solution. Calculate the amount of stock solution needed and determine how you would create the final solution.

Distance Two

You determine the atomic radii by finding the _________________ between two identical nuclei and divide by _______.

Since this is a normal-phase TLC, the stationary phase is polar while the mobile phase is nonpolar. Benzoic acid is more polar than napthalene, which means that benzoic acid would be more attracted to the stationary phase, while napthalene is more attracted to the mobile phase. Therefore, the dot further up on the stationary phase is napthalene while the dot further down on the stationary phase is benzoic acid.

You have a reaction flask that contains napthalene and benzoic acid. You run a sample through normal-phase TLC, and you found that one dot is closer to the top of the silica plate, while another dot is further down on the plate. Identify which dot represents which compound.

Second First Second First

__________ ionization energy is greater than ___________ ionization energy since it requires more energy to remove the _____________ than the ___________ electron.

Form- Acet- Propion- Butyr-

____________ = prefix of the common name of a one-carbon structure in which that carbon is a carbonyl carbon. ___________ = prefix of the common name of a two-carbon structure in which one of those carbons is a carbonyl carbon. __________ = prefix of the common name of a three-carbon structure in which one of those carbons is a carbonyl carbon. __________ = prefix of the common name of a four-carbon structure in which one of those carbons is a carbonyl carbon.

Single Double Triple.

_____________ bonds consists of only one σ bond. _____________ bonds consists of one σ bond and one π bond. ____________ bonds consists of one σ bond and two π bonds.

Redox titrations Acid-Base titrations

_____________ titrations focuses on the transfer of electrons to reach an equivalence point, whereas _________________ titrations focus on the transfer of protons.

Hydrocarbons

______________ are compounds that contains only carbon and hydrogen atoms.

Nonbonding electron pairs Bonding electron pairs

______________ electron pairs exerts more repulsion in a molecule than ____________ electron pairs.

Bromthymol blue Strong Strong

______________ is an indicator that changes from yellow to blue in a pH range of 6-8 (6-7.6). It is commonly used for acid-base titrations that utilize a ___________ acid and a ____________ base.

Condensate Distillate

______________ refers to the liquids being cooled from vaporization in the condenser. ______________ refers to the end product of distillation.

Effective nuclear charge Nuclear charge Electron shielding Z(eff) = Z - S

_______________ ___________ ___________ is the actual amount of charge from the nucleus that is attracting the valence electrons inward. It is determined by the difference between the _____________ ___________ and ______________ _______________. __________ = ________ - _________

Resonance structures

_______________ _____________ are molecular structures that are different in the arrangement of electron pairs, but not bond connectivity or overall charge of the molecule.

Carboxylic acids

_______________ are hydrocarbons that contains both a carbonyl group (C=O) and a hydroxyl group (-OH) on a terminal carbon.

Electronic geometry Molecular geometry

_______________ geometry is the geometry of lone pairs and bonding electrons around the central atom of a molecule. _______________ geometry is the geometry of only the bonding electrons around the central atom of a molecule.

Solubility Saturation

_______________ is the amount of solute contained in a solvent. _______________ refers to maximum solubility of a compound at a given temperature.

Radiation

_______________ is the transfer of energy in the form of electromagnetic waves, and is capable of transferring energy through a vacuum.

Electron Shielding

________________ ______________ is the phenomenon where electrons from the inner shells of an atom repel the valence electrons from being attracted by the nucleus.

Bonding Nonbonding

________________ electrons are electrons in covalent bonds while ____________ electrons are lone pairs.

Phenolphthalein Weak Strong

________________ is an indicator that changes from colorless to magenta over a pH range of 8-10 (8.2-10). It is commonly used for acid-base titrations that utilize a ____________ acid and a ____________ base.

Heat

________________ is the transfer of thermal energy from an object with higher temperature/energy to an object with lower temperature/energy.

Alcohols

_________________ are hydrocarbon compounds that contains at least one hydroxyl group (-OH).

Alkanes Alkenes Alkynes Alkanes Alkenes Alkynes

_________________ are simple hydrocarbon molecules that consists of only single bonds with a chemical formula of C(n)H(2n+2). ____________________ are hydrocarbons that have a double bond in the parent chain. ____________________ are hydrocarbons that have a triple bond in the parent chain. __________________ are indicated by having the suffix -ane in the chemical name. ________________ are indicated by having the suffix -ene in the chemical name. ________________ are indicated by having the suffix -yne in the chemical name.

Polar covalent bonds Ionic bonds Metallic bonds Nonpolar covalent bonds Covalent bonds

_________________ bonds are bonds formed between atoms of slightly different electronegativities, that they unevenly share an electron pair. _________________ bonds are bonds formed by the complete transfer of valence electron(s) between atoms. _______________ bonds are covalent bonds that specifically occurs between atoms of metals, in which the valence electrons are free to move through the lattice. ________________ bonds are bonds formed between atoms of the same or very similar electronegativities, that they share an electron pair evenly. _______________ bonds are bonds formed by the sharing of valence electrons between atoms.

Geminal diols Vicinal diols Geminal diols

_________________ diols are diols that have hydroxyl groups on the same carbon. _________________ diols are diols with hydroxyl groups on adjacent carbons. _________________ diols are also known as hydrates.

Aldehyde Terminal One

_________________ have a carbonyl group at the end of a carbon chain, making it a _________________ functional group. It has precedence over multiple bonds, alcohols, and ketones. They are generally attached to carbon _________.

Methyl red Strong Weak

_________________ is an indicator that changes from red to yellow in a pH range of 4-6 (4.4-6.2). It is commonly used for acid-base titrations that utilize a _________ acid and ___________ base

Stoichiometry

_________________ is the branch of chemistry that deals with the relationship between the relative quantities of substances taking part in a chemical reaction.

Net dipole moment

_________________ is the dipole moment of all polar bonds in a molecule, resulting in the dipole moment of the entire molecule.

Work

_________________ is the process by which energy is transferred as a result of force being applied through a certain distance.

Thermodynamics

_________________ is the study of energy, the conversion of energy between different forms, and the ability for energy to do work.

Heteroatoms

__________________ are atoms that are not carbon and hydrogen, like oxygen, nitrogen, phosphorous, or halogens.

Anhydride Anhydride

__________________ are formed from two carboxylic acid molecules that results in the removal of a water molecule. Many ____________ are cyclic, which may result from intramolecular dehydration of a dicarboxylic acid.

Intramolecular forces Intermolecular forces

__________________ forces are bonding forces within a molecule that hold the atoms of a molecule together. ___________________ forces are forces of attraction between molecules.

Bond length Bond order Bond energy

__________________ is the average distance between the two nuclei of atoms in a bond. _________________ is the number of shared electron pairs between two atoms. ________________ is the energy required to break a bond by separating their components into their isolated, gaseous atomic state.

Excess reagent Limiting reagent

__________________ is the reactant in a chemical reaction that is in excess and remains unconsumed when a reaction stops because the other reactant has been completely consumed. __________________ is the reactant in a chemical reaction that limits the amount of product that is formed. The reaction will stop when all of it is consumed.

Conduction

__________________ is the transfer of energy through the collision of molecules between two objects that are in direct contact with one another.

Convection

__________________ is the transfer of heat due to the bulk movement of molecules in a fluid (liquid or gas), where hot molecules rise to the top of the fluid while cool molecules fall to the bottom of the fluid.

Specific heat

__________________ refers to the amount of heat energy necessary to raise one gram of a substance by one degree/unit Celsius/Kelvin.

partitioning

__________________ refers to the separation of components in the stationary phase.

Ester Amide Anhydride

___________________ is a hydrocarbon that is a derivative of carboxylic acid where their hydroxyl group (-OH) is replaced with an alkoxy group (-OR, where R is a hydrocarbon chain). __________________ is a hydrocarbon that is a derivative of carboxylic acid where their hydroxyl group (-OH) is replaced with an amino group (nitrogen-containing group). _________________ is a hydrocarbon that is a derivative of carboxylic acid that consists of two carbonyl groups (C=O) that are both bonded to the same oxygen atom.

mass spectrometry

___________________ is a technique based on the detection of vaporized molecules and their ionized (charged) fragments; detection and display of the spectra are based on the mass-to-mass charge ratios of the ions; method is specific for qualitative analysis and useful for quantitative analysis

Electronegativity

___________________ is the strength of an atom in a molecule to attract electrons to itself.

Diastereomers

____________________ are configurational isomers that are not mirror images of each other.

Stereoisomers

____________________ are isomers that have the same molecular formula, same structure, but different 3D arrangement of atoms..

Configurational isomers

____________________ are stereoisomers that can only interconvert by breaking a double or triple bond.

Ground state Excited state

____________________ is the energy state of an atom that in which it is at its lowest energy. ___________________ is the energy state of an atom when at least one electron has moved to a subshell of higher than normal energy.

Specific rotation

____________________ is the standardized form of observed rotation that is calculated from the observed rotation and experimental parameters

Carbonyl group

_____________________ are functional groups in which a carbon is double bonded to an oxygen.

Intramolecular Intermolecular

_____________________ forces are much stronger than _______________________ forces.

Electronegativity Atomic radii Electron affinity Ionization energy

_____________________ increases the further up and the further right you go on the periodic table. _____________________ increases the further down and further left you go on the periodic table. ________________________ (generally) increases the further right and the further up you go on the periodic table. __________________________ (generally) increases the further up and the further right you go on the periodic table.

Principal quantum number (n) Azimuthal quantum number (l) Magnetic quantum number (ml) Spin quantum number (mn)

_____________________ quantum number indicates the average energy level of a shell. It is represented by the letter _________. ____________________ quantum number describes the subshells within each principal energy level. It is represented by the letter _______. ___________________ quantum number describes the specific orbital in a subshell an electron is most likely to be found at a given moment in time. It is represented by the letter _________. ___________________ quantum number indicates the spin orientation of an electron in an orbital (either spinning in one particular direction, or spinning in the opposite of that direction. It is represented by the letter ____________.

Dipole-dipole interactions hydrogen bonding London dispersion forces

______________________ intermolecular forces occurs between partially oppositely charged ions. _____________________ intermolecular forces occurs between a partially positive hydrogen atom bonded to a nitrogen, oxygen, or fluorine atom and a partially negative nitrogen, oxygen, or fluorine atom of a neighboring molecule. _____________________ intermolecular forces occurs between temporarily induced dipoles of all molecules.

Structural formula

______________________ is the formula that shows how the molecular formula of atoms are configured

Relative configuration Absolute configuration.

______________________ is the spatial arrangement of groups in a chiral molecule when compared to another chiral molecule. __________________________ is the exact spatial arrangement of atoms or groups in a chiral molecule around a single chiral atom designated by (R) or (S).

Electronegativity Electronegativity

______________________ is the tendency for an atom to attract a bonding pair of electrons. ____________________ unlike other periodic trends is measured within a bond.

E-isomer

_______________________ is a geometric isomer in which the highest priority substituents are on different sides of a double bond or ring.

Observed rotation

_______________________ is the angle that plane-polarized light is shifted by stereoisomer.

Chiral center

_______________________ is the carbon atom in a molecule that has 4 different groups tetrahedrally bonded to it.

Fluoroescence

_______________________ is the process in which the electrons of certain substances are excited to higher energy levels by high-frequency photons, and then emit visible light as the energy is released in two or more steps back to the ground state.

Conformational Isomers

________________________ are stereoisomers that differ by rotation about one or more single bonds, usually represented using Newman projections.

Z-isomer

________________________ is a geometric isomer in which the highest priority substituents are on the same side of a double bond or ring.

Combustion reaction

________________________ is a reaction that involves a fuel - usually a hydrocarbon - and an oxidant - usually oxygen that forms two products of water and carbon dioxide.

Atomic number Atomic mass

________________________ refers to the number of protons and electrons found in a neutral atom. _______________________ refers to the total mass of both protons and neutrons present in a neutral atom.

Meso molecules

_________________________ are molecules that have a chiral center, but also a plane of symmetry.

Neutralization reaction

_________________________ is a specific type of double displacement reaction where an acid and base is used to produce a salt and usually water.

Dipole-dipole interactions Hydrogen bonding London dispersion forces

___________________________ are forces that occurs when the partially positive charged part of a molecule interacts with the partially negative charged part of the neighboring molecule. _________________________ is a special kind of dipole-dipole interaction that occurs specifically between a hydrogen atom bonded to either an oxygen, nitrogen, or fluorine atom, where the partially positive end of hydrogen is attracted to the partially negative end of the oxygen, nitrogen, or fluorine of another molecule. ________________________ is the weakest kind of intermolecular force that exists between all kinds of molecules, regardless of it being ionic or covalent - polar or nonpolar. This intermolecular force involves the brief, temporary dipole-dipole interaction between two molecules.

Double displacement/metathesis reaction

___________________________ is a reaction where elements from two different compounds swap places with each other to form two new compounds.

Axial group Equatorial group

____________________________ in cycloalkane conformational structures are atoms that are sticking up or down from the structure. ___________________________ in cycloalkane conformational structures are atoms that are sticking out from the side of the plane of the ring.

Gravity filtration Vacuum filtration

____________________________ is filtration in which the solvent's own weight pulls it through the filter. __________________________ is filtration in which the solvent is forced through a filter by a vacuum connected to the flask.

Number of particles of interest produced or consumed per molecule of the compound in the reaction.

____________________________ is the mass of a compound, measured in grams, that produces one equivalent of the particle of interest. It is determined by dividing the molar mass of the compound by n, where n = ________________________________.

Law of constant composition Percent composition

_____________________________ states that any pure sample of a given compound will contain the same elements in an identical mass ratio. _____________________________ is the percent of a specific compound that is made up of a given element.

Non-polarized light Plane-polarized light

______________________________ is light in which the electric and magnetic fields of it are randomly oriented in all planes. ____________________________ is light in which the electric and magnetic fields are oriented in only one particular direction.


Ensembles d'études connexes

Market segmentation - (Target markets & segmentation)

View Set

CITI Responsible Conduct of Research (RCR) - Basic

View Set

Unit 7 - The Securities Act of 1933 and the Primary Markets Quiz/Test Questions

View Set

Ultrasound examination review: Spleen

View Set

Oregelbundna verb: be, become, begin, break, bring, buy, catch, choose, come, do

View Set

ATI Engage Mental Health RN: Foundations of Mental Health Nursing - Foundational Concepts of Mental Health Nursing

View Set

CH 2 Introductory Maternity and Pediatric Nursing

View Set